You are on page 1of 221

Item: 1 of 48 ~ 1 • M k -<:J 1>- Jil ~· !

:';-~
QIO: 4 0 51 ..L ar Pre v ious Next Lab~lues Notes Calcula t o r

•1 & &

A 62-year-old man comes into the hospital after feeling severe substernal discomfort. He states that the pain began severa l hours ago and
•2 occasionally radiates to his left arm . He reports that his paternal grandfather died of a stroke at age 70 and that his father died of a myocardial
Infarction (MI) at age 65. He has a 20 pack-year smoking history and denies alcohol use. His pulse is 180/min, blood pressure is 80/60 mm Hg, and
•3 respiratory rate is 36/min.
·4 Advanced life support measures are initiated but are unsuccessful, and the patient dies 2 days later. A postmortem is performed, and the pathology sample is
shown.
•5
•6
•7

·8
.9
• 10
• 11
• 12
• 13
• 14
• 15
• 16
• 17
• 18 Image courtesy of Dr. Daniel Wasdahl
• 19
The Image reflects which of the following findings?
• 20
• 21
• A "''"""'h.-.U h .... ...t; ... ,.

a
Lock
s
Suspend
8
End Bl ock
•2
•3

·4
•5
•6
•7

·8
.9
• 10
Image courtesy of Dr. Daniel Wasdahl
• 11
• 12
The Image reflects which of the following findings?
• 13
• 14 A . Aschoff bodies
• 15
8. Coagulative necrosis and m onocytes
• 16
• 17 C. Coagulative necrosis with neutrophils

• 18 D. Collagen deposition
• 19
E. Granulation tissue
• 20
• 21

a
Lock
s
Suspend
8
End Block
Item: 1 of 48 ~. I • M k <:] t> al ~· ~
QIO: 4051 .l. ar Previous Next lab 'Vfl1ues Notes Calculator

1 The co rrect a nswer is c. 73% cho se this.


. 2 This patient died of a myocardial infarction (MI ). Two days after an MI, histologic inspection typically reveals extensive neutrophilic infiltrations at the site
of the infarct . Vessels in this area will dilat e (hyperemia). During this stage, the risk for arrhythmia is grea t est, as the damaged myocardium may support
•3 the development of reentrant circuits. Electrolyte abnormalities and hypoxia may also be contributing factors. Coagulative necrosis is a common fea ture
.4 of infarcted tissue (except in the brain, which undergoes liquefactive necrosis) and represents the loss of blood in the tissue. It is characterized by stiff
tissue on gross pathologic examination, and preserved cell outlines ("ghosting" ) with red discoloration on microscopic examination .
•5 Hypoxia (medical) Electrolyte Hyperaemia Cardiac muscle Cardiac arrhythmia Necrosis Infarction Histology Coagulative necrosis Water-electrolyte imbalance

•6 Brain

.7 A is no t co rrect. 7% cho se this.


Aschoff bodies are focal area s of granulomatous inflammation with necrotic centers that are found within the myocardium in rheumatic fever; not
•8 following an acute MI. Although myocarditis from rheumatic fever can result in congestive heart failure, this patient's symptoms, laboratory t ests, and
•9 ECG are more consistent with heart failure due to infarction .
Rheumatic fever Aschoff body Myocarditis Heart failure Cardiac muscle Congestive heart failure Granuloma Electrocardiography Inflammation Necrosis
• 10 Rheumatism Fever Infarction
· 11 B is no t co rrect. 16% cho se this.
• 12 Acute inflammation is most pronounced 2 - 3 days after myocardial infarction (MI ). Neutrophils are numerous within the necrotic and damaged tissue .
Mononuclear cells do not appear until 3 - 14 days after MI.
• 13 Neutrophil Necrosis Inflammation
• 14
D is no t co rrect. 2% cho se this.
• 15 Collagen deposition by fibroblasts is extensive at 2 - 3 weeks after myocardial infarction . A healed infarct contains dense fibrotic tissue that has replaced
the necrotic fibers .
• 16 Myocardial infarction Collagen Fibroblast Infarction Necrosis Fibrosis
• 17
E is no t co rrect. 2% cho se this.
• 18 Granulation tissue is hea vily vascularized and provides a leaky network for the influx for inflammatory cells. It is present 1 week after myocardial
infarction . Fibroblast proliferation is also pronounced at this stage .
• 19 Fibroblast Granulation tissue Myocardial infarction Angiogenesis Circulatory system
• 20
• 21
Rotto m 1 ir1P.:

6
lock
s
Suspend
0
End Block
Item: 1 of 48 ~. I • M k <:] t> al ~· ~
QIO: 4051 .l. ar Previous Next lab 'Vfl1ues Notes Calculator

1 E is not correct. 2 % chose this .


. 2 Granulation tissue is heavily vascularized and provides a leaky network for the influx for inflammatory cells. It is present 1 week after myocardial
infarction . Fibroblast proliferation is also pronounced at this stage.
•3 Fibroblast Granulation tissue Myocardial infarction Angiogenesis Circulatory system

.4
•5 Bottom Line:
•6 Coagulative necrosis appears within 4 hours of an acute myocardial infarction, with a neutrophilic infiltration following at about 24 hours. Neutrophils
are eventually replaced with macrophages and then contracted granulation tissue .
.7 Myocardial infarction Granulation tissue Coagulative necrosis Macrophage Necrosis Neutrophil
•8
•9
• 10 lijj ;fi IJ l•l for year:l 2017 ..
FIRST AID FAC T S
· 11
• 12
FA17p294.1
• 13
Evolution of Commonly occluded coronary arteries: LAD > RCA> circumflex.
• 14
myocardial infarction Symptoms: diaphoresis, nausea, vomiting, severe retrosternal pain, pain in left arm and/or jaw,
• 15 shortness of breath, fat igue.
• 16 TIME GROSS LIGHT MICROSCOPE COMPliCATIONS
• 17
0-24 hr one Early coagulative necrosis, Ventricular arrhythmia, HF,
• 18 release of necrotic cell cardiogenic shock.
• 19 contents into blood; edema,
hemorrhage, wavy fibers .
• 20 Occluded Neutrophils appear.
artery
• 21 Reperfusion injury,
/
6
lock
s
Suspend
0
End Block
Item: 2 of 48 ~ 1 • M k -<:J 1>- Jil ~· !:';-~
QIO: 1429 ..L ar Pre v ious Next Lab~lues Not es Calcula t o r

•1
•2
&

A 28-year-old African-American man presents with fever, weight loss, diffuse myalglas and arthralgias, and abdominal pain. On examination, his blood
pressure Is 168/92 mm Hg, pulse is 83/min, and respiratory rate is 18/min. He has areas of ulceration and areas of mottled purple discoloration on his
IA•A] &

lower extremities. His medical history is otherwise significant for hypertension and hepatitis B infection. Laboratory t ests reveal an elevated WBC
•3 count, erythrocyte sedimentation rate, and c -rea ctive protein levels.
·4
•5 What histologic feature is commonly associated with this patient's disease process?

•6
A. Caseat1ng necrosis
•7

·8 B. Eosinophilic infiltrate
.9 c. Fibrinoid necrosis
• 10
o. Granulomatous inflammation
• 11
E• Langhans giant cells
• 12
• 13 F• Onion skinning

• 14
• 15
• 16
• 17
• 18
• 19
• 20
• 21

a
Lock
s
Suspend
8
End Bl ock
Item: 2 of 48 ~ 1 • M k -<:J 1>- Jil ~· !:';-~
QIO: 1429 ..L ar Prev ious Next Lab~lues Not es Calculat o r

& &
1

2
•3
The correct an swer is c. 440/o chose this.
·4 This Individual likely has polyarteritis nodosa (PAN), which is characterized by necrotizing immune complex
•5 Inflammation of medium-sized, muscul ar arteries. PAN typically is associated with fever, malaise, weight
loss, abdominal pain, headache, myalgias, and hypertension and is more common in men than in women .
•6 There are no diagnostic serologic t ests spedfic for PAN. Diagnosis is usually made by the presence of the
above clinical symptoms, physical exam (induding skin findings, motor or sensory loss, and vascular exam
•7 abnormalities), hepatltis/ rheumatologic t est results, and sometimes biopsy or angiography. Patients with
·8 classic PAN are negative for antineutrophil cytoplasmic antibodies and may have low t1ters of rheumatoid
factor or antinuclear antibodies, both of which are nonspecific findings. In patients w1th PAN, appropriate
.9 serologic tests for active hepatitis B infection must be performed, because up to 30% of patients with PAN
are positive for hepatitis B surface antigen. I nflammation of the arterial wall and surrounding connective
• 10 tissue, resulting from immune complex formation and deposition, is associated with fibrinoid necrosis
(Indicated by open arrow in the image) and leads to the loss of elasticity.
• 11
Polyarteritis nodosa Rheumatoid factor Immune complex Hepatitis B Fibrinoid necrosis Antibody Necrosis
• 12 I m age copyright © Yuminaga et a/. Korean
Anti-nuclear antibody Anti-neutrophil cytoplasmic antibody Hepatitis Inflammation Malaise Biopsy Antigen Urol Assoc, 2011.
• 13 Hypertension Connective tissue Headache Abdominal pain Angiography Weight loss Synovitis Artery Serology http://dx.doi.org/ 10.4111 %2Fkju.2011.52 .5.364

• 14 Fever Physical examination Blood vessel

• 15 A is not co rrect . 70fo chose this .


Caseating necrosis is associated wit h t uberculosis infections. The granuloma of tuberculosis is referred to as a t ubercle and is characterized by central
• 16 caseous necrosis and often by La nghans' giant cells. These lesions often coalesce and rupture into bronchi. The caseous contents may liquefy and be
• 17 expelled, resulting in cavitary lesions.
Tuberculosis Caseous necrosis Bronchus Necrosis Granuloma Tubercle
• 18
B i s n ot co rrect . 100/o ch ose this .
• 19 Eosinophilic infiltrate is a prominent fea ture in Churg-Strauss syndrome, a necrotizing vasculitis of small- and medium-sized muscular arteries, capillaries,
• 20 veins, and venules. Patients with Churg-Strauss syndrome often present with severe asthma attacks, pulmonary infilt rates, and mononeuritis multiplex,
along with nonspecific symptoms such as weight loss, fever, malaise, and anorexia. Serum studies in patient s wit h Churg-Strauss syndrome are commonly
• 21 positive for protoplasmic-staining ant i-neut rophil cytoplasmic antibodies.

a
Lock
s
Suspend
8
End Block
Item: 2 of 48 ~. I • M k <:] t> al ~· ~
QIO: 1429 .l. ar Previous Next lab 'Vfl1ues Notes Calculator
w .. • .. • ...

1 •
Tuberculosis Caseous necrosis Bronchus Necrosis Granuloma Tubercle

2 B is no t co rrect. 10 % cho se this.


•3 Eosinophilic infiltrat e is a prominent fea ture in Churg-Strauss syndrome, a necrotizing vasculitis of small- and medium-sized muscular arteries, capillaries,
veins, and venules. Patients with Churg-Strauss syndrome often present with severe asthma attacks, pulmonary infiltrat es, and mononeuritis multiplex,
•4 along with nonspecific symptoms such as weight loss, fever; malaise, and anorexia. Serum studies in patients with Churg-Strauss syndrome are commonly
positive for protoplasmic-staining anti-neutrophil cytoplasmic antibodies.
•5
Anti-neutrophil cytoplasmic antibody Mononeuritis multiplex Asthma Vasculitis Anorexia (symptom) Churg-Strauss syndrome Necrosis Eosinophilic Antibody
•6 Capillary Peripheral neuropathy Anorexia nervosa Malaise Artery Weight loss Cytoplasm Fever Serum (blood) Mononeuropathy
•7 D is no t co rrect. 24 % cho se this.
•8 Granulomatous infiltrat e is a more prominent fea ture in giant cell arteritis (also known as t emporal arteritis), an inflammatory disea se of medium- and
large-sized arteries. Patients with giant cell arteritis are rarely younger than 5 0 years and present most commonly with severe, unilat eral hea dache .
•9 Presentation is often also characterized by j aw pain, fever; anemia, and high erythrocyte sedimentation rat e.
Giant-cell arteritis Erythrocyte sedimentation rate Red blood cell Anemia Granuloma Headache Giant cell Artery Arteritis Inflammation Jaw Fever
• 10

· 11 E is no t co rrect. 7 % cho se this.


La nghans giant cells are associat ed with granulomatosis with polya ngiits (Wegener), a small-vessel vasculitis that is associat ed with granulomas of the
• 12 upper and lower respiratory tracts together with glomerulonephritis. Many cases are associat ed with positivity for circulating antineutrophil cytoplasmic
antibodies.
• 13
Vasculitis Glomerulonephritis Anti-neutrophil cytoplasmic antibody Granuloma Antibody Giant cell Cytoplasm
• 14
F is no t co rrect. 8% cho se this •
• 15 Onion skinning, seen in arteriolosclerosis, is the type of arteriosclerosis associat ed with malignant hypertension. Even though this patient has
hypertension, he is young and unlikely to have had uncontrolled blood pressure long enough to produce this pathology. Also, his skin findings and medical
• 16
history of hepatitis B point to a different etiology, in this case, PAN .
• 17 Arteriosclerosis Hypertensive emergency Hepatitis B Hypertension Arteriolosclerosis Blood pressure Malignancy Etiology Cancer Pathology Medical history
Hepatitis
• 18
• 19
• 20 Bo tto m Line:

• 21 PAN is a systemic necrotizing va sculitis of small- or medium-sized arteries characterized histologically by fibrinoid necrosis. PAN , which is generally
• ;::~ ntin P IItr·nnhil rvtnnl ;::~ c;:mi r ;::~ ntihnrlv n P n ;::~ tivP h ;::~ c;: ;::~ c;t.,-nnn ;::~ c;:c;:n ri ;::~ tinn with h P n ;::~ titic;: R infP rtinn

6
lock
s
Suspend
0
End Block
Item: 2 of 48 ~- I • M k <:] t> al ~· ~
QIO: 1429 .l. ar Previous Next lab 'Vfl1ues Notes Calculator

1
F is not correct. 8% chose this.
2 Onion skinning, seen in arteriolosclerosis, is the type of arteriosclerosis associated with m alignant hypertension. Even though this patient has
hypertension, he is young and unlikely to have had uncontrolled blood pressure long enough to produce this pathology. Also, his skin findings and m edical
•3
history of hepatitis B point to a different etiology, in this case, PAN .
.4 Arteriosclerosis Hypertensive emergency Hepatitis B Hypertension Arteriolosclerosis Blood pressure Malignancy Etiology Cancer Pathology Medical history
Hepatitis
•5
•6
.7 Bottom Line:

•8 PAN is a systemic necrotizing vasculitis of small- or m edium-sized arteries characterized histologically by fibrinoid necrosis. PAN, which is generally
antineutrophil cytoplasmic antibody negative, has a strong association with hepatitis B infection .
•9 Fibrinoid necrosis Hepatitis B Anti -neutrophil cytoplasmic antibody Vasculitis Necrosis Antibody Hepatitis Histology Artery Systemic vasculitis Infection Cytoplasm

• 10

· 11
• 12 I ill ;fi 1!1 I•J for year:[ 2017 ..
FI RST AI D FA CTS
• 13
• 14 FA17 p 302.1
• 15 Vasculitides
• 16 EPIDEMIOLOGY/PRESENTATION PATHOLOGY/lABS

• 17 Large-vessel vasculitis
• 18 Giant cell (temporal) Usually elderly females. Most commonly affects branches of carotid
• 19 arteritis Unilateral headache (temporal artery}, jaw artery.
• 20
claudication. Focal granulomatous inAammation rJ.
May lead to irreversible blindness due to t ESR.
• 21
oohthalmic arterv occlusion. Treat with hi2h-dose corticosteroids orior to
6
lock
s
Suspend
0
End Block
Item: 3 of 48 ~ 1 • M k -<:J 1>- Jil ~· !:';-~
QIO: 1414 ..L ar Pre v ious Next Lab~lues Notes Calcula t o r
& &
1
A 48-year-old man presents to the emergency department 1.5 hours after onset of severe substernal chest pain radiating to his left arm. The pain Is
2 accompanied by diaphoresis and shortness of brea th . His blood pressure Is 165/94 mm Hg, pulse is 82/min, and respiratory rate is 18/min.

•3
Which of the following tests is the most important in his evaluation at this time?
·4
•5
A. Aspartate aminotransferase
•6
•7 B. Creat1ne kinase-myocard ial bound

·8 C. ECG
.9
D. Lactate dehydrogenase
• 10
E. Tropon in I
• 11
• 12
• 13
• 14
• 15
• 16
• 17
• 18
• 19
• 20
• 21

a
Lock
s
Suspend
8
End Bl ock
Item: 3 of 48 ~ 1 • M k -<:J 1>- Jil ~· !:';-~
QIO: 1414 ..L ar Previous Next Lab~lues Notes Calculat or

1
&
Th e co rrect a nswer i s c. 58% chose this. &

2 ECG Is the gold standard for diagnosing myocardial infarction within the first 6 hours of
onset of symptoms. An ECG should be performed within 10 minutes of medical contact in
3 any patient with suspected acute coronary syndrome (ACS), whether In the hospital or in a
pre-hospita l setting. ECG changes associated wit h ACS include ST-segment elevation, like
·4 that shown In the image (signifying transmural infarct), ST-segment depression (signifying
•5 subendocardia l infarct), and/ or Q waves (signifying tra nsmural infarct).
Acute cor nary syndrome Myocardial infarction Gold standard (test; Electrocardiograo-.ry
•6 Coronary circulation Infarction QRS complex Major depressive disorder
•7

·8
.9
Image copyright © 2011 Luther eta/; licensee BioMed
• 10 Central Ltd

• 11
A is not co r rect. 1 Ofo ch ose this •
• 12 Aspartate aminotransferase is a nonspecific protein that is found in cardiac, liver, and skeletal muscle cells. Levels peak around 2 days after a myocardial
Infarction ( MI) and are negative at 3 days post-MI.
• 13
Aspartate transaminase Myocardial infarction Skeletal muscle Uver Protein Aspartic acid Transaminase
• 14
8 is not co rrect . 90/o chose this •
• 15 Serial measurements of the CK-MB fraction and troponin I are made In every patient with suspected myocardial infarction (MI). However, because these
enzymes take 4-6 hours to accumulate in the blood, and ECG results can be obtained faster, ECG is the test of choice when triaging a patient with
• 16 suspected MI. CK-MB levels peak around 24 hours and are negative at 3 days post-MI, making it an important m arker of recurrent MI.
• 17 Myocardial infarction Troponin I CPK-MB test TNNI3 Creatine kinase Troponin Electrocardiography

• 18 D i s n ot correct . JO/o ch ose this .


Lactate dehydrogenase (LDH) was once the test of choice for diagnosing myocardial infarction (MI), but has lost favor in light of the use of CK-MB and
• 19
troponln I. An increase in the blood concentration of LDH-1 (found in heart t issue) to a level greater than that of LDH -2 (found in the reticuloendothelial
• 20 system) Is highly suggestive of MI. The LDH elevation generally appears within 12-24 hours. LDH can remain elevated for 2-7 days post-MI, and generally
peaks around 3 days post-MI.
• 21 Myocar diar infarction Lactate dehydrogenase Troponin Creatine kinase Mononucleat phagocyte system Lactic acid CPK-MB test Infarction

a
Lock
s
Suspend
8
End Bl ock
Item: 3 of 48 ~. I • M k <:] t> al ~· ~
QIO: 1414 .l. ar Previous Next lab 'Vfl1ues Notes Calculator

1
D is no t co rrect. 3% cho se this.
2
Lactat e dehydrogenase (LDH) was once the t est of choice for diagnosing myocardial infarction (MI ), but has lost favor in light of the use of CK-MB and
3 troponin I. An increa se in the blood concentration of LDH-1 (found in heart tissue) to a level grea t er than that of LDH-2 (found in the reticuloendothelial
system ) is highly suggestive of MI. The LDH elevation generally appears within 12 - 24 hours. LDH can remain elevat ed for 2 - 7 days post -MI, and generally
•4 peaks around 3 days post -MI.
•5 Myocardial infarction lactate dehydrogenase Troponin Creatine kinase Mononuclear phagocyte system lactic acid CPK-MB test Infarction

•6 E is no t co rrect. 29 % cho se this •


Serial mea surements of the CK-MB fraction and troponin I are made in every patient with suspected myocardial infarction (MI ). However; because these
•7 enzymes t ake 4- 6 hours to accumulat e in the blood, and ECG results can be obtained faster; ECG is the t est of choice when triaging a patient with
suspected MI. Troponin I is the most specific biomarker for MI ; it remains elevat ed for 7- 10 days after the event .
•8
Myocardial infarction Troponin I Biomarker Creatine kinase CPK-MB test Troponin Electrocardiography TNNI3
•9
• 10
Bo tto m Line:
· 11
ECG is the gold standard for diagnosis of MI within the first several hours following the onset of symptoms (eg, chest pain, diaphoresis) and can help to
• 12 det ermine the extent of infarction .
Perspiration Electrocardiography Chest pain Infarction
• 13
• 14
• 15
I iii I;fi 1!1 I•J f o r year:l 20 1 7 ..
• 16 FI RST AI D FA CTS

• 17
FA17 p295.1
• 18
• 19 Diagnosis of In the fi rst 6 hours, ECC is the gold standard.
• 20
myocardial infarction Cardiac troponin I rises after 4 hours (peaks
at 24 hr) and is t for 7-10 days; more specific
• 21
• than other orotein ma rke rs.

6
lock
s
Suspend
0
End Block
Item: 4 of 48 ~ 1 • M k -<:J 1>- Jil ~· !:';-~
QIO: 2147 ..L ar Pre v ious Next Lab~lues Not es Calcula t o r
& &
1
A 25-year-old man who is a heavy smoker develops acute onset of fever, malaise, muscle pain, hypertension, abdominal pain, bloody stool, and
2 prerenal failure 6 months after recovering from an acute hepatitis B infection.

3
Which of the following treatment regimens is most appropriate for this patient?
•4
•5
A. ~-Blocker
•6
•7 B. Coronary bypass

·8 c. Cyclophosphamide
.9
o. IV Immunoglobulin
• 10
E. Smoking cessation
• 11
• 12
• 13
• 14
• 15
• 16
• 17
• 18
• 19
• 20
• 21

a
Lock
s
Suspend
8
End Bl ock
Item: 4 of 48 ~ 1 • M k -<:J 1>- Jil ~· !:';-~
QIO: 2147 ..L ar Prev ious Next Lab~lues Not es Calculat o r

& &
1
2 Th e correct an sw er i s c . 440/o chose this.
This patient is most likely suffering from polyarteritis nodosa (PAN), a vasculitis (ie, inflammation of a blood
3 vessel) characterized by inflammation affecting small to medium-sized arteries, particularly the renal,
4 cardiac, and gastrointestinal tract vessels (usually not the pulmonary vasculature). As many as 30% of
patients have had prior hepatitis B infections. The best course of treatment Includes corticosteroids and/ or
•5 cyclophosphamide, which helps to decrease the immune system's response and decrease the amount of
Immune complex-mediated vasculitis events.
•6
Despite Its potential adverse genitourinary effects, cyclophospha mide is frequently used to control acute
•7 flares of PAN as its use leads to rapid resolut ion of symptoms. Even in patients with low glomerular
filtration rate secondary to renal involvement of PAN, cyclophosphamide can be renally dosed and remains
·8 a widely used drug to control acute PAN flares.
.9 The X-ray angiogram shown here points out the segmenta l aneurysms (arrows) seen in PAN .
• 10 Polyarteritis nodosa Cyclophosphamide Blood vessel Renal function Hepatitis B Human gastrointestinal tract
• 11 Angiography Corticosteroid Gastrointestinal tract Inflammation vasculitis Immune system Circulatory system

• 12 Hepatitis Artery Genitourinary system Aneurysm Glomerulus X-ray Kidney

• 13
Copyright ©2012 Raman et at.; licensee
• 14 BioMed Central Ltd. I mage courtesy Or.
• 15 Leslie Cooper

• 16 A is not co rrect . aoto ch ose this.


• 17 13-Adrenerglc blocking agents are a class of drugs most commonly used to manage hypertension and cardiac arrhythmias. Hypertension is defined as a
chronic heart condition in which the systemic arterial blood pressure Is Increased or the blood pressure is> 140/90 mm Hg. Additionally, 13-blockers are
• 18 shown to decrease mortality rates in congestive heart failure through mechanisms Including neurohormonal modification, improvement in symptoms of
• 19 left ventricular ejection fraction, survival benefit, arrhythmia prevention, and control of ventricular rate .
Hypertension Ejection fraction Heart failure Congestive heart failure Cardiac arrhythmia Blood pressure Heart rate Neurohormone Heart
• 20
B i s n ot correct. 4 0/o chose this•
• 21 Takayasu arteritis is a vasculitis characterized by fibrotic thickening of the aortic arch (it also affects the pulmonary arteries, the branches of the aortic

a
Lock
s
Suspend
8
End Bl ock
Item: 4 of 48 ~. I • M k <:] t> al ~· ~
.l.
QIO: 2147

1
' ~ . ar Previous Next
. lab 'Vfl1ues
.
Notes Calculator
. . . . .
chronic heart condition in which the systemic arterial blood pressure is increa sed or the blood pressure is> 140/ 90 mm Hg. Additionally, j3 -blockers are
2 shown to decrea se mortality rat es in congestive heart failure through mechanisms including neurohormonal modification, improvement in symptoms of
left ventricular ej ection fraction, surviva l benefit, arrhythmia prevention, and control of ventricular rat e.
3 Hypertension Ejection fraction Heart failure Congestive heart failure Cardiac arrhythmia Blood pressure Heart rate Neurohormone Heart

4 B is no t co rrect. 4 % cho se this.


•5 Takayasu arteritis is a vasculitis chara cterized by fibrotic thickening of the aortic arch (it also affects the pulmonary arteries, the branches of the aortic
arch, and the rest of the aorta in up to one -third of patients). Clinically, patients often have lower blood pressure and weaker pulses in the upper
•6 extremities than in the lower extremities; some patients have ocular disturbances as well. Most patients respond well to steroids, but surgical options
such as coronary bypass grafting are considered for patients who do not respond to steroids.
•7
Takayasu' s arteritis Aortic arch Aorta Vasculitis Pulmonary artery Fibrosis Blood pressure Artery Coronary artery bypass surgery Arteritis Steroid
•8
D is no t co rrect. 1 8 % cho se this •
•9 Kawasaki disea se is a self-limited vasculitis that normally occurs in infants and children and is characterized by conjunctival and oral erythema, fever;
erythema and edema of the palms and soles, generalized rash, and cervical lymph node swelling. About 20% of patients may go on to develop coronary
• 10
artery inflammation and/or aneurysm. Trea tment includes the administration of I V immunoglobulin or aspirin.
· 11 lymph node Kawasaki disease Erythema Cervical lymph nodes Vasculitis Aspirin Antibody Immunoglobulin therapy Edema Conjunctiva Aneurysm Inflammation
Rash Sole (foot) Fever Coronary circulation lymph
• 12
E is no t co rrect. 26 % cho se this •
• 13
Buerger disea se, also known as thromboangiitis obliterans, is a va sculitis that mostly affects arteries and veins of the extremities. As such, patients often
• 14 have intermittent claudication and Ra ynaud phenomenon. The majority of patients are males between the ages of 40 and 45 years, but the disea se can
occur at younger ages as well. Pa tients are typically hea vy smokers and show hypersensitivity to tobacco injected into the skin. Trea tment involves
• 15 smoking cessation .
• 16 Raynaud' s phenomenon Thromboangiitis obliterans Intermittent claudication Smoking cessation Claudication Vasculitis Artery Tobacco

• 17

• 18 Bo tto m Line:
• 19 Polya rteritis nodosa is a systemic necrotizing vasculitis of small to medium-sized arteries. It has a significant association with hepatitis B infection . It is
trea t ed with cyclophosphamide and corticosteroids .
• 20 Polyarteritis nodosa Cyclophosphamide Hepatitis B Vasculitis Corticosteroid Necrosis Systemic vasculitis Hepatitis Artery

• 21

6
lock
s
Suspend
0
End Block
Item: 4 of 48 ~- I • M k <:] t> al ~· ~
QIO: 2147 .l. ar Previous Next lab 'Vfl1ues Notes Calculator

2
3
lijj ;fi IJ l•l for year:l 2017 ..
4 FIRST AID FACTS

•5
•6 FA17 p 302.1
Vasculitides
•7
EPIDEMIOLOGY/PRESENTATION PATHOLOGY/LABS
•8
Large-vessel vasculitis
•9
Giant cell (t empo ral) Usually elderly females. Most commonly affects branches of carotid
• 10
arteritis Unilateral headache (temporal artery}, jaw artery.
· 11 claudication. Focal granulomatous inAammation rJ.
• 12 lay lead to irreversible blindness due to t ESR.
• 13 ophthalmic artery occlusion. Treat with high-dose corticosteroids prior to
• 14 Associated with polymyalgia rheumatica. temporal artery biopsy to prevent blindness.
• 15 Takayasu arteritis Usually Asian fema les< 40 years old. Granulomatous thickening and narrowing of
• 16
"Pulseless disease" (weak upper extremity aortic arch and proximal great vessels rn.
pulses}, fever, night sweats, arthritis, myalgias, t ESR .
• 17
skin nodules, ocular disturbances. Treat with corticosteroids.
• 18
Medium-vessel vasculitis
• 19
Po lyarteritis no dosa Usually middle-aged men. Typically involves renal and visceral vessels, not
• 20
Hepatitis B seropositivity in 30% of patients. pulmonary arteries.
• 21 Fever, weight loss, malaise, headache. Transmural in Aammation of the arterial wall

6
lock
s
Suspend
0
End Block
Item: 5 of 48 ~ 1 • M k -<:J 1>- Jil ~· !:';-~
QIO: 3812 ..L ar Pre v ious Next Lab~lues Notes Calcula t o r
& &
1
A 67-year-old obese man presents with severe substernal chest pain. He also experiences nausea, vomiting, and profuse diaphoresis. He has had
2 similar episodes in the past when climbing stairs, but they have always resolved with rest . Laboratory studies revea l elevated troponin levels. The
patient's ECG is shown in the image. Heart catheterization fails to demonstrate blood flow through the posterior interventricular artery.
3
4
•5
•6
.7
·8
.9
• 10
• 11

• 12
• 13
• 14
Image courtesy of James Heilman, MD
• 15
• 16
Which of the following is the most likely incit ing event in the pathogenesis of acute myocardial infarction in this man?
• 17
. 18 A . Coronary artery dissection
• 19
B. Coronary artery vasospasm
• 20
• 21
c. Plaque rupture

a
Lock
s
Suspend
8
End Bl ock
2
3
4
•5
•6
•7

·8
.9
• 10
Image courtesy
• 11
• 12
Which of the following is the most likely inciting event in the pathogenesis of acute myocardial infarction in this man ?
• 13
• 14 A . Coronary artery dissection
• 15
B. Coronary artery va sospasm
• 16
• 17 c. Plaque rupture

• 18 D. Thromboembolic disease
• 19
E. VIral infection
• 20
• 21

a
Lock
s
Suspend
8
End Block
Item: 5 of 48 ~ 1 • M k -<:J 1>- Jil ~· !:';-~
QIO: 3812 ..L ar Prev ious Next Lab~lues Notes Calculat o r

& &
1 Th e co rrect an sw er i s c . 680/o chose this.
2 This patient is experiencing an acute myocardial infarction (MI). The primary mechanism of acute MI involves acute plaque rupture with a background of
atherosclerosis (illustrated in these images). This promotes thrombogenesls and leads to coronary arterial thrombotic occlusion and absence of blood flow
3 to the area of myocardium supplied by the artery. The infarcted cardlomyocytes spill troponin into the bloodstream. Note that this patient has a posterior
4 MI with ST elevations, despite the fact that it initially appears as an ST depression. Keep in mind that the v 1 lead sits over the anterior wall of the heart

I
and therefore "sees· the posterior wall from the endocardial side. Therefore, disease processes in the posterior wall will be "flipped" in the ECG.
5 Athe oscle os•s Myocardial infarction cardiac muscle Troponin cardiac muscle ce I •farction Endocardium Thrombosis ST elevation Vascular occlusion Blood flow

•6
•7

·8
.9
• 10
• 11
• 12
• 13
• 14
• 15 Narrowed
• 16
• 17 (a) (b)
Image courtesy of OpenStax College
• 18
• 19 A i s n ot correct. 5% ch ose this .
Spontaneous dissection of native coronary arteries is rare, but occurs most often in the setting of coronary injury by catheter tips during invasive
• 20
angiography and also may affect women in the peripartum period. When It occurs, coronary dissection often leads to sudden death. In this case the onset
• 21 of the MI was prior to the angiographic procedure and not a complication caused by the catheter tip .

a
Lock
s
Suspend
8
End Bl ock
Item: 5 of 48 ~ 1 • M k -<:J 1>- Jil ~· !:';-~
QIO: 3812 ..L ar Prev ious Next Lab~lues Notes Calculat o r

1
Image courtesy of OpenS tax College
2
A i s n ot correct. 5 0/o chose this.
3
Spontaneous dissection of native coronary arteries is rare, but occurs most often in the setting of coronary injury by catheter tips during invasive
4 angiography and also may affect women in the peripartum period. When It occurs, coronary dissection often leads to sudden death. In this case the onset
of the MI was prior to the angiographic procedure and not a complication caused by the catheter tip.
5 Angiograpl y Coronary circulation catheter Dissection Artery Chi dbirth
•6

I
8 i s not correct. 100/o chose t his.
.7 Although coronary artery vasospasm (Prinzmetal angina) is an acute event and can be associated with atherosclerosis, it is episodic in nature and
Infrequently causes thrombotic occlusion. It generally mimics stable angina, but it occurs at rest. Vasospasm can also occur secondary to cocaine
·8 Ingestion, exercise, hyperventilation, or exposure to cold.
.9 Prinzmetao s angina Atherosclerosis Hyperventilation Vasospasm Angina pectoris Coo..dine Coronary vasospasm Stable angina Thrombosis Coronary circulation
Vascular occlusion
• 10
0 i s n ot co rrect. 1 50/o chose this •
• 11
Thromboembolic disease is a process in which a thrombus becomes dislodged and travels through the bloodstream to cause occlusion of a vessel.
• 12 Although this can indeed be the initiating factor in acute MI, it is much more likely that plaque rupture caused the thrombosis in this patient, given the
patient's weight and likely atherosclerotic plaques already in place. Emboli that travel through the coronary arteries can arise from two places: First, deep
• 13 vein thromboses that embolize end up occluding pulmonary blood flow (causing pulmonary embolus) but may cross to the left side of the heart If the
• 14 patient has a septal defect (eg, a patent foramen ovale). Second, atrial fibrillation may lead to a development of a left atrial thrombus that could shower
emboli Into the coronary circulation . H owever~ this patient's ECG falls to demonstrate atrial fibrillation .
• 15 Patent foramen ovale Pulmonary embolism Thrombus Deep vein thrombosis Coronary circulation Embolism Atrial septal defect Atrial fibrillation Embolization

• 16 Embolus Atherosclerosis Thrombosis Foramen ovale (heart) Electrocardiography Artery vascular occlusion Deep vein Circulatory system Blood flow Fibrillation

• 17 E is not co rrect. 20/o ch ose this .


VIral Infections such as coxsackie A infection can cause pericarditis and Inflammation of the pericardium. This causes changes that can be seen on ECG;
. 18 however, there usually are diffuse, mild ST elevations in all leads, not In a specific anatomic distribution. In addition, symptoms of pericarditis are not as
• 19 severe or acute as in this patient .
Pericardium Coxsackie A virus Pericarditis Coxsackievirus Electrocardiography Inflammation Infection Virus
• 20
• 21

a
Lock
s
Suspend
8
End Bl ock
Item: 5 of 48 ~. I • M k <:] t> al ~· ~
QIO: 3812 .l. ar Previous Next lab 'Vfl1ues Notes Calculator

2 Bo tto m Line:
3 Acute myocardial infarction (MI) can be diagnosed with ST-segment elevations in an anatomic distribution on ECG. The pathophysiologic event that
most often triggers MI is rupture of an atherosclerotic plaque, which exposes thrombogenic material to the bloodstream, resulting in thrombosis and
4 coronary artery occlusion.
Myocardial infarction Atheroma Atherosclerosis Thrombosis Thrombogenicity Pathophysiology Vascular occlusion Infarction Coronary circulation
5
•6
.7
•8
liU ;fi 1!1J t) f o r yea r :[ 20 17
FIRST AID FAC T S
.. ]

•9
• 10

· 11 Evolution of Commonly occluded coronary arteries: LAD > RCA> circumflex.


FA17p294.1 I
• 12 myocardial infarction Symptoms: diaphoresis, nausea, vomiting, severe retrosternal pain, pain in left arm and/or jaw,
shortness of breath, fat igue.
• 13
• 14 TIME GROSS LIGHT MICROSCOPE COMPliCATIONS

• 15 0-24 hr one Early coagulative necrosis, Ventricular arrhythmia, HF,


release of necrotic cell cardiogenic shock.
• 16
contents into blood; edema,
• 17 hemorrhage, wavy fibe rs.
Occluded Neutrophils appear.
• 18
Reperfusion injury,
• 19
associated with generation
• 20 of free radicals, leads to
• 21 hypercontraction of myofibril s

6
lock
s
Suspend
0
End Block
2

3
4
5 FA17 p 2 93.2
•6 Ischemic heart disease manifestations
.7 Angina Chest pain due to ischemic myocardium 2° to coronary arterr narrowing or spasm; no myocyte
·8 necrosts.
.9
Stable-usually 2° to atherosclerosis; exertional chest pain in classic distribution (usually with
ST depression on ECG), resolving'' ith rest or nitroglycerin .
• 10
• Variant (Prinzmetal)-occurs at rest 2° to coronary artery spasm; transient ST elevation on
• 11 ECG. Smoking is a risk factor, but hypertension and hypercholesterolemia are not. Triggers
• 12 may include cocaine, alcohol, and triptans. Treat '' ith Ca 2+ channel blockers, nitrates, and
• 13 smoking cessation (if applicable).
• Unstable- thrombosis with incomplete coronary artery occlusion; +1-ST depression and/or

I
• 14
T-wave inversion on ECC but no ca rdiac biomarker elevation (unlike NSTEMI); f in Frequency
• 15
or intensity of chest pain or any chest pain at rest.
• 16
Coronary steal Distal to coronary stenosis, vessels arc maximal ly dilated at baseline. Administration of vasodilators
• 17 syndrome (eg, dipyridamole, regadenoson) dilates norma l vessels and shunts blood toward well-perfused areas,
. 18 thereby diverting Aow away from vessels that are stenosed and leading to ischemia in myocardium
• 19 perfused by these vessels. Principle behind pharmacologic stress tests with coronary vasodilators.
• 20 Sudden cardiac death Death from cardiac causes within I hour of onset of srmptoms, most commonly due to a lethal
• 21 arrhythmia (cg, VF). Associated with CAD (up to 70% of cases), cardiomyopathy (hypertrophic,
• •·• • ' '" 1 t 1 •• 1 t • t • 1 "'""" t n t t '\ '"

a
Lock
s
Suspend
8
End Block
Item: 5 of 48 ~ 1 • M k -<:J 1>- Jil ~· !:';-~
QIO: 3812 ..L ar Pre v ious Next Lab~lues Notes Calcula t o r

3 FA17 p296.1

4
Myocardial infarction complications
5 Cardiac arrhythmia Occurs \\ithin the first fC\\ days after \ II. Important cause of death before reaching the hospital
•6
and with in the first 2-+ hours post- 'vii.
.7 Postinfarction Occurs l-3 days after ~ I I. Friction rub.
fibrinous pericarditis
·8
Papillary muscle Occurs 2-7 days after .\II. Posteromedial papillar} muscle rupture rJ t risk due to single blood
.9
rupture supply from posterior descending arterr. Can result in severe mitral regurgitation.
• 10
Interventricular septal Occurs 3-5 days after ~ I I. ~ l acrophagc-media ted degradation - VSD.
• 11
rupture
• 12
Ventricular Occurs 3-14 days after MI. Contained free wall rupture [l.J; l CO, risk of arrhythmia, embolus
• 13 pseudoaneurysm from mural thrombus.
• 14 formation
• 15 Ventricular free wall Occurs 5-14 days after MI. Free wal l rupture - cardiac tamponade. LV hypertrophy and
• 16 rupture previous Ml protect against free wall rupture.
• 17 True ventricular Occurs 2 weeks to several months after 11. Outward bulge with contraction ("dyskinesia"},

I
. 18 aneurysm associated with fibrosis.
• 19 Dressler syndrome Occurs several weeks after II. Autoimmune phenomenon resulting in fibrinous pericarditis.
• 20 LV failure and Can occur 2° to LV infarction, SD, free wall rupture, papillarr muscle rupture with mitral
• 21 pulmonary edema regurgitation .

a
Lock
s
Suspend
8
End Bl ock
Item: 6 of 48 ~ 1 • M k -<:J 1>- Jil ~· !:';-~
QIO: 1416 ..L ar Previous Next Lab~lues Notes Calculator

1 &

A 17-year-old boy dies suddenly while playing basketball for his high school team. He had no medical history, and his parents report that he has been
Involved In various sports for 10 years with no difficulties. His heart is examined at autopsy (see image ).
IA•A] &

3
4
5
•6
.7
·8
.9
• 10
• 11

• 12
• 13
• 14
• 15
• 16
• 17
. 18 Which of the following is the most likely cause of this person's death?

• 19
A. Enterovirus infection
• 20
• 21 8 . Frameshift mutation in the dystrophin gene

a
Lock
s
Suspend
8
End Block
2
3
4
5
•6
•7

·8
.9
• 10
• 11
• 12
Which of the following is the most likely cause of this person's death?
• 13
• 14 A . Enterovirus infection
• 15
8. Frameshift mutation in the dystrophin gene
• 16
• 17 C. Mutation in a sarcomere gene such as 13-myosin heavy chain

• 18 D. Protein deposits such as transthyretin


• 19
E. Trypanosome transmitted by insect bite
• 20
• 21

a
Lock
s
Suspend
8
End Block
Item: 6 of 48 ~ 1 • M k -<:J 1>- Jil ~· !:';-~
QIO: 1416 ..L ar Previous Next Lab~lues Notes Calculator
& &
1 Th e co rrect an sw er i s c . 850/o chose this.
2 The clinical vignette and image are consistent wit h hypertrophic obstructive cardiomyopathy
(HOCM. ) The key pathologic finding is hypertrophy wit h asymmetric septal enlargement, but
3 without free ventricular wall enlargement (shown in t hese cardiovasucalr magnetic
4 resonance images). The disease is ca used by mutations in sa rcomere genes, such as the 13-
myosln heavy chain . HOCM is inherited in an aut osomal dominant fashion. The anatomic

I
5 distortion can lead to a dynamic ventricular outfl ow obst ruction during systolic ejection,
which leads to a systolic murmur, dyspnea, light hea dedness, and syncope. HOCM Is also
6 assodated With sudden cardiac d eath, as seen in t his patient , most often as a result of
ventricular tachycardia.
0 7
Sarcomere Dyspnea Sudden cardiac death Hypertrophic cardiomyopathy Ventricurar tacnycardia
o8 Dominance (genetics) Syncope (medicine) Systole Hypertrophy Tachycardia Ughtheadedness
.9 Cardiomyopathy Ventricular hypertrophy Heart murmur

• 10
• 11
• 12
• 13
0 14
• 15
Images copyright © 2009 Papavassiliu et at; licensee
0
16 BioMed Central Ltd.
0
17
A is not co rrect . 20/o ch ose this .
• 18 I nfection with an Enterovirus, namely coxsackie B virus, init ially causes a subacute m yocarditis but can event ually lea d to dilated cardiomyopathy (DCM).
• 19 Pathologically, DCM produces a grossly enlarged heart wit h dilation of all four chambers and normal vent ricular wall t hickness. Other causes of DCM
Include alcohol abuse, wet beriberi, coca ine use, Chagas disea se, doxorublcln toxicity, hemochromatosis, and peripartum ca rdiom yopathy.
• 20 Chagas disease Beriberi Doxorubicin Myocarditis I ron overload Dilated cardiomyopathy Peripartum cardiomyopathy Cocaine cardiomegaly Coxsackie B virus

• 21 Cardiomyopathy Enterovirus Virus Alcohol Toxicity I nfection Coxsackievirus Alcohol abuse Acute (medicine)

a
Lock
s
Suspend
8
End Block
Item: 6 of 48 ~. I • M k <:] t> al ~· ~
QIO: 1416 .l. ar Previous Next lab 'Vfl1ues Notes Calculator

1 A is no t co rrect. 2 % cho se this.


2 Infection with an Enterovirus, namely coxsackie B virus, initially causes a subacute myocarditis but can eventually lea d to dilat ed cardiomyopathy (DCM).
Pathologically, DCM produces a grossly enlarged heart with dilation of all four chambers and normal ventricular wall thickness. Other causes of DCM
3 include alcohol abuse, wet beriberi, cocaine use, Chagas disea se, doxorubicin toxicity, hemochromatosis, and peripartum cardiomyopathy.
Chagas disease Beriberi Doxorubicin Myocarditis Iron overload Dilated cardiomyopathy Peripartum cardiomyopathy Cocaine Cardiomegaly Coxsackie B virus
4
Cardiomyopathy Enterovirus Virus Alcohol Toxicity Infection Coxsackievirus Alcohol abuse Acute (medicine)
5
B is no t co rrect. 6 % cho se this.
6
A frameshit or nonsense mutation in the dystrophin gene lea ds to the formation of a truncat ed dystrophin protein, resulting in inhibited muscle

I
.7 regeneration . This is the hallmark of Duchenne muscular dystrophy. This results in a dilat ed cardiomyopathy and would not cause the hypertrophy seen
the image .
•8 Dystrophin Duchenne muscular dystrophy Nonsense mutation Dilated cardiomyopathy Gene Cardiomyopathy Muscular dystrophy Protein Mutation Hypertrophy
•9 Muscle

• 10 D is no t co rrect. 4 % cho se this •


Deposits of protein such as light chains, hea vy chains, or transthyretin are associat ed with amyloidosis. Amyloidosis produces a restrictive cardiomyopathy
· 11
in which the ventricular wall and chamber size grossly appear normal, which is not consistent with the reduced chamber size seen in this specimen. In
• 12 addition, amyloidosis is generally a disea se of the elderly and is thus not as likely in this 17-year -old patient .
Transthyretin Restrictive cardiomyopathy Amyloidosis Protein Cardiomyopathy Immunoglobulin light chain
• 13
E is no t co rrect. 3 % cho se this •
• 14
Chagas disea se is caused by a trypanosome that is primarily endemic to South America. Eighty percent of those infected will develop myocarditis, and
• 15 10% will suffer acute cardiac dea th lat er. Grossly, the myocardium can appear normal or slightly dilat ed, with minute hemorrhagic lesions.
Chagas disease Myocarditis Trypanosomatida Trypanosoma Endemism Trypanosoma brucei Cardiac muscle South America
• 16
• 17
Bo tto m Line:
• 18
HOCM, when caused by a mutation of the cardiac j3 -myosin hea vy chain gene, is an autosomal dominant condition associat ed with asymmetric
• 19 ventricular wall enlargement and sudden dea th .
• 20 Gene Hypertrophic cardiomyopathy Dominance (genetics) Mutation Autosome

• 21

6
lock
s
Suspend
0
End Block
Item: 6 of 48 ~. I • M k <:] t> al ~· ~
QIO: 1416 .l. ar Previous Next lab 'Vfl1ues Notes Calculator

1
BottomLine:
2
HOCM, when caused by a mutation of the cardiac j3 -myosin heavy chain gene, is an autosomal dominant condition associat ed with asymmetric
3 ventricular wall enlargem ent and sudden dea th .
Gene Hypertrophic cardiomyopathy Dominance (genetics) Mutation Autosome
4
5
6
lijl;fiiJI•l toryea r:[2017 • ]
.7 FI RST AI D FAC TS

•8

I
FA17p297.1
•9
Cardiomyopathies
• 10
Dilated tvlost common cardiomyopathy (90% of cases). Systolic dysfunction ensues.
· 11
cardiomyopathy Often idiopathic or familial. Other etiologies Eccentric hypertrophy · (sarcomeres added in
• 12
include ch ronic Alcohol abuse, wet Beriberi, series).
• 13 Coxsackie B viral myocarditis, chronic ABCCCD.
• 14
• 15 (. I
·:":,
·~
Cocaine use, Chagas disease, Doxorubicin
toxicity, hemochromatosis, sarcoidosis,
Takotsubo cardiomyopathy: "broken heart
syndrome"- ventricular apical ballooning
• 16 '< ..-......
. -- peripartum cardiomyopathy.
Findings: H F', S3, systolic regurgitant murmur,
likely due to increased sympathetic stimulation
(stressfu l situations).
• 17
dilated heart on echocardiogram, balloon
• 18 appearance of heart on CXR .
• 19 Treatment: a+ restriction, ACE inhibitors,
• 20 P-blockers, diuretics, digoxin, lC D, heart
transplant.
• 21 , .... _,..._ ,. .. . ,.

6
lock
s
Suspend
0
End Block
Item: 6 of 48 ~ 1 • M k -<:J 1>- Jil ~· !:';-~
QIO: 1416 ..L ar Pre v ious Next Lab~lues Notes Calcula t o r
&
- . &
1
Amyloidosis, Sarcoidosis, I lcmochromatosis Loffier syndrome-endomyoca rdial fibrosis with
2 (although dilated cardiomyopathy is more a prominent eosinophilic infiltrate.
3 common) (Puppy LE.\ SII).
4
5 FA17 p 293.2

6 Ischemic heart disease manifestations


0 7 Angina zo
Chest pain due to ischemic myocardium to coronary arterr narrowing or spasm; no myocyte
o8 necrosts.
.9
zo
Stable- usually to atherosclerosis; exertional chest pain in classic distribution (usually with
ST depression on ECG), resolving '' ith rest or nitroglycerin .
• 10
Variant (Prinzmetal)-occurs at rest 2° to coronary artery spasm; transient ST elevation on
• 11 ECG. Smoking is a risk factor, but hypertension and hypercholesterolemia are not. Triggers
• 12 may include cocaine, alcohol, and triptans. Treat'' ith Ca 2+ channel blockers, nitrates, and
smoking cessation (if applicable).

I
• 13
0 14
• Unstable- thrombosis with incomplete coronary artery occlusion; +1- ST depression and/or
T-wave inversion on ECC but no ca rdiac biomarker elevation (unlike NSTEMI); f in rrequcncy
• 15
or intensity of chest pain or any chest pain at rest.
0
16
Coronary steal Distal to coronary stenosis, vessels arc maximally dilated at baseline. Administration of vasodilators
0
17 syndrome (eg, dipyridamole, regadenoson) dilates norma l vessels and shunts blood toward well-perfused areas,
. 18 thereby diverting Aow away from vessels that are stenosed and leading to ischemia in myocardium
• 19 perfused by these vessels. Principle behind pharmacologic stress tests with coronary vasodilators.
• 20 Sudden cardiac death Death from cardiac causes within I hour of onset of srmptoms, most commonly due to a lethal
• 21 arrhythmia (eg, VF). Associated with CAD (up to 70% of cases), cardiomyopathy (hypertrophic,
• ....J : I _ .__ ...J\ _ _ _.. ._ _ _ _ ..J : '- --·=-- _._ ___ t - · •t.. : .. '-- 1 - - - r'\,., ____ J _ ___ o _____ ..J ____ _.. _ ___, n ------ '-

a
Lock Suspend
s 8
End Bl ock
Item: 6 of 48 ~. I • M k <:] t> al ~· ~
QIO: 1416 .l. ar Previous Next lab 'Vfl1ues Notes Calculator

3 FA17 p213.1

4 Amyloidosis Abnormal aggregation of proteins (or their fragments) into ~ -plea ted linear sheets .... insoluble fibrils
-+ cellular damage and apoptosis. Amyloid deposits visualized by Congo red stain f.i.l, polarized
5
light (apple green birefringence) E), and H&E stain (~ shows deposits in glomerular mesangial
6
areas [white arrows], tubular basement membranes [black arrows]).
.7
COMMON TYPES DESCRIPTION
•8
AL (primary) Due to deposition of proteins from lg Light chai ns. Can occur as a plasma cell disorder or
•9 associated with multiple myeloma. Often affects multiple organ systems, including renal
• 10 (nephrotic syndrome), cardiac (restrictive cardiomyopathy, arrhythmia), hematologic (easy
· 11 bruisi ng, splenomegaly), Gl (hepatomegaly), and neurologic (neuropathy).
• 12 AA (secondary) Seen with chronic inflammatory conditions such as rheumatoid arthritis, IBD,
• 13
spondyloarthropathy, familial Mediterranean fever, protracted infection . Fibrils composed of
serum Amyloid A. Often multisystem like AL amyloidosis.
• 14
Dialysis-related Fibri ls composed of ~Tmicroglobulin in patients with ESRD and/or on long-term dialysis. May
• 15
present as carpal tunnel syndrome.
• 16
Heritable Heterogeneous group of disorders, including familial amyloid polyneuropathies due to transthyretin
• 17

I
gene mutation .
• 18
Age-related (senile) Due to deposition of normal (wild-type) transthyretin (TTR) predominantly in cardiac ventricles.
• 19 systemic SIO\\·er progression of cardiac dysfunction relative to AL amyloidosis.
• 20
Organ-specific Amyloid deposition localized to a single organ. Most important form is amyloidosis in Alzheimer
• 21 disease due to deposition of ~-amyloid protein cleaved from amyloid precursor protein (APP).

6
lock
s
Suspend
0
End Block
Item: 7 of 48 ~ 1 • M k -<:J 1>- Jil ~· !:';-~
QIO: 3814 ..L ar Pre v ious Next Lab~lues Notes Calcula t o r

1
&

A healthy 37-year-old man undergoes a physical exa mination as part of an employee health screening program . He is an active individual and reports
only minor joint pains, that he believes are secondary to cross-country running . Otherwise his review of systems is completely negative and he reports
IA•A] &

2
no past medical problems. On examination, t he physician notices an abnormality in cardiac auscultation. The 5 1 is a little louder than usual, and the
3 52 Is split into two sounds. The physician asks the patient to inhale and exhale, and notes no change in the volume or duration of the split 5 2.
4
5 Which of the following structural defects is m ost likely t he cause of this patient's findings on auscultation?

6
A. Defect 1n interatrial septum due to defident tissue
0 7
o8 B. Endocardial cushion defect

.9 c. Failed closure of the interventricular septum


• 10
D. Patent ductus arteriosus
• 11
E. Tricuspid valve atresia
• 12
• 13
0 14
• 15
0
16
0
17
• 18
• 19
• 20
• 21

a
Lock
s
Suspend
8
End Bl ock
Item: 7 of 48 ~. I • M k <:] t> al ~· ~
QIO: 3814 .l. ar Previous Next lab 'Vfl1ues Notes Calculator

2 The co rrect a nswer is A. 52 % cho se this.


3 The patient has an atrial septal defect (A5D ), the most common congenital cardiac anomaly that may first come to clinical attention in adults. (Ventricular
septal defect is the most common congenital cardiac defect overall. ) Given that pulmonary blood flow may be two to four times normal, a systolic flow
4 murmur is often heard . Moreover; the increa sed pulmonary blood flow delays closure of the pulmonic va lve during both inspiration and expiration ; thus a
fixed split 5 2 is often found . An A5D occurs due to a defect in interatrial septum. Most commonly, it affects the fossa ova lis region; however; it is distinct
5
from pat ent foramen ova le (PFO), which is due to failure of fusion of atrial septal tissue. PFO occurs more commonly than A5D, but most patients with
6 PFO are asymptomatic. Most PFOs det ected incidentally in asymptomatic patients do not require trea tment. In contrast, trea tment is indicat ed for A5D
when right ventricular volume enlargement develops.
7 Atrial septal defect Ventricular septal defect Interatrial septum Patent foramen ovale Pulmonary valve Foramen ovale (heart) Systole Fossa ovalis (heart)
•8 Congenital disorder Septum Ventricle (heart) Atrium (heart) Heart murmur Blood flow

•9 B is no t co rrect. 1 5 % cho se this •


Endocardial cushion defects are malformations of the atrioventricular canals and sometimes the atria that occur during embryonic heart development .
• 10
They are most commonly associat ed with Down syndrome (trisomy 2 1 ). Although these can include atrial septal defects (A5D ), they are generally serious
· 11 and require correction, usually in the form of surgery, in infancy. This patient is phenotypically normal and has been asymptomatic his whole life, making it
much more likely that he has a pure A5D .
• 12 Endocardial cushions Down syndrome Atrium (heart) Trisomy Endocardium Heart development

• 13 c is no t co rrect. 1 7 % cho se this.


• 14 Failed closure of the interventricular septum would result in ventricular septal defect (V5D ). These are more frequent than A5Ds, but are more often
symptomatic earlier in life. Initially V5Ds crea t e a left -to-right shunt that overloads the right ventricle and increa ses flow in the pulmonary vasculature .
• 15 Over time, pulmonary resistance rises secondary to hypertrophy and hyperplasia of the vessels, which lea ds to right ventricle hypertrophy to compensat e
for the increa sed resistance. In time, pulmonary resistance becomes grea t er than systemic resistance, and the shunt is reversed right -to-left . This
• 16
condition is referred to as Eisenmenger syndrome.
• 17 Ventricular septal defect Eisenmenger' s syndrome Interventricular septum Ventricle (heart) Hypertrophy Septum Circulatory system Ventricular system
Cardiac shunt Shunt (medical) Hyperplasia
• 18
• 19 D is no t co rrect. 8 % cho se this •
A pat ent ductus arteriosus ( PDA) is a failure of the ductus arteriosus to close after birth, resulting in a left -to-right shunt from the aorta to the pulmonary
• 20 arteries after birth. If present, a PDA results in a loud, continuous machine-like murmur rather than a fixed, split 5 2. PDAs are often trea t ed shortly after
birth with indomethacin, an antiprostaglandin that results in closure of the PDA .
• 21
Patent ductus arteriosus Indometacin Aorta Pulmonary artery Ductus arteriosus Cardiac shunt Artery Heart murmur

6
lock
s
Suspend
0
End Block
Item: 7 of 48 ~. I • M k <:] t> al ~· ~
QIO: 3814 .l. ar Previous Next lab 'Vfl1ues Notes Calculator

1
Cardiac shunt Shunt (medical) Hyperplasia
2
D is not correct. 8 % chose this.
3 A pat ent ductus arteriosus (PDA ) is a failure of the ductus arteriosus to close after birth, resulting in a left -to-right shunt from the aorta to the pulmonary
4 arteries after birth. If present, a PDA results in a loud, continuous machine-like murmur rather than a fixed, split 52. PDAs are often trea t ed shortly after
birth with indomethacin, an antiprostaglandin that results in closure of the PDA.
5 Patent ductus arteriosus Indometacin Aorta Pulmonary artery Ductus arteriosus Cardiac shunt Artery Heart murmur

6 E is not correct. 8% chose this.


7 Tricuspid valve atresia is the congenital absence of a tricuspid valve, which lea ds to the absence of the right ventricle. This defect causes early cyanosis in
neonat es. This would certainly not be asymptomatic until adulthood .
•8 Cyanosis Tricuspid valve Ventricle (heart) Atresia Congenital disorder Asymptomatic Infant Ventricular system Tricuspid atresia

•9
• 10 Bottom Line:
· 11 An A5D results in a loud 5 1 and wide, fixed split 5 2 on cardiac auscultation . It is often asymptomatic or unnoticed on examination until adulthood .
• 12
• 13
• 14 I ill ;fi 1!1 I•J for year:[ 2017 ..
FI RST AI D FA CTS
• 15
• 16
FA17 p276.1
• 17
Pressure-volume loops and cardiac cycle T he black loop rep resents normal card iac
• 18 physiology.
140
• 19 I Contractility I Afterload
• 20 t sv t (Aortic pressure)
120 tEF I SV Phases-left ventricle:
• 21 IESV I ESV 0 lsovolumetric contraction-period
l , I I 1 1 , •

6
lock
s
Suspend
0
End Block
Item: 7 of 48
QIO: 3814
~
..L
1 • Ma r k -<:J
Prev ious
I>
Next Lab
fJlues
£!1}>'

Notes
!!":-~
Cal culat o r

&
.. ~ . .. .
&
1 0 Ol 0.2 0.3 0.4 0.5 0.6 0.7 0.8 tamponade.
2 Time (se<)

3
4 FA17p277.1
5 Splitting
6 Normal splitting Inspiration - drop in intrathoracic pressure
7 - t venous return - t RV filling - t RV
E
I II
Sl A2 P2
stroke ,-olume - t RV ejection time
·8
- delayed closure of pulmonic \'alve. Lldelay
I Noonal
.9
l pulmonary impedance (t capacity of the
• 10 pulmonary circulation) also occurs during
• 11 inspiration, which contributes lo delayed
• 12 closure of pulmonic \'alve .
• 13 Wide splitting Seen in conditions that delay RV emptying (cg,
• 14 pulmonic stenosis, right bundle bwnch block).
E
I I I
Sl A2 P2
Causes delayed pulmonic sound (especially
• 15
on inspiration). An exaggeration of norma l I I
I Abnormal delav
• 16 spl itting.
• 17
Fixed splitting Heard in ASD. ASD - left-to-right shunt
. 18 - t RA and RV volumes - t Aow through
• 19 pulmonic valve such that, regardless of breath, E

• 20 pulmonic closure is greatlr delayed. I


Sl

• 21

I
a
Lock
s
Suspend
8
End Bl ock
Item: 8 of 48 ~ 1 • M k -<:J 1>- Jil ~· !:';-~
QIO: 3811 ..L ar Pre v ious Next Lab~lues Notes Calcula t o r
& &
1
A 33-year-old woman presents to her new primary care physician for a history and physical examination. She reports no current m edical problems
2 and takes no medications. Examination revea ls:

3 Temperature: 37.0°C (98.6°F)


Pulse: 90/min
4 Blood pressure: 108/60 mm Hg
5 Respiratory rate: 12/min
Oxygen saturation: 99% on room air
6
Physical examination is notable for a normal 5 1 and 5 2 with a II/VI late rumbling diastolic murmur at the apex. The murmur is enhanced by expiration.
7 Results of physical examination are oth erwise unremarkable. ECG results are normal. The patient revea ls she was hospitalized for a serious illness as a child.
·8
.9 Wh ich of the following cutaneous eruptions may have accompan ied the Illness that led to this physical finding?

• 10
• 11 A. Discoid rash

• 12 B. Erythema chronicum migrans

• 13
C. Erythema marginatum
• 14
D. Erythema multiforme
• 15
• 16 E. Janewa y lesions

• 17
• 18
• 19
• 20
• 21

a
Lock
s
Suspend
8
End Bl ock
Item: 8 of 48 ~ 1 • M k -<:J 1>- Jil ~· !:';-~
QIO: 3811 ..L ar Previous Next Lab~lues Notes Calculat or
& &
1
2
3 The correct a n sw er i s c. 5 5 01o chose this.
Erythema marginatum is a nonpruritic, circular, pink eruption on the trunk that Is present in the minority of
4 cases of acute rheumatic fever. Rheumatic fever is a consequence of pharyngeal Infection with group A
Streptococcus, usually in childhood. In some cases rheumatic fever can lead to valvular damage, often
5
mitral stenos1s. The characteristic murmur of mitral stenosis (a late, apical, rumbling diastolic murmur that
6 changes with respiration) is described in this Vignette. Jones criteria are used to diagnose acute rheumatic
fever. The major criteria includ e migratory polyarthrit is, carditis, subcutaneous nodules, Sydenham chorea,
7 and a characteristic rash called erythema marginatum (shown in this Image). Minor criteria consist of fever,
arthralgia, increased erythrocyte sed im entation rate, leukocytosis, evidence of heart block, and evidence of
8 streptococcal Infection such as e levated antistreptolysin o.
.9 Itch E ythema marginatum Arthralgia Sydenham's chorea Erythrocyte sedimentation rate Leukocytosis
Red blood cell Rheumatic fever Heart block Chorea Polyarthritis Erythema Anti-streptolysin 0
• 10
Group A streptococcal infection Streptococcus Stenosis Carditis Rash Fever Pharynx Rheumatism Heart murmur
• 11
Mitral valve stenosis Subcutaneous tissue Infection
Image courtesy of RG Gower eta/. Copyright
• 12 ©2011 world Allergy Organization; licensee
BioMed Central Ltd.
• 13
• 14 A is not co rrect . 4 0/o chose t his.
• 15 Discoid lesions are small, flaky, red rashes that are nam ed for their coin shape (shown in this image). They
appear on the face, trunk, and extremities of patients with systemic lupus erythematosus (SLE). The m alar
• 16 rash, also called "butterfly rash," across the cheeks and nasal bridge Is also characteristic of lupus. SLE
usually affects women between the ages of 15 and 45 years. Symptoms Include fever, fatigue, weight loss,
• 17 hllar adenopathy, and arthritis. Multiple organ systems can be affected by SLE. Immune complex
• 18 deposition in the kidneys can lead to renal failure. SLE is also associated with pleuritis, pericarditis, and
Llbman-Sacks endocarditis.
• 19 Ubman-Sacks endocarditis Systemic lupus erythematosus Immune complex Pleurisy Endocarditis Pericarditis
• 20 Malar rash Lymphadenopathy Fatigue (medical) Arthritis Lupus erythematosus Kidney Weight loss Rash Fever

• 21

a
Lock
s
Suspend
8
End Block
Item: 8 of 48 ~ 1 • M k -<:J 1>- Jil ~· !:';-~
QIO: 3811 ..L ar Prev ious Next Lab~lues Notes Calculat o r

& &
1 A is no t co rrect. 4 0/o chose this.
2 Discoid lesions are small, flaky, red rashes that are named for their coin shape (shown in this image). They
appear on the face, trunk, and extremities of patients with systemic lupus erythematosus (SLE). The malar
3 rash, also called "butterfly rash," across the cheeks and nasal bridge Is also characteristic of lupus. SLE
usually affects women between the ages of 15 and 45 years. Symptoms Include fever, fatigue, weight loss,
4 hllar adenopathy, and arthritis. Multiple org an systems can be affected by SLE. Immune complex
5 deposition In the kidneys can lead to renal fa ilure. SLE is also associated with pleuritis, pericarditis, and
Ubman-Sacks endocarditis.
6 Ubman-S.oc s e odocarditis Systemic lupus erythematosus Immune complex Pleu .sy E .docarditis Pencarditis

7 Malar rash Lymphadenopathy Fatigue (medical) Arthritis Lupus erythematosus Kidney Weight loss Rash Fever

8
.9
• 10
• 11 B is not co rrect. 70/o ch ose this •
Erythema chronicum migrans is the expanding "bull's eye" red rash with central clearing that can be
• 12 present In the early stages of Lyme disease (as shown in this image). Lyme disease Is caused by Borrelia
burgdorferi, which is transmitted by the Ixodes tick. Lyme disease is known to cause heart block in later
• 13
stages. However, it is not associated with mitral stenosis.
• 14 Lyme disease Borrelia burgdorferi Heart block Erythema chronicum migrans Borrelia Ixodes Tick Rash
Erythema
• 15
• 16
• 17
. 18
• 19
0 i s n o t correct. 130/o chose this .
• 20
Erythema multiforme is a severe skin erupt ion (shown in this image) that presents with macules, papules,
• 21 bulls, and target lesions. Stevens-Johnson syndrome is the major form of erythema multiforme and is
• ~~<"",..l ~l"of"t u d l"h ovr'u"\~l l r""O t-n e-• • lf ~ ~ nrl ~nt- i,.,._nu• • lc ::3o nt- r(,-, , ,.-.c~· Tt- ic- ,..h~,-~,.....OI"'I .,o,. hH h l ,.,h fouo,.. -,nl"'' c-Vin

a
Lock
s
Suspend
8
End Bl ock
3 D i s n o t correct. 1 30/o chose this.
4 Erythema multiforme is a severe skin eruption (shown in this image) that presents with macules, papules,
bulls, and target lesions. Stevens-Johnson syndrome is the major form of erythema multiforme and is
5 associated w1th exposure to sulfa and anticonvulsant drugs. It is characterized by h1gh fever and skin
6 ulceration, and is highly lethal. Erythema multiforme does not accompany rheumati Cfever.
Anticonvo a ot Erythema multiforme Rheumatic fever Stevens-Johnson syndrome Et f' oema Cutaneous condition
7 Mouth ulcer Sulfonamide (medicine) Peptic ulcer Papule Ulcer (dermatology) Fever
8
.9
• 10
• 11 E is n ot co r rect. 2 1 Ofo ch ose t his.
• 12 Janeway lesions are small erythematous lesions on the palms (indicated by arrows In this image) or soles
that result from embolic material thrown from valvular vegetations in the setting of acute bacterial
• 13 endocarditis. Although endocarditis can cause the appearance of a new heart murmur, given the patient's
history of childhood illness followed by development of mitral stenosis, rheumatic heart disease is the more
• 14
likely explanation. However, it should be noted that mitral stenosis caused by rheumatic heart disease can
• 15 predispose a patient to subacute bacterial endocarditis caused by less virulent organisms such as viridans
group streptococci. The primary care physician may advise the patient to use antibiotic prophylaxis during
• 16 dental procedures to reduce this risk.
Janeway lesion Endocarditis Heart murmur Rheumatic fever Antibiotics Mitral valve stenosis Streptococcus
• 17
Stenosis Embolism Subacute bacterial endocarditis Cardiovascular disease Infective endocarditis Erythema
• 18
Preventive healthcare Physician
• 19
• 20
• 21 Image courtesy of Ji Y, Kujtan L, Kershner 0
• - l?fl1?l

a
Lock
s
Suspend
8
End Block
Stenosis Embolism Subacute bacterial endocarditis Cardiovascular disease Infective endocarditis Erythema
2 Preventive healthcare Physician

3
4
Image courtesy of Ji Y, Kujtan L, Kershner D
5 - (2012)
6
7
Bottom Line:
8
Rheumatic fever can lea d to mitral stenosis, which is characterized by a lat e, rumbling, diastolic murmur at the cardiac apex. Numerous symptoms are
•9 associat ed with acute rheumatic fever; these can be remembered by the mnemonic FEVERSS ( Fever; Erythema marginatum, Valvular damage,
• 10 increased Erythrocyte sedimentation rat e, Red-hot joints, Subcutaneous nodules, Sydenham chorea) .
Rheumatic fever Mitral valve stenosis Chorea Stenosis Fever Diastole Mitral valve Rheumatism Heart murmur
· 11
• 12
• 13
I ill ;fi 1!1 I•J for year:[ 2017 ..
FIRST AID FAC TS
• 14
• 15
FA17 p 300.1
• 16
• 17
Rheumatic fever A consequence of pharyngeal infection with J'INES (major criteria):
group A ~-hemolytic streptococci. Late Joint (migratory polyarthritis)
• 18
sequelae include rheumatic heart disease, 'I (carditis)
• 19 wh ich affects heart valves-mitral > aortic>> Nodules in skin (subcutaneous)
• 20 tricuspid {high-pressure valves affected most). Erythema marginatum
• 21
. . . .
Early lesion is mitral valve regurgitation;
. . . . Syden ham chorea

6
lock
s
Suspend
0
End Block
Ite m: 8 of 48 ~ 1 • Ma rk -<:J 1>- Jil ~· !:';-~
..L Lab~lues
QIO: 3 8 11

1
Previous Next
- ..
Not es
. .
Calculator
.. . - •
Early lesion is mitral valve regurgitation; Sydenham chorea
2 late lesion is mitral stenosis. Associated
3 with Aschoff bodies (granuloma "it h giant
4 cells [blue arrO\\ S in fJ]), A n i t sch~O\\ cells
(enlarged macrophages with ovoid, wm y,
5
rod-like nucleus [red arrow in J), t anti-
6
streptolysin 0 (ASO) titers.
7 Immune mediated (tr pe II hypcrscnsiti\ ity);
8 not a direct effect of bacteria. ntibodie
.9 to M protein cross-react with self antigens
• 10
(molecular mimicry).
Treatment/prophylaxis: penicillin.
• 11
• 12
• 13 FA17 p 132.3

• 14 Viridans group Cram <±>, a -hemolytic cocci. They arc normal Scmguinis = blood. Think, "there is lots of
• 15 streptococci fl ora of the oropharynx that cause dental blood in the heart" (endocarditis). S sanguinis
caries (Streptococcus mutans and S mitis) makes dcxtrans, which bind to librin-platclct
• 16
and subacute bacterial endocarditis at aggregates on damaged heart va lves.
• 17
damaged heart valves (S sanguinis). Rcs i~t a nt iridans group strep live in the mouth because
• 18 to optochin, differentiating them from they are not afraid of-the-chin (op-to-chin
• 19 S pneumoniae, which is a -hemolr tic but is resistant).
• 20 optochin sensitive.
• 21

a
Lock Suspend
s 8
End Bl ock
Item: 9 of 48 ~ 1 • M k -<:J 1>- Jil ~· !:';-~
QIO: 3389 ..L ar Pre v ious Next Lab~lues Notes Calcula t o r

1
&

A 5-year-old girl is brought to the emergency department with a fever of 39.4°C (103°F) that has lasted 5 days. The mother says her child's lips have
become severely cracked and she has been unable to eat well. The mother also Is very concerned about her child's hands and feet being very red .
IA•A] &

2
Physical examination reveals severe conjunctivitis and cervical lymphadenopathy. The girl is very irritable and uncomfortable.
3
4 Which of the following is a dangerous complication t hat can result from the girl's acute disease?
5
6 A. Acute glomerulonephritis
7
B. Aortic stenosis
8
.9
c. Coronary aneurysms

• 10 o. Pericarditis

• 11 E. Renal papillary necrosis


• 12
• 13
• 14
• 15
• 16
• 17
• 18
• 19
• 20
• 21

a
Lock
s
Suspend
8
End Bl ock
Item: 9 of 48 ~ 1 • M k -<:J 1>- Jil ~· !:';-~
QIO: 3389 ..L ar Pre v ious Next Lab~lues Notes Calcula t o r
& &
1
Th e co rrect an sw er i s c . 570/o chose this.
2
This patient has Kawasaki disease, for which coronary aneurysms are the feared complication. Kawasaki
3 disease Is an acute, self-limiting disease that affects infants and children, especially those of Asian descent.
It Is characterized by acute, necrotizing vasculitis of small- or medium-sized vessels. It manifests with high
4 fever, congested nonpurulent conjunctivae, changes in lips/ ora l mucosa (" strawberry tongue" -shown in
this Image), a desquamating and nonvesicular rash, and lymphadenitis.
5
.a~ asa disease Vascul'tis Lymphadenopathy Desquamation Conjunctiva Rash /lleco os1s
6 Squamous epithelial cell Aneurysm Mucous membrane Fever
7
8
9
• 10
Image copyright © 2006 Yonsei University
• 11 College of Medicine
• 12
A is not co rrect. 1 5 0/o chose this •
• 13 Acute glomerulonephritis sometimes is seen in children after acute Streptococcus pyogenes infection.
• 14 Streptococcus pyogenes Glomerulonephritis Streptococcus Acute proliferative glomerulonephritis Infection

• 15 B is not co rrect . aoto chose this .


Aortic stenosis can be due to congenital defects, excessive wear and tear of the aortic valve over time, or scarring due to prior bouts with rheumatic fever.
• 16 Rheumatic fever Aortic stenosis Aortic valve Stenosis Congenital disorder Fever
• 17
D i s n ot co rrect . 140/o ch ose this .
• 18 Pericarditis Is characterized by inflammation of the outermost layer of the heart. It usually is caused by a variety of conditions including systemic lupus
erythematosus, rheumatoid arthritis, viral infections, uremia, and rheumatic fever (Streptococcus pyogenes) . Patients may present with pleurit ic chest
• 19 pain and a pericardia! friction rub on exam.
• 20 Streptococcu~ pyogenes Uremia Systemic lupus erythematosus Rheumatic fever Rheumatoid arthritis Pericarditis Pericardia! friction rub Pleurisy Inflammation
Streptococcus Chest pain Pericardium Arthritis Lupus erythematosus Virus Fever
• 21

a
Lock
s
Suspend
8
End Bl ock
Item: 9 of 48 ~- I • M k <:] t> al ~· ~
QIO: 3389 .l. ar Previous Next lab 'Vfl1 ues Notes Calculator

1 D is not correct. 14% chose this.

2 Pericarditis is characterized by inflammation of the outermost layer of the heart . It usually is caused by a variety of conditions including systemic lupus
erythem atosus, rheumatoid arthritis, viral infections, uremia, and rheumatic fever (Streptococcus pyogenes) . Patients m ay present with pleuritic chest
3 pain and a pericardia! friction rub on exam .
Streptococcus pyogenes Uremia Systemic lupus erythematosus Rheumatic fever Rheumatoid arthritis Pericarditis Pericardia! friction rub Pleurisy Inflammation
4
Streptococcus Chest pain Pericardium Arthritis lupus erythematosus Virus Fever
5
E is not correct. 6 % chose this.
6 Renal papillary necrosis is characterized by the sloughing of renal papillae along with hem aturia and proteinuria. Usually it is seen in patients with
7 diabet es m ellitus, acute pyelonephritis, chronic phenacetin use, and sickle cell anemia.
Phenacetin Renal papillary necrosis Pyelonephritis Hematuria Proteinuria Sickle-cell disease Diabetes mellitus Anemia Necrosis Kidney
8
9
Bottom Line:
0 10
Kawasaki disea se is characterized by acute, necrotizing vasculitis of small- or m edium-sized vessels, with fever; congested nonpurulent conjunctivae,
o ll changes in oral mucosa, rash, and lymphadenitis.
Kawasaki disease Vasculitis lymphadenopathy Oral mucosa Necrosis Conjunctiva Fever Mucous membrane Rash
0
12
0
13
0 14
l i j l ; f i i J I • l toryear:[ 2017 • ]
0 15 FI RST AI D FA CTS

0
16
0 17 FA17 p 302.1
Vasculitides
0
18
EPIDEMIOLOGY/PRESENTATION PATHOLOGY/lABS
0 19
Large-vessel vasculitis
0 20
Giant cell (tempo ral) Usually elderly females. Most commonly affects branches of carotid
0
21
I ln il'lt~ r.-. 1 h~., ,-1 .-.,...hA f ..~n-u"\1"\r•'l l "'r+Ar\1\

6
lock
s
Suspend
0
End Block
Item: 10 of 48 ~ 1 • M k -<:J 1>- Jil ~· !:';-~
QIO: 10 23 ..L ar Pre v ious Next Lab~lues Notes Calcula t o r

1
&

A 56-year-old white man is rushed to the emergency department after complaining of crushing substernal chest pain. He is morbidly obese,
diaphoretic, tachypneic, and clutching his chest. Initial ECG reveals ST-segment elevations in the anterior leads. The patient is stabilized and seems to
IA•A] &

2
be doing well. Several hours after initial presentation, he experiences sudden cardiac arrest and dies shortly thereafter.
3
4 Which of the following is the most likely cause of death in this patient?
5
6 A. Arrhythmia
7
B. Formation of ventricular septa l defect
8
c. Mural thrombosis
9
• 10 D. Myocardial pump failure

• 11 E. Rupture of ventricular free wall


• 12
F. Ruptured papillary muscle
• 13
• 14
• 15
• 16
• 17
• 18
• 19
• 20
• 21

a
Lock
s
Suspend
8
End Bl ock
Item: 10of48 ~. , . M k <:] t> al ~· ~
QIO: 1023 .l. ar Previous Next lab 'Vfl1ues Notes Calculator

1 •

2
3 The co rrect a nswer is A. 68% cho se this.

4
5
6
The patient died suddenly a few hours after an acute myocardial infarction (MI ). Fa t al arrhythmias are the most common cause of dea th (also known as
sudden cardiac dea th) in the first few hours of an infarction . Arrhythmias are due to disruption of the vascular supply to the conduction system, combined
with myocardial irritability after injury. The patient likely suffered from polymorphic ventricular t achycardia or ventricular fibrillation . Also, ventricular
t achyarrhythmias and intraventricular conduction abnormalities (eg, a left bundle branch block) are common long-t erm complications of an MI after the
myocardium becomes scarred and loses its intrinsic conducting abilities.
Ventricular fibrillation left bundle branch block Myocardial infarction Sudden cardiac death Cardiac muscle Tachycardia Ventricular tachycardia Cardiac arrhythmia
I
7
Bundle branch block Infarction Blood vessel Fibrillation
8
B is no t co rrect. 3 % cho se this.
9 Although less likely than a ventricular free-wall rupture, the interventricular septum also may rupture 3 - 7 days after infarction, lea ding to a ventricular
10 septal defect, a left -to-right intra cardiac shunt, and a low cardiac output.
Ventricular septal defect Interventricular septum Cardiac output Septum Ventricular system Shunt (medical)
· 11
c is no t co rrect. 7% cho se this •
• 12 After a myocardial infarction (MI ), there is a risk of thrombus formation over the infarcted area of endocardium due to turbulence in the blood flow. Clot
• 13 formation can lea d to a left -sided embolism; however; this is not the most common cause of dea th in the acute post-MI setting. Additionally, left
ventricular emboli would more likely lea d to ischemic strokes, not sudden cardiac dea th .
• 14 Endocardium Thrombus Myocardial infarction Sudden cardiac death Embolism Ischemia Infarction Heart Stroke Blood flow

• 15 D is no t co rrect. 6% cho se this.


• 16 Myocardial infarction (MI ) can lea d to further complications, such as congestive heart failure (CHF) and cardiogenic shock. However; these complications
rarely cause sudden cardiac dea th in a stabilized patient in the acute setting. Myocardial failure lea ding to CHF and poor cardiac output may t ake weeks
• 17 to months to occur and is the result of ventricular remodeling. Hence j3 -blockers and angiotensin-converting enzyme inhibitors are used widely in the post -
MI period to alter the neurohormonal milieu imposed by the renin-angiotensin-aldosterone system, and to counteract delet erious ventricular remodeling .
• 18 Cardiogenic shock Myocardial infarction Sudden cardiac death Heart failure Congestive heart failure Enzyme Cardiac output Angiotensin-converting enzyme
• 19 Ventricular remodeling ACE inhibitor Infarction Neurohormone Heart Renin-angiotensin system

• 20 E is no t co rrect. 11% cho se this.


• 21 Ventricular free -wall rupture is a complication that usually occurs 3 - 7 days after infarction because of the weakened wall of the damaged area during the
• infbrrHY\;::a tnnt r.oii11L::u · I".Ot'\l"n ;::a ni? ;::a tinn n•·n r.oc::c:: T h .o I.oft \t.ontrir 11L::u · fr·.o.o w ;::a ll k th.o mnd" lilr.oht c:it.o nf n 1nt1 u-.o 1:>11nt1 u-.o c:::hn11lrl it n rr1 u- l.o;::a rlc::: tn hl.o.orlinn intn

6
lock
s
Suspend
0
End Block
Item: 10of48 ~. , . M k <:] t> al ~· ~
QIO: 1023 .l. ar Previous Next lab 'Vfl1ues Notes Calculator
to months to occur and is the result of ventricular remodeling. Hence j3 -blockers and angiotensin-converting enzyme inhibitors are used widely in the post -
1
MI period to alter the neurohormonal milieu imposed by the renin-angiotensin-aldosterone system, and to countera ct delet erious ventricular remodeling.
2 Cardiogenic shock Myocardial infarction Sudden cardiac death Heart failure Congestive heart failure Enzyme Cardiac output Angiotensin-converting enzyme
Ventricular remodeling ACE inhibitor Infarction Neurohormone Heart Renin-angiotensin system
3
4 E is not correct. 11% chose this .
Ventricular free -wall rupture is a complication that usually occurs 3 - 7 days after infarction because of the weakened wall of the damaged area during the
5 inflammatory cellular reorganization process. The left ventricular free wall is the most likely site of rupture. Rupture, should it occur; lea ds to bleeding into
6
7

8
the pericardiaI space and fat al cardiac t amponade.
Cardiac tamponade Pericardium Tamponade Ventricular system Heart

F is not correct. 5% chose this .


A ruptured papillary muscle is a possible complication of myocardial infarction, but it most commonly occurs 3 - 7 days after the ischemic event. Papillary
I
9 muscle rupture is not typically the underlying cause of dea th in the acute setting. Ra ther; it could result in a low cardiac output and acute pulmonary
edema, likely requiring intubation until surgery can be performed to repair the valve.
10 Papillary muscle Myocardial infarction Pulmonary edema Cardiac output Ischemia Edema Muscle Intubation Infarction

· 11
• 12 Bottom Line:
• 13 Arrhythmia is the most likely cause of dea th both before patients present to the hospital and in the first few days after myocardial infarction . Other
complications include left ventricular failure, pulmonary edema, shock, t amponade from cardiac rupture, aneurysm formation, and Dressier syndrome .
• 14
Myocardial rupture Myocardial infarction Pulmonary edema Cardiac arrhythmia Aneurysm Heart failure Edema Dressier syndrome Cardiac tamponade Heart
• 15
• 16
• 17 lijj ;fi IJ l•l for year:l 2017 ..
FI RST AI D FA CTS
• 18
• 19
FA17p294.1
• 20
Evolution of Commonly occluded coronary arteries: LAD > RCA > circu mflex.
• 21
myocardial infarction Svmptoms: diaohoresis. nausea. vomiting, severe rctrosternal pain. pain in left arm and/or iaw.
6
lock
s
Suspend
0
End Block
2

3
4
5 FA17 p 296.1
Myocardial infa rction complications
6
7 Cardiac arrhythmia Occurs ''ithin the first fe,, da)S after \II. Important cause of death before reaching the hospital
and within the first 24 hours po t-\II.
8
Postinfarction Occurs 1-3 days after II. Friction rub.
9
fibrinous pericarditis
10
Papillary muscle Occurs 2-7 days after l\ II. Posteromedial papillar) muscle rupture fJ t risk due to single blood
• 11
rupture supply from posterior descending artery. Can result in severe mitral regurgitation.
• 12
Interventricular septal Occurs 3-5 clays after ~ II. 1\lacrophagc-mcd iatcd degradation ..... VSD.
• 13
• 14
• 15
rupture
Ventricular
pseudoa neurysm
Occurs 3-14 days after MI. Contained free 11'<111 rupture [l.J; ! CO, risk of arrhythmia, embolus
from mural thrombus.
I
• 16 formation
• 17 Ventricular free wall Occurs 5-14 days after MI. Free wa ll rupture ..... ca rdiac tamponade. L hypertrophy and
. 18 rupture previous :VII protect against free wall rupture.
• 19 True ventricular Occurs 2 weeks to several months after .\11. Outward bulge with contraction ("dyskinesia"),
• 20 aneurysm associated with fibrosis.
• 21 Dressler syndrome Occurs se,·eral weeks after ~ II. Autoimmune phenomenon resulting in fibrinous pericarditis.

a
Lock
s
Suspend
8
End Block
Item: 11 of 48 ~ 1 • M k -<:J 1>- Jil ~· !:';-~
QIO: 3820 ..L ar Pre v ious Next Lab~lues Notes Calcula t o r
& &
1
A 61-year-old man with a history of hypertension and diabetes presents to the emergency department 1 hour following the onset of severe chest
2 pain. The medical intern is startled when he finds asymmetric pulses In the patient's upper extremities.

3
Which of the following is the most likely directly fatal complication of the patient's condition?
4
5
A. Acute myocardial infarction
6
B. Aneurysm formation
7
8 c. Aortic insufficiency

9
D. Arrhythm ia
10
E. Pericardia! tamponade
• 11
• 12
• 13
• 14
• 15
• 16
• 17
• 18
• 19
• 20
• 21

a
Lock
s
Suspend
8
End Bl ock
Item: 11 of 48 ~ 1 • M k -<:J 1>- Jil ~· !:';-~
QIO: 3820 ..L ar Prev ious Next Lab~lues Notes Calculat o r

& &
1 Th e co rrect a nswer i s E. 46% chose this.
2 Given the history of hypertension, the nature of chest pain, and asymmetric pulses, the patient has a
clinical presentation that is consistent with an acute a ortic dissecti on until proven otherwise. The
3 presence of asymmetric pulses suggests t he involvement of the aortic arch (and possibly the ascending
aorta), where the right brachiocephalic artery may be affected, with sparing of the left subclavian artery.
4 The most feared complication of aortic dissection is rupture into any body cavity, including the pericardia I,
5 pleural, or peritoneal spaces. Per icar dia I t amp onade, leading to obstructive shock, is the cause of death
In two-thirds of patients with rupture of a proximal aortic dissection. Hemorrhagic shock secondary to
6 rupture is another common cause of death in patients with both proximal and distal aortic dissections. The
image Is a CT of an aortic dissection (red oval); note the presence of a dissection flap separating the aorta.
7 Note that ascendlllg aortic dissection is classically fe lt more as chest pain, whereas descending aortic
dissection Is classically felt more as back (interscapular) or abdominal pain, but the lines blur between this
8 distinction.
9 Subclavian artery Brachrocephalic artery Aorta Aortic dissection Aortic arch Ascending aorta Cardiac tamponade
Shock {circulatory) Hypertension Pleural cavity Pericardium Abdominal pain Chest pain
10
Anatomical terms of location Obstructive shock
11

• 12
• 13
• 14
A is not co rrect . 70fo chose this .
• 15 Myocardial infarctions occur in only 1% - 2% of acute aortic dissections. In this scenario, the dissection involves the coronary arteries, and care must be
taken to avoid thrombolytic therapy.
• 16 Thrombolytic drug Thrombolysis Coronary circulation Myocardial infarction Artery
• 17
B is not co rrect . 170/o ch ose this.
. 18 The development of aneurysms in the weakened aorta is a rather late complication of aortic dissections and is not a common cause of death In the Initial
presentation of a patient with an acute aortic dissection .
• 19 Aortic dissection Aorta Dissection Aneurysm
• 20
C i s not corr ect. 15% ch ose this•
• 21 Aortic Insufficiency often results from disruption of the aortic annulus secondary to retrograde dissection int o the aortic root. This complication can lead to

a
Lock
s
Suspend
8
End Block
Item: 11 of 48 ~. I • M k <:] t> al ~· ~
QIO: 3820 .l. ar Previous Next lab 'Vfl1 ues Notes Calculator

1 B is not correct. 17% chose this.


The development of aneurysms in the weakened aorta is a rather lat e complication of aortic dissections and is not a common cause of dea th in the initial
2
presentation of a patient with an acute aortic dissection .
3 Aortic dissection Aorta Dissection Aneurysm

4 c is not correct. 15% chose this.


5 Aortic insufficiency often results from disruption of the aortic annulus secondary to retrograde dissection into the aortic root . This complication can lea d to
chronic myocardial remodeling but is suitable for surgical repair.
6 Aortic insufficiency Cardiac skeleton Ascending aorta

7 D is not correct. 15% chose this.

8 In the case of aortic dissection, arrhythmias are not usually considered a primary complication but rather occur secondary to other direct complications,
such as myocardial infarction or rupture.
9 Aortic dissection Myocardial infarction Cardiac arrhythmia Dissection

10
11 Bottom Line:
• 12 Cardiac t amponade is a dangerous complication of aortic dissection, symptoms of which include t earing chest pain radiating to the back and asymmetric
pulses.
• 13 Cardiac tamponade Aortic dissection Chest pain Tamponade
• 14
• 15
• 16 I iii I;fi 1!1 I•J for year:l 2017 ..
FI RST AI D FA CTS
• 17

• 18
FA17 p 300.3
• 19
Cardiac tamponade Compression of the heart by Auid (cg, blood, effusions [arrows in r.JJ in pericardia) space) - l CO.
• 20
Equilibration of diastolic pressures in all 4 chambers.
• 21
.81 .
c.
. Find ings: Beck triad (hypotension, distended neck veins, distant heart sounds), f HR, pulsus

6
lock
s
Suspend
0
End Block
Item: 11 of 48 ~ 1 • M k -<:J 1>- Jil ~· !:';-~
QIO: 3820 ..L ar Pre v ious Next Lab~lues Notes Calcula t o r
& FIRST AID FAC TS &
1

2
FA17 p 300.3
3
Cardiac tamponade Compression of the heart b~ Auid (cg, blood, effusions (arrows in tJ] in pericardia) space) .... l CO.
4
Equilibration of diastolic pressures in all 4 chamber~.
5
Findings: Beck triad (hypotension, distended neck \'eins, distant heart sounds), f HR, pulsus
6 paradoxus. ECC shows low-,·oltage QR and electrical alternans (due to "swinging" movement of
7 heart in large effusion).
8 Pulsus paradoxus- l in amplitude of S) stolic BP by> 10 mm I lg during inspiration. Seen in
9 cardiac tamponade, asthma, obstructi' c sleep apnea, pericarditis, croup.
10
11

• 12 FA17p293.1
• 13 Aortic dissection Longitudinal intimal tear forming a fal se lumen (arrows in · show Aap extending into ascending
• 14 aorta, Stanford type A dissection). Associated with hypertension, bicuspid aortic valve, inherited
• 15 connective tissue disorders (eg, \tlarfan syndrome). Can present with tearing, sudden-onset chest
• 16
pain, radiating to the back+/- markedly unequal BP in arms. CXR shows mediastinal widening.
Can result in organ ischemia, aortic rupture, death. Two types:
• 17
• Stanford type A (proximal): involves -\scending aorta . .l\ lay extend to aortic arch or descending
. 18 aorta. May result in acute aortic regurgitation or cardiac tamponade. Treatment: surgery.
• 19 • Stanford type B (distal): only involves descending aorta (Below ligamentum arteriosum). Treat
• 20 medically with ~-blockers, then vasodilators.
• 21

a
Lock
s
Suspend
8
End Bl ock
Item: 12 of 48 ~ 1 • M k -<:J 1>- Jil ~· !:';-~
QIO: 4 0 55 ..L ar Pre v ious Next Lab~lues Notes Calcula t o r

1
2
&

A 65-year-old woman presents to her physician with the new onset of a severe headache and malaise. The headache is constant, unilateral, and
located In the frontotemporal region . The pain is not aggravated by light neck flexion . She has not experienced nausea, vomiting, or a rash, and there
lA• A] &

Is no history of head trauma or headache syndromes, but she has noted a 10-lb (4.5-kg) weight loss in the past 3 weeks. o n physical examination,
3 her temperature is 38. 1oc ( 100.6° F), blood pressure is 120/70 mm Hg, and heart rate is 90/min. MRI of the brain shows no abnormality. A biopsy Is
performed over the area of pain. Two magnifications from the same biopsy specimen are shown.
4
or

~ ~ ~.

~~
5 ('
6 \ .:f'; \
7 .. •I>

8 4 A

9
10 0
• .. 'I
11
• 12
• 13
• 14
• 15
• 16
• 17
• 18
• 19
Images courtesy of Dr. Michael Bonert
• 20
• 21
• What aaent Is most likelv to be an effective treatment for t his patient's condition?

a
Lock
s
Suspend
8
End Bl ock
2
3
4
5
6
7
8
9
10
I mages courtesy of Dr. Michael Bonert
11
• 12
What agent Is most likely to be an effective treatment for this patient's condition?
• 13
• 14 A . Aspirin and intravenous immune globulin
• 15
B. Azathioprine
• 16
• 17 c. Doxycycline

• 18 D. Prednisone
• 19
E. Sumatriptan
• 20
• 21

a
Lock
s
Suspend
8
End Block
Item: 12 of 48 ~ 1 • M k -<:J 1>- Jil ~· !:';-~
QIO: 4 0 55 ..L ar Prev ious Next Lab~lues Notes Calculat o r

& &
1 Th e co rrect an sw er i s o. 76% chose this.
The patient's symptoms (low-grade fever, weight loss, headache) and the biopsy findings .,
"',. • ' 1..~• '
2
Indicate that she has giant cell (temporal) arteritis, the most common form of vasculitis. ••
t
3
4
Hematoxylin-eosin-stained preparation, as shown in this high-power Image (blow-out
version from the vignette), demonstrates degeneration of the internal elastic lamina and
'' •

, .;

· ,
~
\.
\
•• •
... .. - ,.
, • '·~
replacement by granulomatous inflammation. The temporal artery is most commonly I , I • •
• •

I
J
5 affected, followed by other branches of the carotid artery (vertebral and ophthalmic ,
6
arteries). Corticosteroids are quite effective for managing temporal arteritis and must be
adm inistered quickly to prevent the serious complication of blindness. In the United States,
I
"· •
7
appro xim ately 15% of patients w ith tempora l arterit is also have polym ya lgla rheum atica.
Polymyalg· heumatica Giant-cell arteritis Vasculitis Corticosteroid Biopsy Grano 1om Headache
'
8 Superficial temporal artery Giant cell Visual impairment weight loss Inflammation Artery , I
, •
.'
9
10
Arteritis carotid artery Fever

• ~

'• /" .'.


I

I ~
I;
'
... . ' . . .....
-. •; '
,.,
11
12 Image courtesy of Or. Michael Bonert
'
- f.

• 13
A is not co rrect. 50fo chose this •
• 14 Kawasaki disease is treated with intravenous immune globulin and aspirin. It Is a vasculitis of medium -sized arteries that typically occurs in pediatric
patients, with findings of elevated erythrocyte sedimentation rate and c-reactlve protein, high feve1; strawberry tongue, lymphadenopathy, bright-red
• 15
cracked lips, conjunctivitis, and swollen hands and feet . Another name for Kawasaki disease is mucocutaneous lymph node disea se.
• 16 Lymph node Kawasaki disease C-reactive protein Erythrocyte sedimentation rate Aspirin Lymphadenopathy Red blood cell Conjunctivitis Strawberry tongue
Vasculitis Intravenous therapy Globulin Protein Artery Antibody Fever Lymph Pediatrics
• 17
B is not co rrect . 40/o ch ose this .
• 18
Azathioprine is a disease-modifying antirheumatic drug. Someone with rheumatoid arthritis would not present with this patient's symptoms. Rheumatoid
• 19 arthrit is Is an autoimmune disorder that m anifests with joint destruction. Headaches are not typical of rheumatoid arthritis.
Disease-modifying antirheumatic drug Azathioprine Rheumatoid arthritis Autoimmune disease Arthritis Autoimmunity
• 20
C i s not cor r ect. 5 % ch ose this•
• 21

a
Lock
s
Suspend
8
End Bl ock
Item: 12of48 ~. , . M k <:] t> al ~· ~
QIO: 4055 .l. ar Previous Next lab 'Vfl1 ues Notes Calculator

1
B is no t co rrect. 4 % cho se this.
2 Azathioprine is a disea se-modifying antirheumatic drug. Someone with rheumatoid arthritis would not present with this patient's symptoms. Rheumatoid
3 arthritis is an autoimmune disorder that manifests with joint destruction. Headaches are not typical of rheumatoid arthritis.
Disease-modifying antirheumatic drug Azathioprine Rheumatoid arthritis Autoimmune disease Arthritis Autoimmunity
4
c is no t co rrect. 5 % cho se this.
5 Doxycycline is used to trea t Rocky Mountain spotted fever. This infection often manifests with hea dache, but this would be accompanied by high fever;
6 rash not sparing the palms and soles, and constitutional symptoms.

I
Rocky Mountain spotted fever Doxycycline Headache Rash Spotted fever Infection Fever
7
E is no t co rrect. 1 0 % cho se this.
8 Serotonin agonists are sometimes used to trea t migraines, but this patient's clinical picture does not fit with migraine. Migraine hea dache is often one -
9 sided, exacerbat ed by activity and bright lights, and accompanied by nausea and vomiting.
Serotonin Migraine Serotonin receptor agonist Nausea Headache Vomiting Agonist
10
11
Botto m Li ne:
12
An elderly patient with hea daches or j aw pain that persists and with biopsy results demonstrating granulomatous inflammation and multinuclea t ed
• 13 giant cells has t emporal arteritis, which is trea t ed with corticosteroids and may be associat ed with polymyalgia rheumatica.
Polymyalgia rheumatica Giant-cell arteritis Corticosteroid Biopsy Granuloma Giant cell Inflammation Jaw Multinucleate Arteritis
• 14
• 15
• 16
l i j l ; f i i J I • l t o r yea r:[ 20 1 7 • ]
• 17 FI RST AID FAC T S

• 18
• 19 FA17 p 302.1

• 20
Vasculitides
EPIDEMIOLOGY/PRESENTATION PATHOLOGY/lABS
• 21

6
lock
s
Suspend
0
End Block
2
3
FA17p 488.1
4
5
Headaches Pain due to irritation of structures such as the dura, cranial nerves, or extracranial structures. \!lore
common in females, except cluster headaches.
6
CLASSIFICATION LOCALIZATION DURATION DESCRIPTION TREATMENT
7
Cluste,.. Unilateral 15 min-3 hr; Repetitive brief headaches. Acute: sumatriptan, lOO% 0 2
8 repetiti,·e Excruciating periorbital Prophylaxis: verapamil
9 pain "ith lacrimation and
10 rhinorrhea. ~lay present with
11
Horner syndrome.
12
Tension Bilateral > 30 min Steady pain. No photophobia Analgesics, NSAIDs,
(typically 4-6 or phonophobia. 1 o aura. acetaminophen;
• 13
h r); constant amitriptyline for chronic
• 14
pam

I
• 15
Migraine Unilateral 4-72 hr Pulsating pain with Acute: fSAIDs, triptans,
• 16 nausea, photophobia, or dihycl roergota mine
• 17 phonophobia. 1ay hm·e Prophylaxis: lifestyle changes
• 18 "aura." Due to irritation of (eg, sleep, exercise, diet),
• 19
C I , meninges, or blood P-blockers, calcium channel
,·essels (release of substance blockers, amitriptyline,
• 20
P, calcitonin gene-related topiramate, valproate .
• 21 peptide, vasoact i' e pepI ides). POU:\ 0 - Pulsatile, O ne-da)

a
Lock
s
Suspend
8
End Block
Item: 12of48 ~. , . M k <:] t> al ~· ~
QIO: 4055 .l. ar Previous Next lab 'Vfl1 ues Notes Calculator

1 • •
Other causes of headache include subarachnoid hemorrhage ("worst headache of my life"}, meningitis, hydrocephalus,
2 neoplasia, giant cell (temporal) arteritis.
3 • Compare with trigeminal neuralgia, which produces repetitive, unilateral, shooting pain in the distribu tion of CN V that
lasts (typically) for< 1 minute (note: first-l ine therapy is carbamazepine).
4
5
6 FA17 p 145.1

7 Rickettsial diseases Treatment: doxycycline (caution during pregnancy; alternative is chloramphenicol).


and vector-borne
8
illnesses
9
RASH COMMON
10
Rocky Mountain Rickettsia rickettsii, vector is tick. Despite its Classic triad-headache, fever, rash (vasculitis).
11
spotted fever name, d isease occurs primarily in the South Palms and soles rash is seen in C oxsackievirus
12 Atlantic states, especially orth Carolina. A infection (hand, foot, and mouth disease},
• 13 Ras h typically starts at wrists · and ankles and Rocky Mounta in spotted fever, and zoSyphilis
• 14 the n spreads to trunk, palms, and soles. (you dri ve C ARS using your palms and soles).
• 15 Typhus Endemic (Aeas}- R typhi. Rickettsii on the wRists, 1yphus on the Trunk.
• 16 Epidemic (human body louse}- R prowazekii.
Rash starts centrally and spreads out, sparing
• 17

I
palms and soles.
• 18
RASH RARE
• 19
Ehrlichiosis Ehrlichia, vector is tick. M onocytes with M EGA berry-
• 20 morulae rn
(mul berry-like inclusions) in M onocytes = Ehrlich iosis
• 21 cytoplasm. G ranulocytes = Anaplasmosis •

6
lock Suspend
s 0
End Block
Item: 13 of 48 ~ 1 • M k -<:J 1>- Jil ~· !:';-~
QIO: 1415 ..L ar Pre vious Next Lab~lues Notes Calcula t o r
& &
1
A 56-year-old woman presents to her physician due to recent onset of chest pain and dyspnea . Six weeks prior, the patient suffered a myocardial
2 Infarction. Her physical examination is remarkable for a friction rub over the fifth Intercostal space in the midclavicular line (heard in the audio clip),
along with an elevated jugular venous pressure.
3
OPEN MEDIA
4
5
6 II ___ oooo 1ooo1 ~~~

7
8 What Is the most likely cause of this patient's presentation?
9
10 A. Cardiac arrhythmia
11 B. Oressler syndrome
12
c. Left ventricular failure
• 13
• 14 o. Thromboembolism

• 15 E. Ventricular rupture
• 16
• 17
• 18
• 19
• 20
• 21

a
Lock
s
Suspend
8
End Bl ock
Item: 13of48 ~. , . M k <:] t> al ~· ~
QIO: 1415 .l. ar Previous Next lab 'Vfl1ues Notes Calculator

2
The co rrect a nswer is B. 72% cho se this.
3 Dressier syndrome is an autoimmune phenomenon that results in fibrinous pericarditis. This delayed pericarditis typically develops 2 -10 weeks post-
myocardial infarction (MI ) and presents clinically as chest pain and a pericardia! friction rub . It is generally trea t ed with aspirin, nonsteroidal anti-
4 inflammatory agents, or corticosteroids. On auscultation, the heart sound is usually continuous and is heard diffusely over the chest. It typically has three
5 components, one systolic and two diastolic, and is accentuat ed when the patient leans forward .
Pericarditis Uremic pericarditis Aspirin Pericardia! friction rub Auscultation Corticosteroid Autoimmune disease Nonsteroidal anti-inflammatory drug
6 Dressier syndrome Chest pain Pericardium Autoimmunity Systole Diastole Heart sounds Infarction Fibrin
7
A is no t co rrect. 4 % cho se this.
8 Ca rdiac arrhythmia is a common cause of post -myocardial infarction dea th, typically occurring the first few days following the event. It is not associat ed
with a friction rub .
9 Cardiac arrhythmia Pericardia! friction rub
10
c is no t co rrect. 9 % cho se this.
11 Left ventricular failure occurs in 60% of people who suffer a myocardial infarction and can present as congestive heart failure, which can cause chest pain,
dyspnea, and an elevat ed jugular venous pressure. No friction rub is typically present.
12 Dyspnea Myocardial infarction Heart failure Jugular venous pressure Congestive heart failure Chest pain Heart Jugular vein Vein Infarction
13 D is no t co rrect. 4 % cho se this •
• 14 Because ischemic/ scarred myocardial tissue lacks normal contractility, there is increa sed blood stasis and formation of large mural thrombi. Smaller
thromboemboli can break off these large mural thrombi and lea d to cerebrova scular accidents, transient ischemic attacks, and renal artery thrombosis .
• 15 Postmyocardial infarction arrhythmias are also a promoter of blood stasis and subsequent thromboembolic events. However; a friction rub does not
• 16 indicat e thromboembolism .
Thrombosis Renal artery Thrombus Transient ischemic attack Ischemia Cardiac arrhythmia Cerebrovascular disease Stroke
• 17
E is no t co rrect. 11% cho se this •
• 18 Ventricular rupture is a serious cause of postmyocardial infarction dea th that typically occurs 4 - 10 days after the initial event. It can present with
• 19 persistent chest pain, syncope, and distended jugular veins, but most often it presents with sudden dea th . A friction rub would not be observed .
Jugular venous pressure Myocardial rupture Syncope (medicine) Chest pain Jugular vein Ventricle (heart) Infarction
• 20
• 21

6
lock
s
Suspend
0
End Block
Item: 13of48 ~. , . M k <:] t> al ~· ~
QIO: 1415 .l. ar Previous Next lab 'Vfl1ues Notes Calculator

1 E is not correct. 11% chose this.


2 Ventricular rupture is a serious cause of postmyocardial infarction dea th that typically occurs 4 - 10 days after the initial event. It can present with
persistent chest pain, syncope, and distended jugular veins, but most often it presents with sudden death . A friction rub would not be observed.
3 Jugular venous pressure Myocardial rupture Syncope (medicine) Chest pain Jugular vein Ventricle (heart) Infarction
4
5
Bottom Line:
6 Fibrinous pericarditis is both an early and a late complication of myocardial infarction (MI ). It occurs as a component of Dressier syndrome several weeks
7 after an MI.
Uremic pericarditis Myocardial infarction Pericarditis Dressier syndrome Fibrin
8
9
10
lijj ;fi IJ l•l for year:l 2017 ..
FIRST AID FAC T S
11

12
FA17 p296.1
13
Myocardial infarction complications
• 14
Cardiac arrhythmia Occurs within the first few days after MI. Important cause of death before reaching the hospital
• 15
and within the first 24 hours post-MI.
• 16
Postinfarction Occurs l-3 days after MI. Friction rub.
• 17 fibrinous pericarditis
• 18
Papillary muscle Occurs 2-7 days after 11. Posteromedial papillary muscle rupture t'J t risk due to single blood
• 19 rupture supply from posterior descending artery. Can result in severe mitral regurgitation .
• 20 Interventricular septal Occurs 3-5 days after MI. Mac roph<~gc-mediated degradation - VSD.
• 21 rupture

6
lock
s
Suspend
0
End Block
Item: 13 of 48
QIO: 1415

2
~
..L
1 • M
ar
k

pulmonary edema
.
-<:J
Pre v ious
1>-
Next
.
regurgitation.
Jil
Lab~lues
:.:

Notes
:.: . !:';-~
Calcula t o r

p •p p ) p

3
4
5
6
7
8
9
10
11
12
13 FA1 7 p 456.3

• 14 Nonsteroidal Ibuprofen, naproxen, indomethacin, kctorolnc, diclofenac, meloxicam, piroxicam.


• 15 anti-inflammatory
• 16
drugs

• 17 MECHANISM Reversibly inhibit cyclooxygcnasc (both COX-I and COX-2). Block prostaglandin synthesis.
• 18 CLINICAL USE Antipyretic, analgesic, anti-inRammatory. Indomethacin is used to close a PDA.
• 19 ADVERSE EFFECTS Interstitial nephritis, gastric ulcer (prostaglandins protect gastric mucosa), renal ischemia
• 20 (prostaglandins \·asodilate afferent arteriole), aplastic anemia.
• 21

a
Lock
s
Suspend
8
End Bl ock
Item: 14 of 48 ~ 1 • M k -<:J 1>- Jil ~· !:';-~
QIO: 3590 ..L ar Pre v ious Next Lab~lues Notes Calcula t o r

1 &

An exceptionally tall 35-year-old man with long fingers and a history of poor vision presents to the emergency department complaining of severe
chest pain radiating to the middle of his back. The physicians believe that he Is having a heart attack and treat him accordingly. Sadly, he dies during
IA•A] &

2
the resuscitation attempt, and it was later discovered that he had a different fatal complication as a result of a chronic illness. A photomicrograph of a
3 specimen from his aorta is shown in the image.
4
5
6
7
8
9
10
11

12
13
• 14
• 15
• 16
• 17
. 18
• 19
What pathology is responsible for the complication that led to his death?
• 20
• 21
• A. Atherosclerosis

a
Lock
s
Suspend
8
End Bl ock
12
What pathology is responsible for the complication t hat led to his death?
13
• 14 A . Atherosclerosis
• 15
8 . Calcified aortic va lve
• 16
• 17 c. Cystic medial necrosis

• 18 o. Mitral valve prolapse


• 19
E. Syphilitic aortitis
• 20
• 21

a
Lock
s
Suspend
8
End Block
Item: 14 of 48 ~. I • M k <:] t> al ~· ~
QIO: 3590 .l. ar Previous Next lab 'Vfl1 ues Notes Calculator

1 The co rrect a nswer is c. 66% cho se this.


2 This image, showing buildup of mucopolysaccharide and the destruction of elastic fibers in the medial layer of the aortic root, demonstrat es cystic medial
necrosis. Cystic medial necrosis is most commonly found in patients with Marfan syndrome. This pathology increa ses the risk of aortic dissection, which
3 could manifest with sudden chest, abdominal, or back pain. Another frequently encountered heart problem among patients with Marfan syndrome is
mitral va lve prolapse.
4
Marfan syndrome Glycosaminoglycan Aortic dissection Mitral valve prolapse Familial thoracic aortic aneurysm Mitral valve Tunica media Elastic fiber Aorta
5 Ascending aorta Necrosis Pathology Back pain Prolapse
6 A is no t co rrect. 6 % cho se this.
7 Histologically, atherosclerosis manifests as a lesion of the intima and may demonstrat e hyperplasia, collagen deposition, and the accumulation of foam
cells or cholesterol clefts; the image shows focal necrosis of the media but no evidence of atherosclerosis. Atherosclerosis is typically painless, though an
8 atheroma may rupture and embolize, causing occlusion of a vessel and subsequent painful ischemia.
Atheroma Atherosclerosis Collagen Embolization Embolism lesion Histology Tunica intima Cholesterol Necrosis Foam cell Ischemia Hyperplasia
9
Vascular occlusion
10
B is no t co rrect. 8% cho se this.
11
Ca lcified aortic valves are common in the elderly and may lea d to aortic stenosis. However; they would not cause the pathology demonstrat ed in the
12 micrograph, which shows an area of elastic fiber destruction and mucopolysaccharide accumulation within the media of the aorta.
Glycosaminoglycan Elastic fiber Aortic stenosis Aorta Stenosis Aortic valve Calcification Pathology
13
D is no t co rrect. 13% cho se this.
14
Marfan syndrome is associat ed with mitral va lve prolapse. However; the image is from the patient's aorta, and mitral valve prolapse is not directly relat ed
• 15 to aortic dissection .
Marfan syndrome Mitral valve prolapse Aortic dissection Aorta Mitral valve Prolapse
• 16
E is no t co rrect. 7 % cho se this •
• 17
Syphilis can lea d to occlusion of the vasa va sorum (the small blood vessels feeding the aortic wall, shown at the top of the image in the adventitia) and
• 18 subsequently cause weakening or focal damage to the aorta that increa ses the risk for aortic dissection . However; this image shows a focal area of cystic
necrosis in the media, a finding specific for Marfan syndrome .
• 19 Marfan syndrome Aortic dissection Vasa vasorum Syphilis Aorta Adventitia Necrosis Dissection Blood vessel Vascular occlusion
• 20
• 21
Botto m Li ne:

6
lock
s
Suspend
0
End Block
Item: 14 of 48 ~. I • M k <:] t> al ~· ~
QIO: 3590 .l. ar Previous Next lab 'Vfl1 ues Notes Calculator

1 Syphilis can lead to occlusion of the vasa vasorum (the small blood vessels feeding the aortic wall, shown at the top of the image in the adventitia ) and
subsequently cause weakening or focal dam age to the aorta that increases the risk for aortic dissection. However; this image shows a focal area of cystic
2
necrosis in the m edia, a finding specific for Marfan syndrome.
3 Marfan syndrome Aortic dissection Vasa vasorum Syphilis Aorta Adventitia Necrosis Dissection Blood vessel Vascular occlusion

4
5 Bottom Line:
6 Patients with Marfan syndrome are at high risk for developing hea rt problems, including cystic m edial necrosis of the aortic root and mitral valve
prolapse.
7 Marfan syndrome Mitral valve prolapse Familial thoracic aortic aneurysm Mitral valve Ascending aorta Aorta Prolapse Necrosis
8
9
10 liU ;fi 1!1J t) for year:[ 2017
FI RST AID FA CTS
.. ]
11

12
FA11 p 56.1
13
Autosomal dominant Achondroplasia, autosomal dominant polycystic kidney disease, fam ilial adenomatous polyposis,
14
diseases familial hypercholesterolemia, hereditary hemorrhagic telangiectasia, hereditary spherocytosis,
• 15 Huntington disease, Li-Fraumeni syndrome, Marfan syndrome, multiple endocrine neoplasias,
• 16 neurofibromatosis type l (von Recklinghausen disease), neurofibromatosis type 2, tuberous
• 17 sclerosis, von Hippei-Lindau disease.
• 18
• 19 FA11 p48.1

• 20 Elastin Stretchy protein within skin, lungs, large arteries, elastic ligaments, vocal cords, ligamenta Aava
• 21
(connect vertebrae - relaxed and stretched conformations).

6
lock
s
Suspend
0
End Block
Item: 14 of 48 ~ 1 • M k -<:J 1>- Jil ~· !:';-~
QIO: 3590 ..L ar Pre v ious Next Lab~lues Notes Calcula t o r

1
&
""' -- - -·- .. . . - - - &

and eyes. FB 1 gene mutation on ch romosome 15 results in defective librillin, a glycoprotein


2 that forms a sheath around elastin. Findings: tall with long extremities; pectus carinatum
3 (more specific) or pectus excavatum; hypermobile joints; long, tapering fingers and toes
4 {arachnodactyly); cystic medial necrosis of aorta; aortic incompetence and dissecting aortic
aneurysms; Aoppy mitral ,·ahe. Subluxation of lenses, typically upward and temporally.
5
6
FA17 p290.1
7
Congenital cardiac DISORDER DEFECT
8
defect associations Alcohol exposure in utero (fetal alcohol SO, PDA. ASD, tetralogy of Fallot
9
syndrome)
10
Congenital rubella PDA, pulmonary artery stenosis, septal defects
11
Down syndrome AV septal defect (endocardial cushion defect),
12
VSD,ASD
13
Infant of diabetic mother Transposition of great Yessels
14
[arfan syndrome MVP, thoracic aortic aneurysm and dissection,
• 15
aortic regurgitation
• 16
Prenatal lithium exposure Ebstein anomaly
• 17
Turner syndrome Bicuspid aortic valve, coarctation of aorta
• 18
Williams syndrome Supravalvular aortic stenosis
• 19
22qll syndromes Truncus arteriosus, tetralogy of Fallot
• 20
• 21

a
Lock
s
Suspend
8
End Bl ock
Item: 15 of 48 ~ 1 • M k -<:J 1>- Jil ~· !:';-~
QIO: 1426 ..L ar Pre v ious Next Lab~lues Notes Calcula t o r

1
2
&

A 33-year-old man presents with increasing pain in his hands. On physical examination, his hands are cold and tender to palpation, with some regions
of ulceration and dry gangrene over the digits. There is also marked pallor, which the patient claims is exacerbated by cold weather. He has decreased
lA• A] &

radial, ulnar, posterior tibial, and dorsalis pedis pulses, but preserved brachial and popliteal pulses. The rest of the examination is unremarkable.
3
4 Which of the following would be most useful in determining the cause of the patient's condition?
5
6 A. Alcohol history
7
B. Family history
8
c. Sexual history
9
10 o. Smoking history

11 E. Travel history
12
13
14
0 15
0
16
0
17
0
18
• 19
0 20
• 21

a
Lock
s
Suspend
8
End Bl ock
Item: 15 of 48 ~ 1 • M k -<:J 1>- Jil ~· !:';-~
QIO: 1426 ..L ar Prev ious Next Lab~lues Notes Calculat o r

& &
1
Th e co rrect an sw er i s D. 720/o chose this.
2
This patient exhibits signs of Buerger's disease. Buerger's, also known as thromboangiitis obliterans, is a small- to medium-vessel vasculitis. It most often
3 affects young men from South Asia and the Middle East. Because the pathophysiology of Buerger's is essentially small-vessel ischemia, its clinical
presentation mimics that of peripheral arterial disease (including distal pulses being decreased as compared with proximal pulses; thus the brachial and
4 popliteal pulses can still be palpated, whereas the radial, ulnar, and dorsalis pedis pulses cannot). Buerger's disease can also include Raynaud's
5 phenomenon because of peripheral nerve ischemia. Tobacco use is the strongest predispos111g risk factor, and a positive smoking history sharply points
clinical suspicion toward Buerger's. Smoking cessation is first-line treatment.
6 Peripheral 1rte1 y disease Raynaud's phenomenon Thromboang"itis obl;terans Smo 1ng cessation Ischemia Vasculitis Tobacco Tobacco smoking Distal

7 A i s not correct. 4 % chose this.


Alcohol use is not typically related to Buerger's disease or the other vasculitides.
8
'"hromboangiitis obliterans Vasculitis Alcohol
9
B i s not co rrect. 1 6 % chose this.
10 Family history is not useful in diagnosing Buerger's disease. A family history of diabetes or hypertension may predispose an individual to peripheral
11 arterial disease (PAD), which carries a similar presentation, but the two diseases arise from separate etiologies. In differentiating between the two, PAD
Is an atherosclerotic disease that typically shows up in older individuals.
12 Thromboangiitis obliterans Peripheral artery disease Hypertension Diabetes mellitus Atherosclerosis Etiology Family history (medicine)

13 C is not co rrect. soto chose this.

14 Buerger's disease is not known to be associat ed with any sexually transmitted diseases. The question stem paints the picture of a vascular disease. The
most Important infectious disease relat ed to the vascular system is tertiary syphilis, which can cause an ascending aortic aneurysm.
15 Thromboangiitis obliterans Syphilis Aortic aneurysm Infection Aneurysm Circulatory system Vascular disease Tertiary syphilis Blood vessel

• 16 Sexually transmitted infection

• 17 E i s n ot co rrect . J O/o ch ose this .


A travel history, like sexual history, is useful in diagnosing infectious diseases. It would not help in diagnosing a vasculitis. Leprosy is an infectious disease
. 18 that causes ulcerations and gangrene, but it does not fit the rest of the clinical vignette.
• 19 Leprosy Gangrene Vasculitis Ulcer (dermatology) Infection Peptic ulcer

• 20
• 21 Bottom Line:

a
Lock
s
Suspend
8
End Bl ock
Item: 15of48 ~- , . M k <:] t> al ~· ~
QIO: 1426 .l. ar Previous Next lab 'Vfl1ues Notes Calculator
' ' + • • '+
1
A travel history, like sexual history, is useful in diagnosing infectious diseases. It would not help in diagnosing a vasculitis. Leprosy is an infectious disease
2 that causes ulcerations and gangrene, but it does not fit the rest of the clinical vignette .
leprosy Gangrene Vasculitis Ulcer {dermatology) Infection Peptic ulcer
3
4
5
Bottom Line:
Buerger's disease is a thrombosing, small-/medium-vessel vasculitis in young m en that presents similarly to peripheral arterial disease. It is strongly
6 associated with smoking, and in fact, smoking cessation is the single best intervention in slowing the progression of the disease.
7 Thromboangiitis obliterans Peripheral artery disease Smoking cessation Vasculitis

8
9
10
lijl;fiiJI•l toryear:[2017 • ]
FI RST AI D FA CTS

11

12 FA17 p 302.1

13 Vasculitides
EPIDEMIOLOGY/PRESENTATION PATHOLOGY/LABS
14
Large-vessel vasculitis
15
• 16 Giant cell (temporal) Usually elderly females. Most commonly affects branches of carotid
arteritis Unilateral headache (temporal artery}, jaw artery.
• 17
claudication. Focal granulomatous inAammation rJ.
• 18
May lead to irreversible blindness due to t ESR.
• 19 ophthalmic artery occlusion. Treat with high-dose corticosteroids prior to
• 20 Associated with polymyalgia rheumatica. tempore.! artery biopsy to prevent blindness.
• 21 Takayasu arteritis Usua lly Asian fema les< 40 years old. Granulomatous thickening and narrowing of

6
lock
s
Suspend
0
End Block
11
12
FA17 p 445.3
13
Raynaud phenomenon l blood Row to skin due to arteriolar (small vessel) vasospasm in response to cold or stress:
14
color change from white (ischemia) to blue (hypoxia) to red (rcpcrfusion). Most often in the
15 fingers fJ and toes. Called Raynaud disease when 1° (idiopathic), Raynaud syndrome when zo
• 16 to a disease process such as mixed connective tissue disease, SLE, or C REST syndrome (limited
• 17 form of systemic sclerosis). Digital ulceration (critica l ischemia) seen in zo Raynaud syndrome.
• 18
Treat with Ca2+ channel blockers.
• 19
• 20
• 21

a
Lock
s
Suspend
8
End Block
Item: 16 of 48 ~ 1 • M k -<:J 1>- Jil ~· !:';-~
QIO: 1435 ..L ar Pre v ious Next Lab~lues Notes Calcula t o r

1
&

A 52-year-old man presents to the emergency department with severe substernal chest pain radiating to his left arm, together with diaphoresis and
dyspnea. He reports that he has never experienced this kind of pain before, that It came on very suddenly about 30 minutes ago, and that he was not
IA•A] &

2
doing anything particularly strenuous. His blood pressure is 137/87 mm Hg, his pulse Is 96/min, and his respiratory rate is 24/min. An ECG shows ST-
3 segment elevation in leads v 3 - v 6 . While in the emergency department he goes Into cardiac arrest, and efforts to resuscitate him are unsuccessful.
4
5 Which of the following is the most likely cause of this patient's death?

6
A. Arrhythmi a
7
8 B. Congestive heart fa ilure
9 c. Fi bri nous pericard it is
10
o. Left ventricular rupture
11
E. Papillary muscle rupture
12
13
14
15
• 16
• 17
• 18
• 19
• 20
• 21

a
Lock
s
Suspend
8
End Bl ock
Item: 16 of 48 ~ 1 • M k -<:J 1>- Jil ~· !:';-~
QIO: 1435 ..L ar Prev ious Next Lab~lues Notes Calculat o r

& &
1
2 Th e correct an sw er i s A. 7 4 0/o chose this.
3 This Individual is having a myocardial infarction (likely anterolateral given V3-V6 distribution), as suggested by his symptoms and ST-segment elevation
on ECG. Sudden cardiac death in the setting of an MI is most commonly caused by ventricular tachyarrhythmias, in particular ventricular fibrillation (VF).
4 These arrhythmias are thought to be a result of the damaged, ischemic myocardium producing a substrate that is capable of abnormal reentry circuits.
5 This results in the loss of coordinated contraction of the heart and perfusion of vital organs. VF is almost universally lethal if not treated; it very rarely
reverts spontaneously to sinus rhythm. CPR, epinephrine, and defibrillation are Indicated for VF.
6 Vento ocu~a fib ~ation Epinephr;ne Myocardial infarction Sudden cardiac death Ca do a musc.e Defibr lation Sinus rhythm cardiac arrhythmoa Ischemia
Cardiopo mo oa f resuscitation Perfusion Fibrillation Infarction
7
8 B i s not correct. 5% chose this.
Most often, congestive heart failure (CHF) is a later seque la of an MI that affects a large area of myoca rdial tissue. It results from functional myocardium
9 being replaced with nonfunctional scar t issue, decreasing the heart's ability to contract and leading to decreased ejection fraction. Occasionally, you can
10 get acute CHF with flash pulmonary edema in the setting of an MI that Is causing severe left ventricular dysfunction. However, the most common cause of
sudden cardiac death in the setting of MI is arrhythmia.
11 Sequela Pulmonary edema Heart failure Congestive heart failure Sudden cardiac death Cardiac muscle Ejection fraction Cardiac arrhythmia
Flash pulmonary edema Edema Ventricle (heart) Scar
12
13 C is not co rrect. 40/o chose this.
Fibrinous pericarditis is associated with transmural infarctions and typically occurs 1-3 days post -MI. Some degree of diastolic dysfunction may occur due
14 to Inability of the ventricle to fully relax due to the exudate that collects In the pericardia I sac. Physical examination typically reveals a friction rub .
15 Dressier syndrome is an autoimmune condition that appears several weeks after an Ml and also results in a fibrinous or serofibrinous pericarditis.
Pericarditis Uremic pericarditis Diastolic heart failure Exudate Autoimmune disease Ventricle (heart) Diastole Autoimmunity Dressier syndrome Pericardium
16 Physical examination Pericardia! friction rub
• 17 D is n ot co rrect. 100/o ch ose this.
• 18 Left ventricular rupture is a complication that usually occurs 3- 7 days post-MI, when the ventricular wall is in the dynamic process of resorbing necrotic
t issue and building new tissue to fill the defect. If this were the case, the patient may present with the Beck triad (hypotension, increased jugular venous
• 19 pressure, and distant heart sounds), secondary to hemopericardium. Risk factors for free wall rupture include female sex, age >60 years, and preexisting
• 20 hypertension. Many patients present with sudden death and do not present with Beck triad when there is a rupture of the left ventricular free wall with
massive blood loss and hemodynamic failure.
• 21 Hemoper cardium Myocardial rupture Jugular venous pressure Hypotens1on Hypertensoon Heart sounds Necrosis Heart Bleeding Jugular vein Vein Hemodynamics

a
Lock
s
Suspend
8
End Block
Item: 16of48 ~. , . M k <:] t> al ~· ~
QIO: 1435 .l. ar Previous Next lab 'Vfl1ues Notes Calculator

1 D is not correct. 10% chose this .


Left ventricular rupture is a complication that usually occurs 3 - 7 days post-MI, when the ventricular wall is in the dynamic process of resorbing necrotic
2
tissue and building new tissue to fill the defect. If this were the case, the patient may present with the Beck triad (hypotension, increa sed jugular venous
3 pressure, and distant heart sounds), secondary to hemopericardium . Risk factors for free wall rupture include female sex, age > 60 years, and preexisting
hypertension. Many patients present with sudden dea th and do not present with Beck triad when there is a rupture of the left ventricular free wall with
4 massive blood loss and hemodynamic failure.
Hemopericardium Myocardial rupture Jugular venous pressure Hypotension Hypertension Heart sounds Necrosis Heart Bleeding Jugular vein Vein Hemodynamics
5
Ventricle (heart)
6
E is not correct. 7 % chose this .
7
Papillary muscle rupture occurs 2 - 7 days after an MI and results in mitra l regurgitation . Severe mitral regurgitation can lea d to left heart failure and
8 pulmonary edema.
Papillary muscle Pulmonary edema Heart failure Edema Mitral insufficiency Regurgitation (circulation) Muscle Heart
9
10
Bottom Line:
11
The most common cause of dea th immediat ely following an MI is an arrhythmia that lea ds to cardiac arrest.
12 Cardiac arrhythmia Cardiac arrest
13
14
15 I iii I;fi 1!1 I•J for year:l 2017 ..
FI RST AI D FA CTS
16
• 17
FA17 p296.1
• 18 Myocardial infarction complications
• 19 Cardiac arrhythmia Occurs within the fi rst few days after MI. Important cause of death before reaching the hospital
• 20 and within the first 24 hours post-M I.
• 21 Postinfarction Occurs l-3 days after MI. Friction rub.

6
lock
s
Suspend
0
End Block
2
3
4
FA17 p 294.1
5
Evolution of Commonly occluded coronar) arteries: LAD > RCA> circumflex.
6 myocardial infarction S} mptoms: diaphoresis, nausea, \Omiting, se,ere retrosternal pain, pain in left arm and/or ja\\,
7 shortness of breath, fatigue.
8 TIME GROSS liGHT MICROSCOPE COMPliCATIONS
9 0 - 24 hr None Early coagulati' e necrosis, entricular arrhythmia, II F',
10 release of necrotic cell cardiogenic shock.
11 contents into blood; edema,
hemorrhage, \\'3\'Y fibers.
12 Occluded eutroph ils appear.
13 Reperfusion injury,
14 a~sociated with general ion
of free rad icals, leads to
15
~i- Dark mottling. hypercontraction of myofibrils
16 pale with through t Free calcium influx.
• 17
• 18
• 19
tetrazolium
Stain

-
- -.. Q

• 20
• 21

a
Lock
s
Suspend
8
End Block
Item: 16 of 48 ~ 1 • M k -<:J 1>- Jil ~· !:';-~
QIO: 1435 ..L ar Pre v ious Next Lab~lues Notes Calcula t o r
& &
1

2
FA17 p295.1
3
4 Diagnosis of In the first 6 hours, ECC is the gold standard.
5
myocardial infarction Cardiac troponin I rises after -f hours (peaks e
- 50
at 24 hr) and is f for 7-10 days; more speci fi c ~
6 0
than other protein markers. ~ 10
7
8
C K-MB rises after 6-12 hours (peal s at
16-2-f hr) and is predominantly found
-
~
:::>
0 5
9 in m,·ocardium but can also be released ~
' a. C MB
10
from skeletal muscle. Useful in diagnosing =
:::>

2
reinfarction following acute ~ II because b els 1
11
return to normal after 48 hours. 1 2 3 4 5 6 7 8
Days after Ml onset
12 Large Mls lead to greater elevations in troponin
13 I and CK-M B. Exact curves vary with testing
14 procedure.
ECC changes ca n include ST elevation
15
(STEM!, transmural infarct), ST depression
16
( ISTEMI, subendocardial infarct),
• 17 hyperacute (peaked) T wm·cs, T-wavc
. 18 inversion, new left bundle branch block, and
• 19 pathologic Q waves or poor R " a\ e progression
• 20
(evolving or old transmural infarct).

• 21

a
Lock
s
Suspend
8
End Bl ock
Item: 17 of 48 ~ 1 • M k -<:J 1>- Jil ~· !:';-~
QIO: 2952 ..L ar Pre v ious Next Lab~lues Notes Calcula t o r

1 &

An 85-year-old man dies of aspiration pneumonia as a complication of Alzheimer disease. Autopsy reveals a small (230 g) heart that appears grossly
dark brown in color. Hematoxylin and eosin staining of cardiac muscle cells Is shown In the image below. A liver biopsy is taken, and a similar staining
IA•A] &

2
pattern Is noted in the hepatocytes. The pathologist determines this phenomenon to be a consequence of age and not a causative agent in the
3 patient's death.
4
5
6
7
8
9
10
11

12
13
14
15
16
• 17
. 18
• 19 Accumulation of which of the following substances is the most likely cause of the brown pigmentation seen in this specimen?
• 20
• 21 A. Bilirubin

a
Lock
s
Suspend
8
End Bl ock
10
11 Accumulation of which of the following substances is the most likely cause of the brown pigmentation seen in this specimen?
12
13 A. Bilirubin
14 8. Cholesterol
15
c. Glycogen
16
• 17 o. Hemosiderin

. 18 E. Lipofuscin
. 19
F. Melanin
• 20
. 21

a
Lock
s
Suspend
8
End Block
Item: 17 of 48 ~. I • M k <:] t> al ~· ~
QIO: 2952 .l. ar Previous Next lab 'Vfl1 ues Notes Calculator

1 The co rrect a nswer is E. 59 % cho se this.


2 The combination of an atrophic heart and lipofuscin accumulation is referred to as brown atrophy. Lipofuscin is a non-degradable pigment formed due to
iron-cat alyzed oxidative process from autophagocytosed organelles. Lipofuscin accumulat es slowly over the years and give cells a brown and granular
3 appearance. This is a benign finding in elderly patients. It is deposited in the tissues of many different organs including the liver; kidney, heart muscle,
retina, adrenals, ganglion cells, and any non-replicating cell in the body. Accumulation in postmitotic cells is inevitable, wherea s proliferative cells
4
efficiently dilute it during division.
5 lipofuscin Retina Atrophy Ganglion Kidney liver Organelle Retinal ganglion cell GO phase Pigment Cardiac muscle Mitosis Benign tumor Ganglion cell Redox
Muscle
6
7 A is no t co rrect. 4 % cho se this.
Bilirubin can accumulat e and stain internal organs, producing a yellowish discoloration called j aundice. However; lack of clinical evidence of a hemolytic or
8 obstructive process in this case makes this option highly unlikely. Furthermore, accumulation of bilirubin is a pathologic process and is not considered a
normal consequence of aging.
9
Bilirubin Jaundice Hemolysis
10
B is no t co rrect. 5 % cho se this.
11 Cholesterol may accumulat e as atheromatous plaques in the coronary arteries; however; it would not be seen intra cellularly with a hematoxylin and eosin
stain. Furthermore, its accumulation is not associat ed with cardiac atrophy or brown pigmentation .
12
Atheroma H&E stain Eosin Cholesterol Haematoxylin Coronary circulation Artery
13
c is no t co rrect. 3 % cho se this.
14 Glycogen accumulation may be seen in glycogen stora ge disorders such as von Gierke disea se and Pompe disea se. These are inherited conditions that
generally manifest early in life. A periodic acid-Schiff stain would be best to visualize glycogen, rather than a hematoxylin and eosin stain. Furthermore, a
15
build-up of glycogen would be evidence of a pathologic process, not a normal effect of aging.
16 Glycogen storage disease Glycogen Periodic acid-Schiff stain Glycogen storage disease type I Eosin Haematoxylin H&E stain Glycogen storage disease type II
Periodic acid-Schiff
17
• 18 D is no t co rrect. 26% cho se this •
Hemosiderin is a hemoglobin-derived, iron-containing pigment that consists of aggregat es of ferritin . Small amounts of hemosiderin can be seen in
• 19 normal tissue macrophages found in the bone marrow, liver; and other tissues. Excess amounts are seen in disea se stat es such as hemochromatosis,
particularly in hepatocytes, cardiac myocytes, and pancrea tic islet cells. Hemosiderin deposits within the heart would appear as brownish gra nules that
• 20
turn blue when stained with Prussian blue. The cardiac complications of hemochromatosis include restrictive cardiomyopathy and arrhythmias but not
• 21 atrophy.

6
lock
s
Suspend
0
End Block
Item: 17 of 48 ~. I • M k <:] t> al ~· ~
QIO: 2952 .l. ar Previous Next lab 'Vfl1 ues Notes Calculator

1 • •

2
D is not co rrect. 26 % chose this .
Hemosiderin is a hemoglobin-derived, iron-containing pigment that consists of aggregat es of ferritin . Small amounts of hemosiderin can be seen in
3 normal tissue m acrophages found in the bone m arrow, liver; and other tissues. Excess amounts are seen in disea se stat es such as hemochromatosis,
particularly in hepatocytes, cardiac myocytes, and pancrea tic islet cells. Hemosiderin deposits within the hea rt would appea r as brownish granules that
4 turn blue when stained with Prussian blue . The cardiac complications of hemochromatosis include restrictive cardiomyopathy and arrhythmias but not
5 atrophy.
Ferritin Hemosiderin Iron overload Prussian blue Cardiomyopathy Macrophage liver Hepatocyte Bone marrow Atrophy Cardiac arrhythmia Bone
6
Restrictive cardiomyopathy
7
F is not co rrect. 3 % chose this .
8 Melanin is an endogenous pigment formed when the enzyme tyrosinase cat alyzes the oxidation of tyrosine in m elanocytes. It appea rs brown-black in
color. It is not found in cardiac tissue and thus would not be considered to be a normal consequence of aging.
9
Tyrosinase Catalysis Melanin Enzyme Pigment Tyrosine Melanocyte Redox Endogeny (biology)
10
11
Botto m Line :
12
Lipofuscin is a non-degradable pigment that is deposited in the tissues of m any different organs (liver; kidney, hea rt muscle, retina, adrenals, ganglion
13 cells, and any non-replicating cell in the body) as a part of the aging process. It is composed of oxidized and polymerized m embrane lipids from
autophagocytosed organelles that accumulat e slowly over yea rs.
14 lipofuscin Retina Organelle Ganglion Kidney liver Pigment Retinal ganglion cell lipid Cardiac muscle Redox lipid bilayer Muscle Ganglion cell Biological membrane
Adrenal gland
15
16
17
• 18
I ill ;fi 1!1 I•J fo r yea r:l 20 1 7 ..
FI RST AI D FA CTS

• 19
0 20 FA11 p 213.2

• 21 Lipofuscin A yellow-brown "wear and tear" pigment r.J associated with normal aging.

6
lock
s
Suspend
0
End Block
Item: 18 of 48 ~ 1 • M k -<:J 1>- Jil ~· !:';-~
QIO: 2144 ..L ar Pre v ious Next Lab~lues Not es Calcula t o r

1
2
&

A 15-year-old high school student presents to an outpatient clinic with a sore throat and fever. He complains of pain with swallowing and denies
cough. A physical examination reveals tonsillar exudates and the student tests positive for Streptococcus pyogenes. He asks the physician what could
lA• A] &

happen to him if he does not take the prescribed antibiotics.


3
4 Taking antibiotics during this infection will decrease the likelihood of which of the following sequelae?
5
6 A. Bacterial invasion of the fascia and muscle
7
B. Deposition of immun e complexes in the glomerular basement membrane
8
c. I mpetigo
9
10 D. Mitral stenosis

11 E. Toxic shock syndrome


12
13
14
15
16
17
. 18
. 19
• 20
. 21

a
Lock
s
Suspend
8
End Bl ock
Item: 18 of 48 ~ 1 • M k -<:J 1>- Jil ~· !:';-~
QIO: 2144 ..L ar Prev ious Next Lab~lues Not es Calculat or
& &
1
Th e co rrect a n sw er i s D. 590/o chose this .
2
There are two major autoimmune complications from an infection with Streptococcus pyogenes: rheumatic fever and glomerulonephritis. Antibiotics have
3 been shown to decrease the risk of rheumatic fever only when taken In a timely fashion during a case of s. pyogenes pharyngitis. The most Important
consequence of rheumatic fever is rheumatic heart disease, which is the most frequent cause of mitral stenosis (over 99% of cases) and can cause atria l
4 fibrillation. The mechanism of rheumatic heart disease is believed to Involve molecular mimicry; that is, the humoral and cellular immune responses to
group A streptococca l antigens cross-react with host heart antigens, causing subsequent mitral valve disease.
5
Streptoco .cus pyogenes Molecular mimicry Rheumatic fever Atrial fobr' lation Pha "Yngitis Mitral insufficiency Mitral valve Glomerulonephritis Streptococcus
6 Mit a val e ste• osis Stenosis Autoimmunity Antigen Antibiotics Auto1mmune disease Cardiovascular disease Infection Fever Rheumatism
7 A i s not correct. 5 % chose this.
8 Necrotizing fasciltis is a skin infection that spreads deeply into muscle. It is commonly caused by infection of broken skin with aggressive strains of
Streptococcus pyogenes (certain strains proliferate more freely through tissue due to the presence of M proteins, which block phagocytosis),
9 Staphylococcus aureus, gram-negative enterics, and/ or Clostridium species. It Is not related to streptococcal pharyngitis.
10 Streptoco :cal pharyngitis Necrotizing fasciitis Streptococcus pyogenes Phagocytosis Staphylococcus aureus Gram-negative bacteria Pharyngitis Streptococcus
Staphylococcus Necrosis Skin infection Infection Clostridium Muscle
11
B is not co rrect . 23% c hose this.
12
Glomerulonephritis is a potential autoimmune complication of Streptococcus pyogenes pharyngitis, but the risk for this complication has not been shown
13 to decrease with the use of antibiotics. The exact pathology of postinfectlous glomerulonephritis remains unclear, but it is an inflammatory process In the
kidneys believed to be a type III hypersensitivity rea ction . Immune complexes become lodged in the glomerular basem ent m embrane. Complement
14 activation leads to destruction of the basem ent m embrane. It has also been proposed that specific antigens from certain nephrotoxic streptococcal
Infections have a high affinity for basem ent m embrane proteins, giving rise to particularly severe, long -lasting antibody response .
15
Streptococcus pyogenes Type III hypersensitivity Glomerular basement membrane Hypersensitivity Basement membrane Glomerulonephritis Pharyngitis
16 Nephrotoxicity Antibody Autoimmune disease Autoimmunity Antigen Immune complex Streptococcus Glomerulus Antibiotics Kidney Protein Pathology
17 Inflammation

18 C i s n ot co rrect . 70fo ch ose this .


• 19 Impetigo is a vesicular and crusty skin infection that is most commonly caused by Streptococcus pyogenes and/or Staphylococcus aureus. It Is not related
to streptococcal pharyngitis.
• 20 Streptococcal pharyngitis Impetigo Staphylococcus aureus Pharyngitis Streptococcus Staphylococcus Skin infection Vesicle (biology and chemistry)
• 21 E i s not correct. 6 % chose t his•

a
Lock
s
Suspend
8
End Block
Item: 18of48 ~. , . M k <:] t> al ~· ~
QIO: 2144 .l. ar Previous Next lab 'Vfl1ues Notes Calculator

1 Inflammation

2 c is not correct. 7 % chose this .


Impetigo is a vesicula r a nd crusty skin infection tha t is most commonly ca used by Streptococcus pyogenes a nd/or Staphylococcus aureus. It is not re la ted
3 to streptococca l pha ryngitis.
4 Streptococcal pharyngitis Impetigo Staphylococcus aureus Pharyngitis Streptococcus Staphylococcus Skin infection Vesicle (biology and chemistry)

5 E is not correct. 6% chose this .


Toxic shock syndrome is a multisyste m disea se thought to be ca used by exotoxins re leased by certa in stra ins of Streptococcus pyogenes a nd
6 Staphylococcus aureus. These proteins a ct a s supera ntige ns, a ble to bind MHC II with T-cell receptors in a nonspecific m a nne r. This results in polyclona l T-
cell a ctiva tion . Antibiotic use during a ca se of streptococca l pha ryngitis does not reduce the risk of toxic shock syndrome.
7
Streptococcal pharyngitis Streptococcus pyogenes Toxic shock syndrome Staphylococcus aureus Pharyngitis Antibiotics MHC class II Superantigen Streptococcus
8 Exotoxin Staphylococcus T cell T cell receptor Systemic disease Major histocompatibility complex
9
10
Bottom Line:
11
Rheuma tic hea rt disea se is a potentia l complica tion of S. pyogenes pha ryngitis, but the a dministra tion of a ntibiotics decrea ses its like lihood.
12 Rheumatic fever Pharyngitis Streptococcus pyogenes Cardiovascular disease Antibiotics Rheumatology Rheumatism

13
14
15 lijl;fiiJI•l toryear:[ 2017 • ]
FI RST AI D FA CTS

16
17 FA17 p 300.1

18 Rheumatic fever A consequence of pharyngeal infection with J'INES (major c rite ria):
• 19 g roup A ~-hemolytic streptococc i. Late Joint (migratory polya rthritis)
• 20 sequelae include rhe umatic heart disease, 'I (carditis)
wh ich affects heart valves- m itral > aortic>> Nodules in skin (subc utaneous)
• 21

6
lock
s
Suspend
0
End Block
Item: 18 of 48 ~ 1 • Ma rk -<:J 1>- Jil ~· !:';-~
QIO: 2144 ..L Pre v ious Next Lab ~lues Notes Calcula t o r
& &
1 Immune mediated (type II hypersensitivity);
2 not a direct effect of bacteria. Ant iboclies
3 to M protein cross-react with self antigens
(molecular mimicry).
4
Treatment/prophr laxis: penicillin.
5
6
FA17 p 132.4
7
8 Streptococcus Gram $ cocci in chains · . Group i\ strep j ¥ 1\ ES (major criteria for acute rheumatic
pyogenes (group A cause: fe,er):
9
streptococci) • Pyogenic-pharyngitis, cellulitis, impetigo Joints-polyarthritis
10
("honey-crusted" lesions), er) sipclas • -carditis
11 • Toxigenic- scarlet b ·cr, toxic shock- like ~ od ules (subcutaneous)
12 syndrome, necrotizing fasciit is Err thema marginatum
13 • Immunologic- rheumatic fever, Sydenham chorea
14 glomeruloneph ritis Pharyngitis can result in rheumatic "phcvcr"
Bacitracin sensitive, ~-hem olyti c, pyrroliclonyl a ncl glomerulonephritis.
15
arylamidase (PYR) $. Hyaluron ic acid capsnlc Impetigo usually precedes glomerulonephrit·is.
16 c inh ibits phagocytosis. Antibodies to M protein Scarlet fever- blanching, sandpaper-like body
17 enhance host defenses against S (Jyogenes but rash, strawberry tongue, and circumoral
18 can gi,·e rise to rheumatic fever. pallor in the setting of group A streptococcal
. 19 ASO titer or anti-DNasc B antibodies indicate pharyngitis (erythrogenic toxin $ ).
recent S pyogenes infection.
• 20
. 21

a
Lock
s
Suspend
8
End Bl ock
Item: 19 of 48 ~ 1 • M k -<:J 1>- Jil ~· !:';-~
QIO: 10 98 ..L ar Pre v ious Next Lab~lues Notes Calcula t o r

1
2
&

An 82-year-old woman presents to the emergency department with a 3-week history of fever, weight loss, and malaise in the setting of hip and
shoulder girdle pain that is most severe in the morning. She also reports a 1-week history of headaches and left-sided j aw pain that occurs at every
lA• A] &

meal. The patient's temperature is 38.2°C ( 100.8°F), her pulse is 104/mln, and her blood pressure is 140/80 mm Hg. Laboratory studies show a
3 hemoglobin level of 11.8 g/dl, a WBC count of 11,900/mm3 , and an erythrocyte sedimentation rate of 121 mm/h. Physical examination is unremarkable
except for moderate synovitis of the ankles and wrist .
4
5
Which procedure is most likely to be diagnostic in this patient?
6
7 A. Arthrocentesis
8
B. Mesenteric angiogram
9
10 c. Temporal artery biopsy

11 o. Testing for anti-double-stranded DNA and antinuclear antibody levels

12
E. Testing for rheumatoid factor and anti-cyclic citrullinated peptide levels
13
14
15
16
17
18
. 19
• 20
. 21

a
Lock
s
Suspend
8
End Bl ock
Item: 19 of 48 ~ 1 • M k -<:J 1>- Jil ~· !:';-~
QIO: 1 0 98 ..L ar Previous Next Lab~lues Notes Calculat or
& &
1
2
3 Th e correct a n swer i s c. 6 4% chose this.
This patient has symptoms consistent wit h polymyalgia rheumatica (PMR) and giant cell arteritis (temporal arterit is) . She requires immediate steroids for
4 treatment and subsequent temporal artery biopsy t o confirm the diagnosis. PMR occurs in SO% of patients wit h tempora l arteritis and involves
5 symmetrical aching of the proximal muscl es and girdle stiffness. The elevated erythrocyte sedimentation rate indicates a generalized inflammatory
process, and addotional evidence is provid ed by t he new-onset jaw claudication and constitutional sympt oms t hat usually present in patients with
6 temporal artentos.
Polymyalg eumatica Goant-cell arteritis Erythrocyte sedimentation rate Red blood celo Claudication Biopsy Giant cell Superficial temporal artery Jaw Steroid
7
Muscle A teritis
8
A is not correct. 6% chose this.
9 Analysis of joint fluid would be neithe r diagnostic nor possible in this patient because she is only currently suffe ring from synovitis of her wrists and ankles.
10 Synovitis Synovial fluid

11 B is not correct. 3% chose this.


Mesenteric angiography is the prima ry imaging modality used to determine the presence of aneurysms a nd vessel narrowing in patients with polyarteritis
12 nodosa (PAN). PAN is a necrotizing vasculitis that typically presents with constitutional signs as well as with myalgias, arthralgias, and fatigue. This
patient's girdle stiffness and j aw claudication are not consistent with this diagnosis.
13
Polyarteritis nodosa Angiography Vasculitis Arthralgia Aneurysm Claudication Fatigue (medical) Systemic vasculitis Necrosis
14
0 is not co rrect . 100/o chose t h is.
15 Testing for anti-double -stranded DNA and ant inuclea r ant ibody levels would be appropriate to diagnose lupus, which is uncommon in patients this age and
does not Involve jaw claudication .
16
Anti-nuclear antibody Claudication Lupus erythematosus Antibody DNA Systemic lupus erythematosus
17
E is n ot co rrect . 170/o ch ose this.
18 Testing for rheumatoid factor and anti-cyclic cit rullinat ed peptide (anti-CCP) levels would be appropriat e to diagnose rheumatoid arthritis, which may
19 produce symmetrical and proximal joint sympt oms. However, rheumatoid arthrit is does not cause j aw claudication and does not usually present for the
first t ime In someone this elderly.
• 20 Rheumatood factor Rheumatoid arthritis Arthritis Claudication Peptide Anatomical term~ of location

. 21

a
Lock
s
Suspend
8
End Block
Item: 19 of 48 ~- I • M k <:] t> al ~· ~
QIO: 1098 .l. ar Previous Next lab 'Vfl1ues Notes Calculator

1 E is not correct. 17% chose this.


2 Testing for rheumatoid factor and anti-cyclic citrullinat ed peptide (anti-CCP) levels would be appropriat e to diagnose rheumatoid arthritis, which m ay
produce symmetrical and proximal joint symptoms. However; rheumatoid arthritis does not cause j aw claudication and does not usually present for the
3 first time in someone this elderly.
Rheumatoid factor Rheumatoid arthritis Arthritis Claudication Peptide Anatomical terms of location
4
5
6 Bottom Line:
7 Always think about t emporal arteritis in an older woman with new -onset hea daches and j aw pain. Associat ed symptoms that support this diagnosis are
fever; weight loss, scalp t enderness, and vision loss. The erythrocyte sedimentation rat e is eleva t ed in about 80% of cases. Diagnosis is m ade with
8 t emporal artery biopsy. Many individuals m ay also have polymyalgia rheumatica.
Polymyalgia rheumatica Giant-cell arteritis Erythrocyte sedimentation rate Red blood cell Biopsy Superficial temporal artery Arteritis Weight loss Visual impairment
9 Jaw Scalp Fever

10
11

12 lijl;fiiJI•l toryear:[ 2017 • ]


FI RST AID FAC T S
13
14 FA17 p 302.1
15 Vasculitides
16 EPIDEMIOLOGY/PRESENTATION PATHOLOGY/lABS
17 Large-vessel vasculitis
18 Giant cell (temporal) Usually elderly females. Most commonly affects branches of carotid
19 arteritis Unilateral headache (temporal artery}, jaw artery.
0 20 claudication. Focal granulomatous inAammation rJ.
May lead to irreversible blindness due to t ESR.
0
21
n nhth"ll m if"' "l r b >n 1 1'\l"'t'lu cin n

6
lock
s
Suspend
0
End Block
2

3
c
4
5
FA17 p 444.2
6
Polymyalgia rheumatica
7
SYMPTOMS Pain and stiffness in shoulders and hips, often '' ith fever, malaise, weight loss. Does not cause
8
muscular '' eakness. lore common in women > 50 )Cars old; associated with giant cell (temporal)
9 arteritis.
10 FINDINGS t ESR, t C RP, normal CK.
11 TREATMENT Rapid response to low-dose corticosteroids.
12
13
FA17p111 .1
14
Autoantibodies AUTOANTIBODY ASSOCIATED DISORDER
15
Anti-ACh receptor M)'asthen ia gravis
16
Anti-glomerular basement membrane Goodpasture S)'ndrome
17
Anti-i}z gi)'COprotein Antiphospholipid syndrome
18
19
Anticardiolipin, lupus anticoagulant SLE, antiphospholipid syndrome
• 20 Anticentromere Limited scleroderma (C REST syndrome)
• 21 Anti-desmoglein (anti-desmosome) Pemphigus vulgaris

a
Lock
s
Suspend
8
End Block
Item: 20 of 48 ~ 1 • M k -<:J 1>- Jil ~· !:';-~
QIO: 1411 ..L ar Pre v ious Next Lab~lues Not es Calcula t o r
& &
1
A 48-year-old obese man presents to his primary care physician with complaints of lower leg pain that occurs after he walks a few city blocks and Is
2 relieved with rest. He has no other complaints. His blood pressure is 165/85 mm Hg, his pulse is 83/min, and his respiratory rate is 18/min. After
further questioning he admits to smoking two packs of cigarettes per day for 20 years.
3
4 Which of the following types of vessels is most likely involved in the pathologic process surrounding this patient's symptoms upon walking?
5
6 A. Arteries
7
B. Arterioles
8
c. Capillaries
9
10 o. Veins

11 E. Venules
12
13
14
15
16
17
18
19
• 20
. 21

a
Lock
s
Suspend
8
End Bl ock
Item: 20 of 48 ~ 1 • M k -<:J 1>- Jil ~· !:';-~
QIO: 1411 ..L ar Prev ious Next Lab~lues Not es Calculat o r

& &
1
Th e co rrect an sw er i s A. 420/o ch ose this.
2
This patient is describing symptoms of intermittent claudication. Combined with his long history of smoking, he likely has significant peripheral artery
3 disease (PAD). Peripheral atherosclerosis t argets areas of high turbulence, such as points at which arteries branch. The most common sites for PAD are
the abdominal aorta, and the iliac, femora l, popliteal, tibia l, and peroneal arteries. Calf pain in this patient is likely caused by narrowing of the popliteal
4 artery. Medical therapy with antiplatelet drugs such as aspirin has had moderate success, with surgical revascularization reserved for severe cases. Risk
5 factors for atherosclerosis include smoking, hypertension, diabetes mellitus, hyperlipidemia, and a positive family history.
Intel mitte t claudication Peripheral artery disease Atherosclerosis Diabetes me itus AUdom1nal aorta Aspirin Hyperlipidemia Claudication Aorta Popl!teal artery
6 Hypertens'o Re •ascularization Artery Anti platelet drug Tobacco smoking Femoral artery
7 B i s not correct. 3 0 % chose this.
8 Arterioles help provide the dynamic regulation of blood flow through the capillary beds. Although they are the prindpal site of resistance in blood flow,
they are not the site of the significant turbulence necessary to predispose to atherosclerotic formation. They, along with small muscular arteries, are the
9 major vessels affected by hypertension.
Atheroscle osis Hypertension Capillary Arteriole Artery Turbulence Blood flow
10
11 C i s n ot co rrect. 7 % chose this.
Capillaries are the smallest vessels and are the site of exchange of gases and solutes between the blood and extracellular fluid. They are not the site of
12 atherosclerosis but they play a major role in the pathophysiology of edema.
Atherosclerosis Extracellular fluid Capillary Edema Pathophysiology Solution Extracellylar
13
14 0 is not co rrect. 150/o chose t his.
Veins are the vessels that return blood back to the heart and are not typically affected by atherosclerosis.
15 Atherosclerosis
16 E i s not co rrect. 60fo ch ose this.
17 venules are the first vessels to collect blood from the capillary beds and are not affected by atherosclerosis.
Atherosclerosis Venule Capillary
18
19
20 Bottom Lin e:

. 21 Intermittent claudication is a painful ischemic condition caused by periphera l atherosclerosis of the arteries.
• Intel mittent claudication Atherosclerosis Claudication Ischemia Arterv

a
Lock
s
Suspend
8
End Block
Item: 20 of 48 ~. I • M k <:] t> al ~· ~
QIO: 1411 .l. ar Previous Next lab 'Vfl1ues Notes Calculator
Venules are the first vessels to collect blood from the capillary beds and are not affected by atherosclerosis.
1
Atherosclerosis Venule Capillary
2

3
Bottom Line:
4
Intermittent claudication is a painful ischemic condition caused by periphera l atherosclerosis of the arteries.
5 Intermittent claudication Atherosclerosis Claudication Ischemia Artery

6
7

8 lijl;fiiJI•l toryear:[2017 • ]
FIRST AID FACTS
9
10 FA17 p292.1
11
Atherosclerosis Very common. Disease of clastic arteries and large- and med ium-sized muscular arteries; a form of
12 arteriosclerosis caused by buildup of cholesterol plaques.
13 LOCATION Abdominal aorta > coronary artery > popliteal artery> carotid artery f,l.
14
15
16
17
18
19
20
RISKFACTORS Modifiable: smoking, hypertension, hyperlipidemia (t LDL), diabetes.
• 21
nnmnr1ih~hiP• ~OP t:.PV {f in ll1PO !1 11rl nn~tn'\Pllf"\T'\~11<:::3) \ V f'lfl'lPn ) r~n1i l v hfc:: tnrv

6
lock
s
Suspend
0
End Block
Item: 20 of 48 ~ 1 • M k -<:J 1>- Jil ~· !:';-~
QIO: 1411 ..L ar Pre v ious Next Lab~lues Not es Calcula t o r
& &
1

2
COMPLICATIONS Aneurysms, ischemia, infarcts, peripheral vascular disease, thrombus, emboli.
3
4 FA17 p 291.2

5 Arteriosclerosis Hardening of arteries, with arteria l wall thickening and loss of elasticity.
6 Arteriolosclerosis Common. .\ ffects small arteries and arterioles. T" o types: hyaline (thickening of \esse I "ails in
7
essential hypertension or diabetes mellitus ) and hyperplastic ("onion skinning" in severe
8
rn
hypertension with proliferation of smooth muscle cells).

9
Monckeberg sclerosis Uncommon. Affects medium-sized arteries. Calcification of internal elastic lamina and media of
(medial calcific arteries ..... \'asctdar stiffening without obstruction. ''Pipestem" appearance on x-rar . Does not
10
sclerosis) obstruct blood Ao\\; intima not in,olved.
11
12 = -=-~
~ -~
13
14
15
16
17
18
19
20
. 21

a
Lock
s
Suspend
8
End Bl ock
2
3
4
5
6
7
8 FA17 p 290.2

9
Hypertension Defined as persistent systolic BP ~ HOmm Hg and/or diastolic BP ~ 90 mm Hg
10 RISK FACTORS t age, obesity, diabetes, physical inactivity, c:~.cess salt intake, excess alcohol intake, fa mily history;
African American > Caucasian > sian.
11
12
FEATURES 90% of hypertension is !0 (essent ial) and related to t CO or t TPR. Remaining 10% mostly zoto
renal/renovascular disease (eg, fibromuscular dysplasia [which has characteristic "string of beads"
13
appearance of renal artery · ], atherosclerosis) and ! 0 hyperaldosteronism.
14 Hypertensive u rgency -severe (~ 1 80/~ 120 mm I lg) hypertension without acute end-organ
15 damage.
16 Hypertensive emergency-severe hypertension with evidence of acute end-organ damage (eg,
17 encephalopathy, st·roke, retinal hemorrhages and exudates, papilledema, MJ, HF, aortic dissection,
kidney injury, microangiopathic hemolytic anemia, eclampsia).
18
19 PREDISPOSES TO CAD, LVH, HF, atrial fibrillation; aortic dissection, aortic aneurysm; stroke; chronic kidney disease
20 (hypertensi,·e nephropathy); retinopathy.
. 21

a
Lock
s
Suspend
8
End Block
Item: 21 of 48 ~ 1 • M k -<:J 1>- Jil ~· !:';-~
QIO: 5 0 86 ..L ar Pre v ious Next Lab~lues Notes Calcula t o r
& &
1
A tall 27-year-old man with pectus excava tum presents to the emergency department with intense chest pain. The patient reports that the pain
2 began approximately 1 hour ago and radiat es to his back. X-ray of the chest shows mediastinal widening, and the patient is rushed to the operating
room .
3
4 What pathology of the great vessels predisposed t he patient to this emergency department presentation?
5
6 A. Atherosclerosis
7
B. Cystic medial necrosis
8
c. Disruption of the vasa vasorum
9
10 D. Granulomatous vasculitis

11 E. Telangiectasia
12
13
14
15
16
17
18
19
20
. 21

a
Lock
s
Suspend
8
End Bl ock
Item: 21 of 48 ~ 1 • M k -<:J 1>- Jil ~· !:';-~
QIO: 5 0 86 ..L ar Prev ious Next Lab~lues Notes Calculat o r

& &
1
Th e co rrect an sw er i s B. 560/o ch ose this.
2
This patient is experiencing an aortic dissection. Major risk factors for aortic dissection include hypertension
3 or cystic medial necrosis (shown in this image) associated with Marfan syndrome. This syndrome is often
associated with both aortic dissections and ascending aortic aneurysms, although in this case the patient is
4 likely experiencing a dissection. Marfan syndrome is an autosomal dominant disease caused by a mutation
5 In the flbrlllin gene. Patients with Marfan syndrome display characteristic skeletal abnormalities such as
pectus excavatum, hyperextensible joints, tall stature, and long extremities and dig1ts. Cardiovascular
6 problems Include aortic aneurysms and mitral valve prolapse.
Marfan sy od orne Pectus e cavatum Aortic dissection Mitral valve prolapse Famo.oal ~. oo ac•c aortic aneurysm
7
Gene Dominance (genetics) Fibrillin Mitral va lve Hypertension Aneurysm Mutation Autosome Prolapse
8 Circulatory system Necrosis
9
10
11
12
13
14
15
16
17
18
19 Image courtesy of Dr. Michael Bonert

20 A i s not cor r ect. 8 0/o ch ose this.


21 Atherosclerosis is a risk factor for aneurysm of the abdominal aorta, rather than the thoracic aorta. Patients with aneurysms from atherosclerosis tend to

a
Lock
s
Suspend
8
End Block
I mage courtesy of Or. Michael Bonert
3
4 A is no t co rrect. 8% cho se this.
5 Atherosclerosis is a risk factor for aneurysm of the abdominal aorta, rather than the thora cic aorta. Patients with aneurysms from atherosclerosis t end to
have associat ed risk factors, such as smoking, hypertension, diabet es, and hyperlipidemia, and t end to be older.
6 Atherosclerosis Abdominal aorta Aorta Thoracic aorta Hyperlipidemia Aneurysm Hypertension Diabetes mellitus Risk factor Thoracic vertebrae Thorax

7 c is no t co rrect. 25 % cho se this.


Disruption of the vasa va sorum occurs in t ertiary syphilis, lea ding to dilation of the aorta through the thickening and weakening of the vessel wall. As a
8
result, aneurysms of the ascending aorta or aortic arch can develop, as well as aortic va lve incompet ence. The physical exam findings of t all stature and
9 pectus excava tum in the vignette are more closely associat ed with Marfan syndrome than with t ertiary syphilis.
Marfan syndrome Vasa vasorum Syphilis Ascending aorta Aorta Aortic arch Aortic valve Pectus excavatum Aneurysm Vasodilation Physical examination
10
D is no t co rrect. 6 % cho se this.
11
Takayasu arteritis and giant cell arteritis also can cause aortic arch and descending thoracic aneurysms. Histologically, granulomatous changes may be
12 seen. These two disorders are more common in females and are associat ed with signs of inflammation (elevat ed erythrocyte sedimentation rat e). They
are not associat ed with pectus excava tum .
13 Takayasu's arteritis Giant-cell arteritis Erythrocyte sedimentation rate Red blood cell Aneurysm Histology Granuloma Inflammation Thoracic vertebrae Giant cell
14 Arteritis

15 E is no t co rrect. 5 % cho se this.


16 Telangiectasias are malformations that occur in small blood vessels. They do not cause aneurysms of the grea t vessels. Telangiectasias are often seen in
Osler -Weber -Rendu syndrome and at axia -t elangiectasia.
17 Ataxia telangiectasia Hereditary hemorrhagic telangiectasia Telangiectasia Aneurysm Blood vessel

18
19 Bo tto m Line:
20 Marfan syndrome is an autosomal dominant disorder in which patients are predisposed to aortic aneurysms. Other fea tures of this disorder include
pectus excava tum, ectopia lentis, mitral valve prolapse, and hyperextensible joints.
21 Marfan syndrome Pectus excavatum Ectopia lentis Mitral valve prolapse Dominance (qenetics) Mitral valve Autosome Prolapse Aneurysm

6
lock
s
Suspend
0
End Block
Item: 21 of 48 ~. I • M k <:] t> al ~· ~
QIO: 5086 .l. ar Previous Next lab 'Vfl1 ues Notes Calculator

• Ataxia telangiectasia Hereditary hemorrhagic telangiectasia Telangiectasia Aneurysm Blood vessel


1
2
3 Botto m Line:
4 Marfan syndrome is an autosomal dominant disorder in which patients are predisposed to aortic aneurysms. Other features of this disorder include
pectus excavatum, ectopia lentis, mitral valve prolapse, and hyperextensible joints.
5 Marfan syndrome Pectus excavatum Ectopia lentis Mitral valve prolapse Dominance (genetics) Mitral valve Autosome Prolapse Aneurysm

6
7
8 lijj ;fi IJ l•l fo r yea r:l 20 17 ..
FIRST AID FACTS
9
10 FA17p56.1
11 Autosomal dominant Achondroplasia, autosomal dominant polycystic kidney disease, fam ilial adenomatous polyposis,
12 diseases familial hypercholesterolemia, hereditary hemorrhagic telangiectasia, hereditary spherocytosis,
13 Huntington disease, Li-Fraumeni syndrome, Marfan syndrome, multiple endocrine neoplasias,
14 neurofibromatosis type I (,·on Recklinghausen disease}, neurofibromatosis type 2, tuberous
sclerosis, von 1-1 ippei-Lindau disease.
15
16
FA11 p293.1
17
Aortic dissection Longitudinal intimal tear forming a false lumen (arrows in fit show Aap extendi ng into ascending
18
aorta, Stanford type A dissection). Associated with hypertension, bicuspid aortic valve, inherited
19
connective tissue disorders (eg, Marfan syndrome). Can present with teari ng, sudden-onset chest
20 pain, radiating to the back+/- markedly unequal BP in arms. CXR shows mediastinal widening.
21 Can result in organ ischemia, aortic rupture, death. Two types:

6
lock
s
Suspend
0
End Block
Item: 22 of 48 ~ 1 • M k -<:J 1>- Jil ~· !:';-~
QIO: 3561 ..L ar Pre v ious Next Lab~lues Not es Calcula t o r
A A

2 A 27-year-old pregnant woman from Guat em ala dies suddenly while lifting a heavy object in her home. She has no cardiac history, but her mother
tells the coroner that her daughter developed a murmur 3 or 4 years ago. A histologic specimen of her heart at autopsy is shown in the image.
3
4
1
5
6
7
8
9
10

11

12
13
14
15
What Is the most likely cause of the pathology observed in this patient?
16
17 A . Atherosclerosis
18
B. Bicuspid aorta
19
C. Hypertrophic obstructive cardiomyopathy
20
21 D. Infectious endocarditis

• 22 •
a
Lock
s
Suspend
8
End Bl ock
2
3
4
5
6
7
8
9
10
11
12
13 What Is the most likely cause of the pathology observed in this patient?
14
15 A . Atherosclerosis

16
B. Bicuspid aorta
17
c. Hypertrophic obstructive cardiomyopathy
18
19 D. Infectious endocarditis

20 E. Rheumatic heart disease


21
. 22 •
a
Lock
s
Suspend
8
End Block
Item: 22 of 48 ~ 1 • M k -<:J 1>- Jil ~· !:';-~
QIO: 3561 ..L ar Previous Next Lab~lues Not es Calculat or
A A

3 Th e co rrect a n swer i s E. 600/o chose this.


4 Although rheumatic fever has become a rarity in the United States, It Is still quite common in
underdeveloped countries. One of the most devastating complications of rheumatic fever is rheumatic
5 heart disease, which most often manifests as an autoimmune insult to the mitral valve that leads to mitral
stenosis and a heart murmur. This can often go unnoticed until excessive strain Is put on the cardiovascular
6 system (such as in pregnancy), which leads to heart failure. The photomicrograph shows a classic Aschoff
7 body, within the orcle, containing Anitschkow cells (large macrophages with wavy nuclei, at 1 o'clock within
the circle).
8 Rhe• mati e •er Heart murmur Mitral valve stenosis Mitral valve Macrophage c.. c... dlooy system Stenosis

9 Autoimmune disease Cardiovascular disease Heart failure Autoimmunity Developing country Rheumatology
Rheumatism Fever Pregnancy
10
11
Image courtesy of Ed Uthman, MD
12
A is not co rrect. 30/o chose this.
13
Coronary plaques, when they are large enough, can cause myocardial Infarction, the sequelae of which can lead to sudden cardiac death. However,
14 considering the age of this patient and the pathology shown, atherosclerosis Is an unlikely cause.
Atherosclerosis Myocardial infarction Sudden cardiac death Coronary artery disease Pathology Sequela
15
8 is not co rrect. 40/o chose this.
16
A bicuspid aortic valve is a congenital anomaly that can go undetected throughout a patient's lifetime. These lesions m ay be discovered later In life as the
17 valve calcifies, and they rarely cause sudden death . Additionally, the pathology Image points to rheumatic heart disease.
Rheumatic fever Bicuspid aortic valve Aortic valve Congenital disorder Mitral valve Cardiovascular disease Pathology
18
C is no t co rrect. 190/o ch ose this.
19
Hypertrophic obstructive cardiomyopathy is a cause of sudden death that often presents in young athletes. Biopsy shows myocytes in disarray. Treatment
20 Includes 13-blockers, and surgical myotomy in refractory cases.
Hypertrophic cardiomyopathy Hypertrophy cardiomyopathy Myocyte Biopsy Ventricular hypertrophy
21
0 i s not correct. 140/o chose this.
22
Infectious endocarditis is a less likely cause of sudden death, although It can cause the development of valve pathology. More common among Intravenous
• 23 • druq abusers, lesions often appear on the riqht side, placinQ the patient at hlqher rrsk of pulmonary embolism. The patient does not fall into this risk

a
Lock
s
Suspend
8
End Bl ock
Item: 22 of 48 ~. I • M k <:] t> al ~· ~
.l. ar Previous lab 'Vfl1ues Notes Calculator
.
QIO: 3561 Next
'
3 Hypertrophic obstructive cardiomyopathy is a cause of sudden dea th that often presents in young athlet es. Biopsy shows myocytes in disarray. Trea tment
4 includes 13-blockers, and surgical myotomy in refractory cases.
Hypertrophic cardiomyopathy Hypertrophy Cardiomyopathy Myocyte Biopsy Ventricular hypertrophy
5
D is not correct. 14% chose this.
6 Infectious endocarditis is a less likely cause of sudden dea th, although it can cause the development of va lve pathology. More common among intravenous
7 drug abusers, lesions often appear on the right side, placing the patient at higher risk of pulmonary embolism. The patient does not fall into this risk
cat egory, however; and microscopic findings in this case point to a diagnosis of rheumatic heart disea se.
8 Pulmonary embolism Rheumatic fever Endocarditis Infective endocarditis Embolism Intravenous therapy Drug injection Cardiovascular disease Pathology

9 Rheumatism

10
11 Bottom Line:

12 Rheumatic heart disea se is rare in the United Stat es but is still common in developing nations. Its pathologic fea ture is the Aschoff body, a focal area
of mixed mononuclear inflammatory cells with plump, activa t ed macrophages (Anitschkow cells) which occasionally coalesce to form multinuclea t ed
13 giant cells. Aschoff bodies may also be associat ed with focal area s of fibrinoid necrosis.
Aschoff body Rheumatic fever Fibrinoid necrosis Macrophage Cardiovascular disease Anitschkow cell Necrosis Giant cell Rheumatism United States Pathology
14
Developing country
15
16
17 I iii I;fi 1!1 I•J for year:l 2017 ..
FI RST AID FA CTS
18
19
FA17 p 300.1
20
Rheumatic fever A consequence of pharr ngeal in fection with J'INES (major c rite ria):
21
group A ~-hemolytic streptococci. Late Joint (migratory polya rthri tis)
22 sequelae include rhe umatic heart disease, '1 (card itis)
• 23 wh ich affects hea rt valves-m itral > aortic>> Nodules in skin (subc uta neous)

6
lock
s
Suspend
0
End Block
Item: 23 of 48 ~ 1 • M k -<:J 1>- Jil ~· !:';-~
QIO: 1428 ..L ar Pre v ious Next Lab~lues Notes Calcula t o r

3
A

A 72-year-old woman has a 1-month history of left-sided jaw pain when chewing food, headache, fever, and fatigue. She has never experienced this
problem before and has no major medical history except for obesity, high cholesterol controlled with a statin, and a hysterectomy more than 20 years
IA•A] A

4
prior. Physical examination reveals diminished pulses throughout the body and Ischemia on funduscopic examination . Laboratory studies reveal an
5 elevated erythrocyte sedimentation rate . Her physician suspects a vasculltic syndrome.

6
What likely would be seen on a biopsy of her affected arteries?
7
8
A. Cyst1c medial necrosis
9
B. Eosinophilic infiltration
10
11 c. Granulomatous inflammation

12 o. Mycotic aneurysms
13
E. Necrotizing inflammation
14
15
16
17
18
19
20
21
22
. 23 •
a
Lock
s
Suspend
8
End Bl ock
Item: 23 of 48 ~ 1 • M k -<:J 1>- Jil ~· !:';-~
QIO: 1428 ..L ar Previous Next Lab~lues Notes Calculat or
A A

3
4 The correct answ er is c. 700/o chose this.
5 This Individual likely has giant cell (temporal) arteritis (GCA), the most common systemic vasculitis in adults. GCA, which affects large- to medium-sized
arteries, typically presents in people older than so years of age and Is more common in women. Patients commonly present with constitutional symptoms
6 (anorexia, fatigue, weight loss), unilateral temporal or occipital headache with overlying sca lp tenderness, jaw claudication, and impaired vision, and their
erythrocyte sedimentation rate is usually elevated, in contrast to the types of Inflammation listed. The superficial temporal artery is the artery most
7 commonly affected 1n patients with GCA, and it is affected in this patient. On biopsy, affected arteries are characterized by nodular thickening that
reduces the size of the lumen, granulomatous inflammation with mononuclear and giant cells, and fragmentation of the internal elastic membrane (as
8
shown In the 1mages). GCA is treated with high-dose corticosteroids to reduce Inflammation rapidly and prevent permanent blindness.
9 Erythrocyte sedimentation rate Red blood cell Vasculitis Corticosterood Superficia tempoo ao artery Granuloma Inflammation Biopsy Claudication Headache

10 Anorexia (symptom) Fatigue (medical) Giant cell Arteritis Visual impairment Weight loss Lumen (anatomy) Artery Scalp

11
H&E stain Elastin stain
12
13
14
15
Thickening
16 ...- ofthe
artery wall
17
18
'
19
20 Breakdown of elastin around 11\e artery
21 Images courtesy of Y. Chu, MD

22 A i s not correct. 6 0/o chose this.


23 Cystic medial necros1s is a feature of blood vessels in Marfan syndrome; In these patients, a defect in the fibrillin protein results in weak vessel walls,

a
Lock
s
Suspend
8
End Block
Item: 23 of 48 ~. I • M k <:] t> al ~· ~
QIO: 1428 .l. ar Previous Next lab 'Vfl1ues Notes Calculator

3 l \
4 Breakdown of elastin around tlie artery
I m ages courtesy of Y. Chu, MD
5
6 A is not co rrect. 6 % chose this .
Cystic medial necrosis is a fea ture of blood vessels in Marfan syndrome; in these patients, a defect in the fibrillin protein results in weak vessel walls,
7
predisposing them to aneurysmal dilation . Patients with Marfan disea se t end to be t all and thin, with hyperdistensible joints, mitral valve prolapse, and
8 ocular abnormalities.
Marian syndrome Aneurysm Mitral valve prolapse Familial thoracic aortic aneurysm Mitral valve Fibrillin Prolapse Necrosis Protein Blood vessel Vasodilation
9
B is not co rrect. 7 % chose this .
10
Granulomatous inflammation rich in eosinophils is observed in vessels of the respiratory tract in patients with Churg-Strauss syndrome; frequently such
11 patients have comorbid atopic disea ses such as asthma.
Churg-Strauss syndrome Asthma Eosinophil Atopy Granuloma Respiratory tract Inflammation Comorbidity Atopic dermatitis
12
D is not co rrect. 4 % chose this .
13 Mycotic aneurysms are area s of vessel wall dilation resulting from localized infection of the vessel wall; the infection results in suppurative inflammation
14 and local destruction of the intima and media, weakening the vessel wall and potentiating aneurysmal dilation .
Aneurysm Inflammation Tunica intima Pus
15
E is not co rrect. 1 3 % chose this .
16
Area s of inflammation and necrosis are observed in several vasculitic syndromes, including polyarteritis nodosa, granulomatosis with polya ngiitis, and
17 microscopic polyangiitis. The distribution of affected arteries and the presence of specific antibodies in a patient's serum are critical in distinguishing these
disea se processes.
18 Granulomatosis with polyangiitis Polyarteritis nodosa Microscopic polyangiitis Vasculitis Antibody Necrosis Inflammation Artery Blood plasma

19
20 Botto m Line:
21 Giant cell (temporal) arteritis (GCA) is a chronic va sculitis of medium to large arteries most common in people > SO years of age. The most commonly
affected vessel is the superficial t emporal artery, from which a biopsy can be obtained for diagnosis.
22
Vasculitis Biopsy Temporal artery Superficial temporal artery Giant cell Artery Arteritis
23 •

6
lock
s
Suspend
0
End Block
3 Vasculitis Biopsy Temporal artery Superficial temporal artery Giant cell Artery Arteritis

4
5
6 lijj ;fi IJ l•l for year:l 2017 ..
FIRST AID FACTS
7

8 FA17 p 302.1

9 Vasculitides
EPIDEMIOLOGY/PRESENTATION PATHOLOGY/LABS
10
11
Large-vessel vasculitis

12 Giant cell (t empo ral) Usually elderly females. Most commonly affects branches of carotid
arteritis Unilateral headache (temporal artery}, jaw artery.
13
claudication. Focal granulomatous inAammation rJ.
14
lay lead to irreversible blindness due to t ESR.
15 ophthalmic artery occlusion. Treat with high-dose corticosteroids prior to
16 Associated with polymyalgia rheumatica. temporal artery biopsy to prevent blindness.
17 Takayasu arteritis Usually Asian fema les< 40 years old. Granulomatous thickening and narrowing of
18 "Pulseless disease" (weak upper extremity aortic arch and proximal great vessels rn.
19 pulses}, fever, night sweats, arthritis, myalgias, t ESR.
skin nodules, ocular disturbances. Treat with corticosteroids.
20
Medium-vessel vasculitis
21
22
Po lyarteritis no dosa Usually middle-aged men. Typically involves renal and visceral vessels, not
Hepatitis B seropositivity in 30% of patients. pulmonary arteries.
23 C'- ··- - ···-'-1.• •--- ___ ,_,__ 1.---•--•-- ,...r_........... --.. -.... 1 : .... a ...................... ..: ........ .... c .. 1... ......... _ ........ : .... 1 .. ..... 11

6
lock
s
Suspend
0
End Block
Item: 24 of 48 ~ 1 • M k -<:J 1>- Jil ~· !:';-~
QIO: 1412 ..L ar Pre v ious Next Lab~lues Not es Calcula t o r
A A

4 A 54-year-old African-American man dies in a motor vehicle crash and an autopsy is requested . His family states that he avoided routine care until
recently. He was diagnosed with diabetes several months ago but did not adhere to his medication regimen . They add that a skin-lesion biopsy
5
several weeks ago was positive for melanoma and was subsequently removed by Mohs surgery. During autopsy, the man's heart is examined and
6 noted to be abnormal.

7
8
9
10
11
12
13
14
15
16
17
18
19
What process would likely cause the findings seen in the gross transverse section shown in this image?
20
21 A. Aortic valve insufficiency
22
B. Atrial septal defect
23
• 24 c. Myocardial mfarction

a
Lock
s
Suspend
8
End Bl ock
15 What process would likely cause the findings seen in the gross transverse section shown in this image?
16
17 A . Aortic valve insufficiency

18 8. Atrial septal defect


19
c. Myocardial infarction
20
21 o. Pulmonary hypertension

22 E. Systemic hypertension
23
• 24

a
Lock
s
Suspend
8
End Block
Item: 24 of 48 ~. I • M k <:] t> al ~· ~
QIO: 1412 .l. ar Previous Next lab 'Vfl1ues Notes Calculator

4 The co rrect a nswer is E. 71% cho se this.


The image is a transverse section of the heart displaying concentric hypertrophy of the left ventricle. The myocardium hypertrophies in response to the
5
increa sed pressure workload caused by the increa se in systemic vascular resistance, as is seen in chronic hypertension. The concentric hypertrophy allows
6 the heart to continue perfusing the body's tissues against the heightened resistance. With continued wall thickening, however; the left ventricular
hypertrophy can lea d to diastolic dysfunction; because the ventricular wall is less compliant, it is unable to fully relax and therefore allows less blood to
7 enter the chamber during diastole. The end result is a smaller stroke volume per bea t . Histologically, the hypertrophic myocytes have thicker fibers and
enlarged, hyperchromic, rectangular nuclei. Similar findings are observed in the hearts of patients with aortic stenosis.
8 left ventricular hypertrophy Diastole Hypertension Cardiac muscle Stroke volume Ventricle (heart) Aortic stenosis Diastolic heart failure Hypertrophy Stenosis
9 Myocyte Vascular resistance Ventricular hypertrophy Concentric hypertrophy Stroke Heart Blood vessel Histology Perfusion
10 A is no t co rrect. 10 % cho se this.
11 Aortic valve insufficiency can lea d to severe volume overload, as a portion of ea ch stroke volume ej ected flows back into the left ventricle. Severe volume
overload usually lea ds to ventricular dilation and, as a result, decrea sed contractility. Massive ventricular hypertrophy, as is seen in the image, is a
12 response to the pressure overload that is experienced in conditions such as systemic hypertension and aortic stenosis.
Aortic insufficiency Aortic stenosis Stroke volume Aortic valve Hypertension Ventricle (heart) Ventricular hypertrophy Volume overload Hypertrophy Stenosis
13
Contractility Pressure overload Stroke Vasodilation
14
B is no t co rrect. 3 % cho se this.
15
An atrial septal defect (ASD ) results in a dilat ed right ventricle. In an ASD, blood preferentially flows from the left atrium into the right atrium during
16 diastole, resulting in right ventricular volume overload. Such severe, persistent volume overload usually lea ds to ventricular dilation and, ultimat ely,
decrea sed contractility.
17 Atrial septal defect Atrium (heart) Diastole Ventricle (heart) Contractility Volume overload Ventricular system
18 c is no t co rrect. 6 % cho se this.
19 In the first 24 hours after a myocardial infarction, a gross examination of myocardial tissue would reveal area s of pallor; indicating regions of persistent
ischemia. Within 24 - 72 hours, the gross appearance of the heart would become mottled, with patchy area s of pallor in regions with persistent ischemia
20 and other area s of dark-red color where blood flow has been reestablished (hyperemia).
Myocardial infarction Ischemia Hyperaemia Pallor Infarction Blood flow
21
22 D is no t co rrect. 10 % cho se this.
Pulmonary hypertension would lea d to right ventricular hypertrophy, not left ventricular hypertrophy. The right ventricle is the crescent-shaped area on
23 the left side of the image; the right ventricle appears to be of normal size, but the left ventricle is immensely hypertrophied, most likely indicating long-
standing hypertension.
24 ....... _ ----. "-· ·- _____ : -- • -&......·--a.-:-···-- "-· ·- __.. ___ ..._ .. .-.:_..._ .... ·--a.-:-···-- "-· ·- __.. ___ . ._..... ·- _____: -- .. __ .. _: -···-- . . . ·- __.. ___ . ._....__ .. _: -·- ,..._ ---'- ... ·- __.. ___ .._ ... ---
6
lock
s
Suspend
0
End Block
Item: 24 of 48 ~. I • M k <:] t> al ~· ~
QIO: 1412 .l. ar Previous Next lab 'Vfl1ues Notes Calculator
I I y
4 Atrial septal defect Atrium (heart) Diastole Ventricle (heart) Contractility Volume overload Ventricular system
5 c is no t co rrect. 6% chose this.
6 In the first 24 hours after a myocardial infarction, a gross examination of myocardial tissue would reveal area s of pallor; indicating regions of persistent
ischemia. Within 24 - 72 hours, the gross appearance of the heart would become mottled, with patchy area s of pallor in regions with persistent ischemia
7 and other area s of dark-red color where blood flow has been reestablished (hyperemia).
Myocardial infarction Ischemia Hyperaemia Pallor Infarction Blood flow
8
9 D is no t co rrect. 10% chose this.
Pulmonary hypertension would lea d to right ventricular hypertrophy, not left ventricular hypertrophy. The right ventricle is the crescent-shaped area on
10 the left side of the image; the right ventricle appears to be of normal size, but the left ventricle is immensely hypertrophied, most likely indicating long-
standing hypertension.
11
Pulmonary hypertension left ventricular hypertrophy Right ventricular hypertrophy Hypertension Ventricular hypertrophy Ventricle (heart) Hypertrophy Heart
12
13
Bo tto m Line:
14
Left ventricular concentric hypertrophy is a complication of hypertension characterized by wall thickening, as seen in the image.
15 Hypertension Concentric hypertrophy Hypertrophy Heart

16
17

18 I iii I;fi 1!1 I•J f o r year:l 2 0 1 7 ..


FI RST AI D FA CTS
19
20 FA17 p 290.2
21 Hypertension D efin ed as persiste nt systolic BP :e:: 140 mm Hg a nd /or diastolic BP :e:: 90 mm Hg
22 RISK FACTORS t age, o besity, diabetes, physical inactivity, excess salt intake, excess alcohol intake, fam ily h istory;
23 African American > Caucasia n >Asian.
24 FEATURES 90% of hypertcnsion is 1° (essential) and related to t C O or t T PR. Remaining 10% mostly zoto
6
lock
s
Suspend
0
End Block
Item: 24 of 48 ~ 1 • M k -<:J 1>- Jil ~· !:';-~
QIO: 1412 ..L ar Pre v ious Next Lab~lues Not es Calcula t o r
A A

5 FA17 p294.1

6 Evolution of Commonly occluded coronar) arteries: I.AD > RCA > circumAex.
7 myocardial infarction Symptoms: diaphoresis, nausea, \ Omiting, se, ere retroslernal pain, pain in left arm and/or ja\\,
shortness of breath, fati gue.
8
9 TIME GROSS LIGHT MKROS(OPE (OMPUCATIONS

10
0 - 24 hr ~one Early coagulati'e necrosis, Ventricular arrhythmia, I IF',
release of nee rotic cell cardiogenic shock.
11
contents into blood; edema,
12 hemorrhage, wa\'y fibers.
Occluded eutrophils appear.
13 artety
Repcrfusion injury,
14
associated "ith generalion
15 offree radicals, leads to
16 ··~~ Dark mottlmg. hypercontraction of myofibrils
pale with through f free calcium in Aux.

-
17 tetrazolium
stain
18
19
20
0 - - 0

21
22
23
24 •
1- 3 days .,,, Extensi' e coagulati,·e necrosis.
"T': •••• _ . ....- .. .. .J: .. - ;_(... __ •
Postinfarction fibrin ous
---=---.J:.:.
a
Lock
s
Suspend
8
End Bl ock
Item: 25 of 48 ~ 1 • M k -<:J 1>- Jil ~· !:';-~
QIO: 5164 ..L ar Pre v ious Next Lab~lues Not es Calcula t o r

5
A

A 35-year-old man presents to the emergency department with a 1-hour history of difficulty speaking and weakness in his right arm . MRI of the head
and neck confirms a stroke in the left pariet al lobe and widely patent carotid arteries. There is no history or ECG changes suggestive of atrial
IA•A] A

6
fibrillation. Auscultation reveals the sound heard in the audio clip .
7
OPEN MEDIA
8
9
II - oooo i ooo2 ~~~
10
11
12 What best accounts for this patient's stroke?

13
14 A. Aortic stenosis

15 B. Atrial septal defect


16 c. Left atrial thrombus
17
o. Patent ductus arteriosus
18
19 E. Pulmonary stenosis

20 F. Ventricular septal defect


21
22
23
24
• 25 •
a
Lock
s
Suspend
8
End Bl ock
Item: 25 of 48 ~. I • M k <:] t> al ~· ~
QIO: 5164 .l. ar Previous Next lab 'Vfl1ues Notes Calculator

5
6
7
The co rrect a nswer is B. 41% cho se this.
8
9
10
This patient has an atrial septal defect (A5D ), which likely allowed a paradoxical embolus from the right side of the heart to pass into the left side of the
heart and subsequently embolize in the left middle cerebral artery. Normally, inspiration lea ds to increa sed blood in the right ventricle, causing delay in
the closure of the pulmonic valve, and expiration decrea ses blood in the right ventricle, lea ding to earlier closure of the pulmonic valve. However; with an
A5D a certain amount of blood is passing from the left atrium to the right atrium on every bea t . This decrea ses the variation in pulmonary va sculature on
respiration, causing a wide and "fixed" 52 heard on auscultation .
I
11 Atrial septal defect Pulmonary valve Atrium (heart) Middle cerebral artery Auscultation Ventricle (heart) Embolus Circulatory system Embolism Embolization

12 Cerebral arteries Heart

13 A is no t co rrect. 15% cho se this.


Aortic stenosis typically produces a midsystolic crescendo-decrescendo murmur that is best heard at the right second intercostal space. Aortic stenosis
14
would not allow for a paradoxical embolus.
15 Aortic stenosis Stenosis Embolism Intercostal space Embolus Heart murmur

16 c is no t co rrect. 18% cho se this.


A left atrial thrombus could certainly cause a stroke. Inadequat e contraction of the atrium, as seen in atrial fibrillation, lea ds to blood stasis and formation
17 of a thrombus that can spontaneously embolize into the systemic circulation . However the patient does not have atrial fibrilation .
18 Atrial fibrillation Thrombus Embolization Embolism Circulatory system Atrium (heart) Stroke Fibrillation

19 D is no t co rrect. 10% cho se this.


Pa t ent ductus arteriosus causes a continuous machine -like murmur that is loudest at 52. This is not consistent with the examination findings.
20
Patent ductus arteriosus Ductus arteriosus Heart murmur
21
E is no t co rrect. 6 % cho se this.
22 Pulmonary stenosis usually produces a systolic ej ection murmur at the left second intercostal space. The murmur in this question is similar to what one
might hear in pulmonary stenosis, but you would not expect to hear fixed splitting of 52. A paradoxical embolus would not be possible in this case either;
23 since there is no connection between the right heart and left heart circulation .
24 Pulmonic stenosis Stenosis Systole Heart Heart murmur

25 F is no t co rrect. 10% cho se this.

6
lock
s
Suspend
0
End Block
Item: 25 of 48 ~. I • M k <:] t> al ~· ~
QIO: 5164 .l. ar Previous Next lab 'Vfl1ues Notes Calculator

5 E is not correct. 6 % chose this.


6 Pulmonary stenosis usually produces a systolic ej ection murmur at the left second intercostal space. The murmur in this question is similar to what one
might hear in pulmonary stenosis, but you would not expect to hear fixed splitting of 5 2. A paradoxical embolus would not be possible in this case either;
7 since there is no connection between the right heart and left heart circulation .
Pulmonic stenosis Stenosis Systole Heart Heart murmur
8
F is not correct. 10% chose this.
9
Although a paradoxical embolus would certainly be possible with a ventricular septal defect (V5D), the murmur heard here is not consistent with this.
10
11
12
With a V5D you typically hear a loud, harsh, or blowing holosystolic murmur over the left sternal border in the third or fourth intercostal space. V5D
causes paradoxical emboli only if there is a right-to-left shunt ( Eisenmenger syndrome).
Ventricular septal defect Eisenmenger' s syndrome Right-to-left shunt Embolism Intercostal space Ventricle (heart) Heart murmur I
13
Bottom Line:
14
A fixed and widely split 5 2 is pathognomonic for an atrial septal defect (A5D). A5Ds can allow a venous thrombus to enter the left side of the heart and
15 cause embolization in the systemic circulation (known as a paradoxical embolus).
Atrial septal defect Thrombus Embolism Circulatory system Embolus Split 52 Embolization Atrium (heart) Venous thrombosis Vein
16
17
18
lijj ;fi IJ l•l for year:l 2017 ..
19 FIRST AID FAC T S

20
FA17p277.1
21
Splitting
22
Normal spl itting Inspiration .... drop in intrathoracic pressure
23
24
-+ t venous return -+ t RV filling -+ t RV
E
I II
Sl A2 P2
stroke volu me -+ t RV ejection time
25 - delayed closure of pulmon ic valve. I .. t~l. .
6
lock
s
Suspend
0
End Block
Item: 25 of 48 ~ 1 • M k -<:J 1>- Jil ~· !:';-~
QIO: 5164 ..L ar Pre v ious Next Lab~lues Not es Calcula t o r
• R • • • R -

A A

5 "paradoxically" eliminating the split (usually


6 heard in expiration).
7
8 FA17p288.1
9 Congenital heart diseases
10 RIGHT·TO·lEfT SHUNTS Early cranosis-"blue babies." Often diagnosed The 5 Ts:
11 prenatally or become e\·ident immediately 1. 'I runcus arteriosus (I ,·essel)
after birth. Usually require urgent surgical 2. Transposition (2 switched vessels)
12
treatment and/or maintenance of a PDA. 3. Tricuspid atresia (3 = Tri)
13
4. 'Jetralogy of Fallot (4 = Tetra)
14 5. TAPVR (; letters in the name)
15
16
17
Persistent truncus
arteriosus
Truncus arteriosus fai ls to divide into
pulmonary trunk and aorta due to lack of
aorticopulmonary septum forma tion; most
patients have accompanying VSD.
I
18
19 D-transposition of Aorta leaves RV (anterior) and pu lmonary trunk
great vessels leaves LV (posterior) .... separation of systemic Pulmonary
20 artery
and pulmonary circulations. 1 ot compatible
21
with life unless a shunt is present to allo11
22 mixing of blood (eg, VSD, PDA, or patent
23 foramen ovale). Right
ventricle
24 Due to failure of the aorticopulmonmy septum
25 •
to spiral.
• "' • • • I
. ... . . ,. . ..
a
Lock
s
Suspend
8
End Bl ock
Item: 26 of 48 ~ 1 • M k -<:J 1>- Jil ~· !:';-~
QIO: 100 7 ..L ar Pre v ious Next Lab~lues Notes Calcula t o r
A A

6 A 22-year-old man collapses and dies while participating in an athletic event. He had recently been seen by a physician for a sore throat and fever,
but he was otherwise healthy. His family members report that he had an uncle who died suddenly at a young age.
7
8
Which of the following would most likely be observed in a biopsy specimen of the cardiac muscle at autopsy?
9
10
A. Coagulative necrosis of myofibers and numerous neutrophlls
11
B. Diffuse lymphocytic infiltration
12
13 c. Disarray of bundles of myocytes and sarcomeres within cells

14 D. Discrete foci of fibrinoid degeneration and mononuclear inflammatory cells


15
E. Positive Congo red staining within the myofibers
16
17
18
19
20
21
22
23
24
25
. 26 •
a
Lock
s
Suspend
8
End Bl ock
Item: 26 of 48 ~ 1 • M k -<:J 1>- Jil ~· !:';-~
QIO: 100 7 ..L ar Prev ious Next Lab~lues Notes Calculat o r

A A

6 Th e co rrect a nswer i s c . 90% chose this.


Known as hypertrophic obstructive cardiomyopathy or asymmetric septal hypertrophy with idiopathic
7 hypertrophic subaortic stenosis, this disorder follows an autosomal dominant mode of inheritance in more
8 than SO% of cases, but is otherwise idiopathic. The image of microscopic pathology shows myofiber
hypertrophy and disarray. It is a common cause of sudden death in young athletes.
9 Hypertrophic cardiomyopathy Idiopathy Myocyte Stenosis Dominance (genetics) Hypertrophy Cardiomyopathy

10 Autosome Pathology Ventricular hypertrophy Aorta

11
12
13
14 Image courtesy of Mecchia D, eta/. (2013)
15
A is not correct.
16 Extensive neutroph ilic i nfiltrate within necrotic myofibers on h istology Is typically seen 3-4 days following a myoca rdial infarction. This man died
17 Immediately after h is cardiac event.
Myocardial infarction Histology Necrosis Neutrophil
18
6 is not co rrect . 4 0/o chose this.
19 Lymphocytic Infiltration is typically seen in viral myocarditis. Although this young man complained of symptoms consistent with a mild viral Illness, his
20 sudden death and family history make a diagnosis of hypertrophic cardiomyopathy more likely.
Hypertrophic cardiomyopathy Myocarditis Cardiomyopathy Hypertrophy Ventricular hypertrophy Lymphocyte Virus
21
D is no t co rrect. 1 0/o ch ose this.
22 Aschoff bodies are collections of lymphocytes, m acrophages, and occasionally giant cells within the myocardium surrounded by fibrinoid degeneration. This
23 finding Is pathognomonic for rheumatic heart disease.
Aschoff body Rheumatic fever Pathognomonic cardiac muscle Macrophage Lymphocyte Giant cell Cardiovascular disease
24
E i s n o t correct. 5% chose this.
25 Congo red sta ining that turns green under polarized light indicates the presence of amyloid within the myocardium. Restrictive cardiomyopathy Is an
26 uncommon form of cardiomyopathy involving infilt rative diseases such as amyl oidosis, which lead to ventricular stiffening and impaired cardiac filling

a
Lock
s
Suspend
8
End Block
Item: 26 of 48 ~. I • M k <:] t> al ~· ~
QIO: 1007 .l. ar Previous Next lab 'Vfl1ues Notes Calculator

6 Aschoff bodies are collections of lymphocytes, macrophages, and occasionally giant cells within the myocardium surrounded by fibrinoid degeneration . This
finding is pathognomonic for rheumatic heart disea se.
7 Aschoff body Rheumatic fever Pathognomonic Cardiac muscle Macrophage lymphocyte Giant cell Cardiovascular disease
8 E is not correct. 5 % chose this.
9 Congo red staining that turns green under polarized light indicat es the presence of amyloid within the myocardium . Restrictive cardiomyopathy is an
uncommon form of cardiomyopathy involving infiltrative disea ses such as amyloidosis, which lea d to ventricular stiffening and impaired cardiac filling
10 during diastole.
Restrictive cardiomyopathy Cardiac muscle Amyloidosis Diastole Cardiomyopathy Staining Congo red
11

12
13 Bottom Line:

14 There are three main types of cardiomyopathy, including dilat ed cardiomyopathy (enlarged ventricular chambers), restrictive cardiomyopathy (reduced
myocardial compliance), and hypertrophic cardiomyopathy, also known as idiopathic hypertrophic subaortic stenosis (abnormally enlarged ventricular
15 septum).
Hypertrophic cardiomyopathy Dilated cardiomyopathy Restrictive cardiomyopathy Cardiomyopathy Interventricular septum Stenosis Hypertrophy
16 Ventricular hypertrophy Septum Ventricular system Idiopathy Aorta
17
18
19 I ill ;fi 1!1 I•J for year:[ 2017 ..
FI RST AID FA CTS
20
21
FA17p297.1
22 Cardiomyopathies
23 Dilated Most common cardiomyopathy (90% of cases). Systolic dysfunction ensues.
24 cardiomyopathy Often idiopathic or familial. Other etiologies Eccentric hypertro phy · (sarcomeres added in
25 include ch ronic Alcohol abuse, wet Beriberi, series).
26 Coxsackie B viral myocarditis, chronic ABCCCD.

6
lock
s
Suspend
0
End Block
Item: 26 of 48 ~ 1 • M k -<:J 1>- Jil ~· !:';-~
QIO: 100 7 ..L ar Pre v ious Next Lab~lues Notes Calcula t o r
A
~ . ~ . . .. A

6 Amyloidosis, Sarcoidosis, l lemochromatosis Loffier syndrome-endomyocard ial fibrosis with


7 (although dilated cardiomyopathy is more a prominent eosinophilic infiltrate.
8 common) (Puppy LE.\ SIJ).
9
10 FA17 p 300.1

11 Rheumatic fever A consequence of phar) ngeal infection with J•:-.. ES (major criteria):
12 group A P-hemol}1ic streptococci. Late Joint (migratory polrarthritis)
sequelae include rheumatic heart disease. • (carditis)
13
which affects heart valves- m itral > aortic >> l\odules in skin (subcutaneous)
14
tricuspid (high-pressure valves affected most). 1-.ryt bema marginatum
15 Early lesion is mitral valve regurgitation; S\'clenham chorea

16 late lesion is mitral stenosis. Associated
17 with Aschoff bodies (granuloma" ilh giant
cells [blue arrows in r:J]), Anitschkow cells
18
(enlargeclmacrophages with ovoid, wavy,
19
rod-like nucleus [red arrow in fJ 1), t anti-
20 streptolysin 0 (ASO) titers.
21 Immune mediated (type II hypersensit ivity);
22 not a direct effect of bacteria. Ani ibodies
23
toM protein cross-react with self antigens
(molecular mimicry).
24
Treatment/prophylaxis: penicillin.
25
26 •
a
Lock
s
Suspend
8
End Bl ock
Item: 27 of 48 ~ 1 • M k -<:J 1>- Jil ~· !:';-~
QIO: 5163 ..L ar Pre v ious Next Lab~lues Notes Calcula t o r

7
A

A 57-year-old man presents with chest pain. He has no known history of heart disease, but has not seen a physician in 40 yea rs. On examination, the
Romberg test was positive. Echocardiogra phy reveals aortic valvular insufficiency and a linearly calcified, aneurysmal ascending aorta. Before a repair
•A]
IA A

8
can be performed, the patient dies.
9
10 Which laboratory test would be most helpf ul in determining the etiology of this patient's aortic pathology?
11
12 A. 5-Hydroxymdoieacetic add

13 B. Antistreptolysin o titer
14
c. Bacterial cultures
15
D. Fluorescent treponema! antibody-a bsorption t est
16
17 E. Urine drug screen

18
19
20
21
22
23
24
25
26
. 27 •
a
Lock
s
Suspend
8
End Bl ock
Item: 27 of 48 ~ 1 • M k -<:J 1>- Jil ~· !:';-~
QIO: 5163 ..L ar Prev ious Next Lab~lues Notes Calculat o r

A A

7
Th e co rrect an swer i s D. 660/o ch ose this.
8 Fluorescent treponema! antibody-a bsorption test (FTA-ABS) is a test used to detect antibodies directed
9 against the species Treponema palladium . This would be the most appropriate test in this case because
the patient has displayed signs and symptoms consistent with this pathology. Tertiary syphilis disrupts the
10 vasa vasorum of the aorta, consequently dilating the aortic root and aortic valve ring. This leads to aortic
aneurysm and thus aortic insufficiency. Additionally, the ascending arch of the aorta is often calcified and
11 atherosclerotic. Tertiary syphilis can lead t o "t ree-barking, • irregular wrinkling of the tunica intima of the
aorta, as well as gummas (like that shown here). Tertiary syphilis may also lead to tabes dorsalis,
12 manifesting as sensory ataxia with a positive Romberg sign, as seen in this case.
13 Tunica intima Tabes dorsa ·s Vasa vasorum Aortic insufficiency Sypho os Aortic ane y ·m .:.orta Ataxia

14 Aortic valve Atherosclerosis Aneurysm Antibody Aortic arch Tertiary syphilis Pathology Ascending aorta

15
Image courtesy of CDC/ J. Pledger
16
17 A is not co rrect. 70/o chose this.

18 5-Hydroxyindoleacetic acid is elevated in ca rcinoid syndrome. Carcinoid syndrome can lead to right-sided valvular lesions, but is not known to cause aortic
aneurysms.
19 Carcinoid syndrome Carcinoid 5-Hydroxyindoleacetic acid

20 8 is not co rrect. 120/o chose this.


21 Antistreptolysin 0 titers would be helpful in diagnosing streptococcal Infection. Rheumatic fever is known to occur after a streptococcal infection, typically
pharyngeal. Rheumatic fever can cause valvular damage (mitral > aortic>> tricuspid), but is not associated with the aortitis characteristic of tertiary
22 syphilis.
Rheumatic fever Syphilis Anti-streptolysin 0 Streptococcus Group A streptococcal infection Aortitis Infection Fever Rheumatism Pharynx
23
C is n ot co rrect. 100/o ch ose this.
24
Bacterial cultures could be helpful in diagnosing endocarditis. Endocarditis of the aortic valve could contribute to aortic insufficiency, but vegetations would
25 likely be seen on echocardiogram. I n addition, endocarditis is usually accompanied by systemic symptoms of malaise, low-grade fever, weight loss, and
splinter hemorrhages, none of which were seen in this patient.
26 Aortic nsuftoclency Echocardiography Aortic valve Endocarditis Bleeding Weight oss Fever Malaise
27 • F ic nnt rnrrPrt. ~ om rhn~~;_p t hic;..

a
Lock
s
Suspend
8
End Block
Item: 27 of 48 ~. , . M k <:] t> al ~· ~
QIO: 5163 .l. ar Previous Next lab 'Vfl1ues Notes Calculator
Aortic insufficiency Echocardiography Aortic valve Endocarditis Bleeding Weight loss Fever Malaise
7
E is not correct. 5 % chose this .
8
Substances such as cocaine and alcohol are known to cause dilated cardiomyopathy, but are not known to cause the aortitis seen with tertiary syphilis.
9 Dilated cardiomyopathy Syphilis Aortitis Cardiomyopathy Cocaine Alcohol

10
11 Bottom Line:
12 Tertiary syphilis occurs years after the initial infection. It is characterized by formation of gummas, neurologic dysfunction, and cardiac complications of
syphilitic aortitis, aortic aneurysm, and aortic regurgitation.
13
Syphilis Aortic insufficiency Syphilitic aortitis Gumma (pathology) Aortic aneurysm Aortitis Aneurysm Tertiary syphilis Infection Neurology
14
15
16 I iii I;fi 1!1 I•J for year:l 2017 ..
FI RST AI D FA CTS
17
18
FA17 p 143.1
19
Syphilis Caused by spirochete Treponema pallidum.
20
Primary syphilis Localized disease presenting with painless chancre fi.1. lf available, use dark-field microscopy to
21 visualize treponemes in fluid from chancre : . VORL® in ~ 80%.
22 Secondary syphilis Disseminated disease with constitutional symptoms, maculopapular rash ~ (including palms (!]
23 and soles), condylomata lata 0 (smooth, moist, painless, wart-like white lesions on genitals),
24 lymphadenopathy, patchy hair loss; also confirmable with dark-field microscopy.
25 Serologic testing: VDRL/RPR (nonspecific), confirm diagnosis with specific test (eg, F'TA-ABS).
Secondary syphil is= Systemic. Latent syphilis (® serology without symptoms) may follow.
26
27
Tertiary syphilis Cummas D (chronic granulomas), aortitis (vasa vasorum destruction), neurosyphilis (tabes dorsalis,

6
lock
s
Suspend
0
End Block
7
8
9
10
11
12
13
14
15
16
17
18
19
20
FA17 p 300.4
21
Syphilitic heart 3° syphilis disrupts the vasa vasorum of the Can result in aneurysm of ascending aorta or
22 disease aorta with consequent atrophy of' cssel wa II aortic arch, aortic insufficiency.
23 and dilatation of aorta and valve ring.
24 May see calcification of aortic root, ascending
25 aortic arch, and thoracic aorta. Leads to "tree
bark" appearance of aorta.
26
27 •
a
Lock
s
Suspend
8
End Block
Item: 28 of 48 ~ 1 • M k -<:J 1>- Jil ~· !:';-~
QIO: 5166 ..L ar Pre v ious Next Lab~lues Not es Calcula t o r
A A

8 A 53-year-old man presents to an outpatient clinic with a chief complaint of unremitting diarrhea of 2 weeks' duration . o n physical examination the
9 physician discovers a heart murmur. The patient is sent for an echocardiogram, which reveals pulmonic valve stenosis and tricuspid insufficiency. The
aortic and mitral valves are normal. The physician is concerned that the patient's valvular disease may be relat ed to an underlying neoplasm. The
10 appropriate oncologic work-up is performed, and a neoplasm is indeed discovered.

11
Which of the following markers likely aided t he physician in the diagnosis of this neoplasm?
12
13
A. o-fetoprotein
14
B. 5-hydroxymdoleacetic add
15
16 c. Cancer antigen-125

17 o. Carbohydrate antigen 19-9


18
E. Carcinoembryonic antigen
19
20
21
22
23
24
25
26
27
. 28 •
a
Lock
s
Suspend
8
End Bl ock
Item: 28 of 48 ~ 1 • M k -<:J 1>- Jil ~· !:';-~
QIO: 5166 ..L ar Prev ious Next Lab~lues Not es Calculat o r

A A

8
9 The correct an swer i s B. 500/o chose this.
10 Serotonin Is secreted by carcinoid tumors and causes symptoms of flushing, watery diarrhea, and right-sided va lvular dysfunction. 5-hydroxylndoleacetlc
acid (5-HIAA) is a metabolite of serot onin t hat ca n be measured in a 24-hour urine collection. suspect intravenous drug use or carcinoid syndrome when
11 there Is any pathology involving right-sided valves.
Carcinoid ~ynd ome 5-Hydroxyindoleacetic acid Serotonin carcinoid Diarrhea Intravenous therapy Metabolite Urine Pathology Neoplasm Flushing (physiology1
12
A is not correct. 12% chose this.
13
o-fetoprotein (AFP) is a tumor marker used to d et ect hepatocellular carcinoma and gonadal germ cell t umors (such as yolk sac cardnoma). AFP Is not a
14 marker used to detect or diagnose cardnoid tumors.
Tumo ma e Hepatocellular carcinoma Germ cell Yolk sac Carcinood Germ eel tumo l'leoplasm Carcinoma Carcinoid syndrome
15
Cis not correct. 11% chose this.
16
Cancer antigen-125 (CA-12 5) is a tumor marker used to monitor the response to treatment and to detect recurrence of ovarian cancer. About 80% of
17 patients with ovarian cancer have elevated serum levels of CA-12 5, which vary by tumor stage. CA- 125 is not helpful in the diagnosis of carcinoid tumors.
Tumor marker CA-125 Ovarian cancer Carcinoid Cancer staging Neoplasm Carcinoid syndrome Cancer Serum (blood)
18
0 is not co rrect. 11 Ofo chose this.
19
Carbohydrate antigen 19 -9 (CA-19-9) is a t umor marker used to monitor the progress of pancreatic adenocarcinoma. It is not helpful for the diagnosis of
20 carcinoid tumors. While pancreatic adenocarcinoma can occassionally present with non-bacterial thrombotic endocarditis, this valvular pathology Is not
consistent with the patient's symptoms.
21 Tumor marker CA19-9 Carbohydrate Endocarditis Carcinoid Antigen Neoplasm Carcinoid syndrome Adenocarcinoma Pathology Pancreatic cancer Thrombosis
22 E is no t co rrect. 160/o chose this.
23 Carclnoembryonic antigen (CEA) is a relatively nonspecific t umor marker. It Is produced in N 70% of colorectal and pancreatic ca ncers. It is also produced
by some gastric and breast carcinomas. CEA is not helpful in the diagnosis of carcinoid tumors.
24 Carcinoembryonic antigen Tumor marker Carcinoid Antigen Carcinoid syndrome Neoplasm
25
26
Bottom Line:
27
Carcinoid tumors secrete serot onin, which ca uses flushing, watery diarrhea, and right-sided valvular lesions. 5-HIAA is a metabolite of serotonin that can
28 be detected in the urine.

a
Lock
s
Suspend
8
End Block
Item: 28 of 48 ~- I • M k <:] t> al ~· ~
QIO: 5166 .l. ar Previous Next lab 'Vfl1ues Notes Calculator
consistent with the patient's symptoms.
8 Tumor marker CA19-9 Carbohydrate Endocarditis Carcinoid Antigen Neoplasm Carcinoid syndrome Adenocarcinoma Pathology Pancreatic cancer Thrombosis
9 E is not correct. 16% chose this .
10 Carcinoembryonic antigen (CEA) is a relatively nonspecific tumor marker. It is produced in ~ 70% of colorectal and pancreatic cancers. It is also produced
by some gastric and breast carcinomas. CEA is not helpful in the diagnosis of carcinoid tumors.
11 Carcinoembryonic antigen Tumor marker Carcinoid Antigen Carcinoid syndrome Neoplasm
12
13
Bottom Line:
14 Carcinoid tumors secrete serotonin, which causes flushing, watery diarrhea, and right -sided valvular lesions. 5-H IAA is a metabolite of serotonin that can
15 be detected in the urine.
Serotonin 5-Hydroxyindoleacetic acid Diarrhea Carcinoid Metabolite Carcinoid syndrome Urine Neoplasm Flushing (physiology)
16
17

18
I iii I;fi 1!1 I•J for year:l 2017 ..
FI RST AI D FA CTS
19
20
FA17 p 220.2
21
22
Serum tumor markers Tumor markers should not be used as the !0 tool for cancer diagnosis or screen ing. They may be
used to monitor tumor recurrence and response to therapy, but definitive diagnosis is made via
23
biopsy.
24 MARKER ASSOCIATED CANCER NOTES
25 Alkaline phosphatase Metastases to bone or liver, Paget disease of Must exclude hepatic origin by checking LFTs
26 bone, sem inoma (placental ALP). and GGT levels.
27 a -fetoprotein Hepatocellular carcinoma, hepatoblastoma, yolk Normally made by fetus. Transiently elevated in
28 sac (endodermal sinus) tumor, mixed germ pregnancy. High levels associated with neural

6
lock
s
Suspend
0
End Block
Item: 28 of 48 ~ 1 • M k -<:J 1>- Jil ~· !:';-~
QIO: 5166 ..L ar Pre v ious Next Lab~lues Not es Calcula t o r
A A

8 Chromogranin l'\euroendocrine tumors.


9 PSA Prostate cancer. Prostate-specific antigen.
10 Can also be eb·ated in BPI-I and prostatitis.
11 Questionable risk/benefit for screening.
12
Suneillance marker for recurrent disease after
prostatectomy.
13
14
FA17 p 338.5
15
16
Carcinoid syndrome Rare syndrome caused by carcinoid tumors Rule of l/3s:
(neuroendocrine cells · ; note prominent 1/3 metastasize
17
rosettes [arrow]), especially metastatic small 1/3 present with 2nd malignancy
18 bowel tumors, which secrete high levels l/3 are multiple
19 of serotoni n (5-HT). Not seen if tumor is i\ lost common mal ignancy in the small
20 Iimited to Cl tract (5-HT undergoes first-pass intestine.
21
metabolism in liver). Results in recurrent
diarrhea, cutaneous Aushing, asthmatic
22
wheezing, right-sided valvular heart disease
23 (tricuspid regurgitation, pulmon ic stenosis).
24 f 5-hydroxyindoleacetic acid (5-HIAA) in
25 urine, niacin deficiency (pellagra).
26 Treatment: surgical resection, somatostatin
analog (eg, octreotide).
27
28 •
a
Lock
s
Suspend
8
End Bl ock
Item: 29 of 48 ~ 1 • M k -<:J 1>- Jil ~· !:';-~
QIO: 2439 ..L ar Pre v ious Next Lab~lues Not es Calcula t o r
A A

9 A 4-year-old boy with a history of intellectual disability and seizures is brought to the physician with a 3-month history of worsening shortness of
breath. During physical examination, the physician notices numerous acne-like papules on the patient's face . Echocardiography shows significant left
10
ventricular outflow obstruction.
11
12 Which of the following is the most likely diagnosis for this patient's heart condition?

13
14 A. Dilated cardiomyopathy

15 B. Lipoma
16
c. Myxoma
17
o. Rhabdomyoma
18
19 E. Transposition of the great vessels

20
21
22
23
24
25
26
27
28
. 29 •
a
Lock
s
Suspend
8
End Bl ock
Item: 29 of 48 ~ 1 • M k -<:J 1>- Jil ~· !:';-~
QIO: 2439 ..L ar Prev ious Next Lab~lues Not es Calculat o r

A A

9
Th e co rrect an swer i s D. 560/o chose this.
10 Tuberous sclerosis is a genetic condition (autosomal dominant) characterized by nodular proliferation of
11 multinucleated atypical astrocytes. These form tubers, which are found throughout the cerebral cortex and
perlventrlcular areas. The classic t riad, which manifests in only the most severe of cases, consists of
12 seizures, intellectual disability, and facial angiofibromas (a lso known as adenoma sebaceum). Half of
patients with tuberous sclerosis develop rhabdomyomas (shown in the Image), primary tumors of cardiac
13 muscle that, although benign, may compromise cardiac function. They may be found on atrioventricular
valves or on ventricle walls, causing outflow tract obstruction. Tuberous sclerosis is also notable for a link to
14
angiom yolipomas of the kidney.
15 Tubero10s lerosis Cerebral cortex Dominance (genetics) Intellectual disability cardidc muscle Astrocyte Kidney
Heart valve Autosome Adenoma Ventricular system Epileptic seizure Angiomyolipoma Neoplasm
16
Genetic disorder Rhabdomyoma Muscle
17
18
Image courtesy of Armed Forces Institute of
19
Pathology
20
A is not co rrect. 100/o chose this.
21
Dilated cardiomyopathy is often idiopathic. It involves four-chamber hypertrophy and dilation, and eventually heart failure . This condition is not associated
22 with tuberous sclerosis. Note that hypertrophic ca rdiomyopathy also causes ventricular outflow obstruction and is often responsible for sudden death in
young athletes.
23
Hypertrophic cardiomyopathy Dilated cardiomyopathy Cardiomyopathy Hypertrophy Idiopathy Ventricular hypertrophy Heart failure Vasodilation
24
B is no t co rrect . aoto ch ose this.
25 Lipomas, like rhabdomyomas, are capable of obstruction. These can create ball-valve obstructions and are most often located in the left ventricle, right
atrium, or atrial septum. However, in conjunction with the history suggesting tuberous sclerosis, lipoma is a less likely diagnosis.
26
Upoma Atrium (heart) Tuberous sclerosis I nteratrial septum Ventricle (heart) Ventrjcylar system Rhabdomyoma Septum
27
C i s n ot correct. 190/o ch ose this.
28 Myxomas, like rhabdomyomas, are capable of obstruction. However, these tumors are almost exclusively seen in adults and are usually located In the
atria, making this a less likely diagnosis.
29 •
a
Lock
s
Suspend
8
End Block
Item: 29 of 48 ~. I • M k <:] t> al ~· ~
QIO: 2439 .l. ar Previous Next lab 'Vfl1 ues Notes Calculator

9 lipomas, like rhabdomyomas, are capable of obstruction. These can crea t e ball-valve obstructions and are most often locat ed in the left ventricle, right
atrium, or atrial septum. However; in conjunction with the history suggesting tuberous sclerosis, lipoma is a less likely diagnosis.
10 lipoma Atrium (heart) Tuberous sclerosis Interatrial septum Ventricle (heart) Ventricular system Rhabdomyoma Septum

11 c is not correct. 19% chose this.


12 Myxomas, like rhabdomyomas, are capable of obstruction. However; these tumors are almost exclusively seen in adults and are usually locat ed in the
atria, making this a less likely diagnosis.
13 Atrium (heart) Rhabdomyoma Atrium (architecture)

14 E is not correct. 7 % chose this.


15 Transposition of the grea t vessels is a situation in which the pulmonary trunk arises from the left ventricle and the aorta arises from the right ventricle.
This arrangement is incompatible with life, and a compensatory anomaly such as a pat ent ductus arteriosus is necessary.
16 Aorta Ventricle (heart) Pulmonary artery

17

18 Bottom Line:
19 Tuberous sclerosis is an autosomal-dominant condition, classically manifesting with seizures, intellectual disability, and facial angiofibromas; it is also
associat ed with development of cardiac rhabdomyomas.
20 Tuberous sclerosis Intellectual disability Genetic disorder Dominance (genetics) Rhabdomyoma Epileptic seizure
21
22
23 I ill ;fi 1!1 I•J for year:[ 2017 ..
FI RST AI D FA CTS
24
25
FA17 p495.1
26
Neurocutaneous disorders
27
Sturge-Weber Congenital, no nin herited (sporadic), develo pmental anomaly of neural c rest derivatives due to
28 syndrome somatic mosaicism for a n activating mutatio n in one copy of the G AQ gene. Affects small
29 (encephalotrigeminal (capillary-sized) blood vessels .... port-wine sta in of the face r.J (nevus Aammeus, a non-neo plastic

6
lock
s
Suspend
0
End Block
10
11 FA17 p 56.1

12 Autosomal dominant Achondroplasia, autosomal dominant polycystic kidney disease, familial adenomatous polyposis,
13 diseases familial hypercholesterolemia, hereditar) hemorrhagic telangiectasia, h ered ita r~ spherOC) tosis,
14 Huntington disease, Li-Fraumeni S) ndrome, \ larfan srndrome, multiple endocrine neoplasias,
neurofibromatosis type I (von Recklinghausen disease), neurofibromatosis type 2, tuberous
15
sclerosis, von II ippel-Lindau disease.
16
17
FA17 p 301 .1
18 Cardiac tumors l\lost common heart tumor is a metastasis (eg, melanoma).
19
Myxomas !\ lost common 1° cardiac tumor (red arrows) in adults fl. 90% occur in the atria (mostly left
20 atrium). Myxomas are usually described as a "ball valve" obstruction in the left atrium (associated
21 with mu ltiple syncopal episodes). \!lay auscultate early diastolic "tumor plop" sound. Histology:
22 gelatinous materi;d, myxoma cel ls immersed in glycosaminoglycans.
23
24
25
26
27 Rhabdomyomas .l\lost frequent 1° cardiac tumor in children (associated with tuberous sclerosis). Histology:
hamartomatous growths.
28
29 •
a
Lock Suspend
s 8
End Block
Item: 30 of 48 ~ 1 • M k -<:J 1>- Jil ~· !:';-~
QIO: 1026 ..L ar Pre v ious Next Lab~lues Notes Calcula t o r
A A

10 A 3-year-old boy presents to his pediatrician with fever, conjunctivitis, erythema of the oral mucosa, and cervical lymphadenopathy. The diagnosis Is
vasculitis of the small- and medium-sized vessels. The patient improves with supportive care alone, but the parents are warned about a future
11
complication from the disease that will require pediatric subspecialty care .
12
13 What Is the complication associated with the disease?
14
15 A. Coronary aneurysm

16 B. Glomerulonephritis
17
c. Scarlet fever
18
D. Splenomegaly
19
20 E. Stroke

21
22
23
24
25
26
27
28
29
o30 •
a
Lock
s
Suspend
8
End Bl ock
Item: 30 of 48 ~ 1 • M k -<:J 1>- Jil ~· !:';-~
QIO: 1 0 26 ..L ar Prev ious Next Lab~lues Notes Calculat o r

A A

10
Th e co rrect an sw er i s A. 700/o ch ose this.
11 This child has Kawasaki disease, a self-limiting process that typically affects Infants and children <4 years of age, especially those of Asian ethnlclty.
12 Typical presenting symptoms include conjunctival erythema, oral blisters, edema of the hands and feet, a desquamative rash, and enlarged cervical lymph
nodes. The most common histopathologic sign of Kawasaki disease is acute necrotizing vasculitis of small- and medium-sized vessels. The cardiac
13 complications from this disease can lead to vasculitic changes in the coronary arteries, with subsequent coronary artery aneurysms and possible
myocardial Infarction. These patients require serial echocardiograms to screen for disease progression. Occasionally these patients require cardiac
14 catheterfzatlons .
•a 1asa disease Myocardial infarction Erythema Vasculitis 5ystemoc vasculitis Nee• usos Edema Coronary circulation Rash Conjunctiva Aneurysm
15
Cervoca tymp odes Lymph node Artery Histopathology Lymph
16
B i s not correct. 18% chose this.
17
Unlike most vasculitides, Kawasa ki disease has minimal effect on the renal microvasculature. One very common pediatric illness that may be confused
18 with the above presentation is poststreptococcal glomerulonephritis (PSGN ). The clinical presentation is similar, but 4 - 6 weeks after the febrile Illness the
patient develops hematuria, proteinuria, and hypertension. The pathophysiology of PSGN involves immune complex deposition at the glomerular
19 basement membrane.
Kawasaki disease Immune complex Hematuria Proteinuria Glomerular basement membrane Basement membrane Glomerulonephritis Hypertension vasculitis
20
Fever Glomerulus Streptococcus Microcirculation Kidney
21
C is not co rrect. 40/o chose this.
22
Scarlet fever Is included in the differential diagnosis, rather than a complication of Kawasaki disease. It is caused by group A Streptococcus Infection. The
23 clinical features are similar to Kawasaki disea se; however, scarlet fever Is distinguished by a positive group A Streptococcus throat culture.
Kawasaki disease Scarlet fever Differential diagnosis Streptococcus
24
D is not co rrect. JO/o ch ose this.
25
Another part of the differential of Kawasaki disea se is infectious mononucleosis (IM) caused by Epstein-Barr virus infection . Splenomegaly Is a
26 characteristic finding of IM, a disease that shares some clinical features (eg, lymphadenopathy and fever) with Kawasaki disease. However, the
conjunctivitis and desquamative rash in this patient, as well as the description of the disease as a vasculitis, are much more congruent with a diagnosis of
27 Kawasaki disease. Also, splenomegaly is not a typical finding in Kawasaki disease.
Infectious mononucleosis Epstein-Barr virus Kawasaki disease Splenomegaly Lymphadenopathy Conjunctivitis Vasculitis Virus Rash Fever Infection
28
29 E i s n ot corr ect. 5% ch ose this.
Although this disease is a vasculitis, it is unlikely to be a source of strokes In the affected pediatric population.
30 • V~c;r~ll tic; PPrli~t rc;

a
Lock
s
Suspend
8
End Bl ock
Item: 30 of 48 ~ 1 • M k -<:J 1>- Jil ~· !:';-~
QIO: 1 0 26 ..L ar Prev ious Next Lab~lues Notes Calculat o r

A A

10
Th e co rrect an sw er i s A. 700/o ch ose this.
11 This child has Kawasaki disease, a self-limiting process that typically affects Infants and children <4 years of age, especially those of Asian ethnlclty.
12 Typical presenting symptoms include conjunctival erythema, oral blisters, edema of the hands and feet, a desquamative rash, and enlarged cervical lymph
nodes. The most common histopathologic sign of Kawasaki disease is acute necrotizing vasculitis of small- and medium-sized vessels. The cardiac
13 complications from this disease can lead to vasculitic changes in the coronary arteries, with subsequent coronary artery aneurysms and possible
myocardial Infarction. These patients require serial echocardiograms to screen for disease progression. Occasionally these patients require cardiac
14 catheterfzatlons .
•a 1asa disease Myocardial infarction Erythema Vasculitis 5ystemoc vasculitis Nee• usos Edema Coronary circulation Rash Conjunctiva Aneurysm
15
Cervoca tymp odes Lymph node Artery Histopathology Lymph
16
B i s not correct. 18% chose this.
17
Unlike most vasculitides, Kawasa ki disease has minimal effect on the renal microvasculature. One very common pediatric illness that may be confused
18 with the above presentation is poststreptococcal glomerulonephritis (PSGN ). The clinical presentation is similar, but 4 - 6 weeks after the febrile Illness the
patient develops hematuria, proteinuria, and hypertension. The pathophysiology of PSGN involves immune complex deposition at the glomerular
19 basement membrane.
Kawasaki disease Immune complex Hematuria Proteinuria Glomerular basement membrane Basement membrane Glomerulonephritis Hypertension vasculitis
20
Fever Glomerulus Streptococcus Microcirculation Kidney
21
C is not co rrect. 40/o chose this.
22
Scarlet fever Is included in the differential diagnosis, rather than a complication of Kawasaki disease. It is caused by group A Streptococcus Infection. The
23 clinical features are similar to Kawasaki disea se; however, scarlet fever Is distinguished by a positive group A Streptococcus throat culture.
Kawasaki disease Scarlet fever Differential diagnosis Streptococcus
24
D is not co rrect. JO/o ch ose this.
25
Another part of the differential of Kawasaki disea se is infectious mononucleosis (IM) caused by Epstein-Barr virus infection . Splenomegaly Is a
26 characteristic finding of IM, a disease that shares some clinical features (eg, lymphadenopathy and fever) with Kawasaki disease. However, the
conjunctivitis and desquamative rash in this patient, as well as the description of the disease as a vasculitis, are much more congruent with a diagnosis of
27 Kawasaki disease. Also, splenomegaly is not a typical finding in Kawasaki disease.
Infectious mononucleosis Epstein-Barr virus Kawasaki disease Splenomegaly Lymphadenopathy Conjunctivitis Vasculitis Virus Rash Fever Infection
28
29 E i s n ot corr ect. 5% ch ose this.
Although this disease is a vasculitis, it is unlikely to be a source of strokes In the affected pediatric population.
30 • V~c;r~ll tic; PPrli~t rc;

a
Lock
s
Suspend
8
End Bl ock
Item:30of48 ~- , . M k <:] t> al ~· ~
QIO: 1026 .l. ar Previous Next lab 'Vfl1ues Notes Calculator

characteristic finding of IM, a disea se that shares some clinical fea tures (eg, lymphadenopathy and fever) with Kawasaki disea se. However; the
10
conjunctivitis and desquam ative rash in this patient, as well as the description of the disea se as a vasculitis, are much more congruent with a diagnosis of
11 Kawasaki disea se. Also, splenomegaly is not a typical finding in Kawasaki disea se.
Infectious mononucleosis Epstein-Barr virus Kawasaki disease Splenomegaly lymphadenopathy Conjunctivitis Vasculitis Virus Rash Fever Infection
12
E is not correct. 5 % chose this.
13
Although this disea se is a vasculitis, it is unlikely to be a source of strokes in the affected pediatric population .
14 Vasculitis Pediatrics

15
16 Bottom Line:
17 Kawasaki disea se usually m anifests in children under age 4 yea rs. Classically, those affected present with high fever; conjunctival injection,
desquam ating rash, cervical adenopathy, and oral mucositis ("strawberry tongue"). There is a risk of myocardial infarction lat er; as well as a risk of
18 coronary aneurysm.
Kawasaki disease Coronary artery aneurysm Myocardial infarction lymphadenopathy Desquamation Conjunctiva Aneurysm Fever Rash Squamous epithelial cell
19
Conjunctivitis
20
21
22 I ill ;fi 1!1 I•J for year:[ 2017 ..
FI RST AI D FA CTS
23
24
FA17 p 302.1
25
Vasculitides
26 EPIDEMIOLOGY/PRESENTATION PATHOLOGY/lABS
27 Large-vessel vasculitis
28
Giant cell (temporal) Usually elderly females. Most commonly affects branches of carotid
29 arteritis Unilateral headache (temporal artery}, jaw artery.
30 claudication. Focal granulomatous inAammation rJ.
6
lock
s
Suspend
0
End Block
Item: 30 of 48
QIO: 10 26
~

..L
,. Ma rk -<:J
Prev i-ous
I>
Next Lab fJlues
£!1?'

Notes
~~~
Cal culat o r

I \ I
., ~

*
11
12
FA17p271 .1
13
Anatomy of the heart SA and A nodes arc supplied by branches of
14
the RCA. infarct may cause nodal dysfunction
15 Right coronal)' artery'"'"'"'"" left main coronary artery ILCAI
(bradrcardia or heart block).
16 left circumflex coronary Right-dominant circulation (85%) = PO.\ arises
artery ILCXl supplies lateral
17 and posteriOr watts of left from RCA.
18
ventlicte. anterolateral papillary Left-dominant circulation (8%) =PDA arises
muscle
from LCX.
19
r-~_, , - -- left anterior descending Codominant circulation (7%) = PDA arises
20 artery (lAO) supplies anterior 'II from both LCX and RCA.
of interventricular septum.
21 Coronary artery occlusion most commonly
anterolateral papillary muscle,
22 and anterior surface of left ventncle occurs in the LAD.
23 Coronary blood Aow peaks in early diastole.
Right (acute)
Left (obtuse) marginal artery
24 marginal artery
supplies right
25 ventricle

26

27
Posterior descending/interventricular artery (POAl
28 supplies AV node. posterior ''' of Interventricular septum.
29 '1,
posterior waUs of ventricles. and posteromed.al paJllllary muscle

30 •
a
Lock
s
Suspend
8
End Bl ock
Item: 31 of 48 ~ 1 • M k -<:J 1>- Jil ~· !:';-~
QIO: 3826 ..L ar Pre v ious Next Lab~lues Notes Calcula t o r
A A

11 A 32-year-old woman calls her physician complaining of pain with urination and having to urinate every 30 minutes. She reports being sexually active.
The physician diagnoses her as having an uncomplicat ed urinary tract Infection and calls in a prescription for trimethoprim-sulfam ethoxa zole to the
12
pharmacy. A week later the patient presents t o the em ergency department complaining of nausea, vomiting, and fever. She t ells the emergency
13 department physician that her doctor sa id she had a urinary tract infection, but she never picked up the prescript ion beca use she was afraid the medicine
would be too expensive, and one of her friends t old her t he infection would go away on its own. The physician asks f or a urine sample, but the patient says
14 she doesn't think she can give him one, as over t he past day or so she has been making barely any urine. Her temperature is 39.2°C ( 102.6°F), pulse Is
110/ mln, blood pressure is 95/ 60 mm Hg, respiratory rate is 16/ min, and oxygen saturation is 99o/o on room air. A basic m etabolic panel shows a blood urea
15 nitrogen level of 85 mg/ dl and a serum creatinine of 3.6 mg/ dL. Along with starting fluids and antibiot ics, t he physidan ord ers an ECG.
16
17 Which of the following changes is th e physidan ch ecking for on this patient's ECG?

18
19 A. Peaked T waves

20 B. QT prolongation
21
c. ST-segment depression
22
o. T-wave inversion
23
24 E. U waves

25
26
27
28
29
30
. 31 •
a
Lock
s
Suspend
8
End Bl ock
Item: 31 of 48 ~ 1 • M k -<:J 1>- Jil ~· !:';-~
QIO: 3826 ..L ar Pre v ious Next Lab~lues Notes Calcula t o r
A A

11
12 The correct an swer i s A. s o oto chose this.
13 This patient is suffering from acute renal
failure as a result of ascending urinary t ract
14
15
Infection that has most likely reached t he
kidney (In which case, the diagnosis would be -- ___ .. __.,.- ,--..- _,.._ ~ ...,...
pyelonephritrs), or is in the early stages of 1 ., .. \I •
16 sepsis. Either way, this patient's highly '
17
elevated blood urea mtrogen and creatinine,
l t

-
along w ith the patient's report of recent-onset ~
oliguri a, suggest acute rena l fa ilure. Acute •
18 renal fa ilure leads to the inability of th e kidney

to properly secrete potassium in th e urine, \t
19 I I. ,,
20
leading to hyperkalemia. Untreated,
hyperkalemia can lead to ECG changes, most
I
•• , I'
I
notably peaked T waves (shown in the image).
21 It Is extremely important to treat
hyperkalemia in the setting of acute renal \-.........-;.... - .......... ~ ..~ ,.. .._ .... • -+-""\--"'- ~~...! j , . • __.. -- ~... , ... . ... ~ J.,
22 W til IS ' \6
failure In order to prevent ECG changes, which
23 can lead to arrhythmias and even cardiac
I
1:
arrest. This Is why along with mea suring serum
24 potassium, an ECG must always be checked in ~
a patient with acute renal failure and/ or
25
hyperkalemia .
26 Urinary tract infection Hyperkalemia

27 Pyelonephritis Oliguria Blood urea nitrogen Acute kidney injury urea Sepsis Creatinine Urinary system Electrocardiography Urine Potassium Kidney Blood plasma
Cardiac arrest Cardiac arrhythmia Nitrogen Serum {blood) Infection
28
B i s n o t correct. 170/o chose this.
29
Causes of QT prolongation include the long QT syndrome, a rare inborn heart condition in which delayed repolarization of t he heart following a heartbeat
30 Increases the risk of episodes of t orsade de pointes, a form of irregular heartbeat that originates from t he vent ricles. These episod es may lead to
palpitations, fainting, and sudden death due t o vent ricular fibrillation. The so-called early after-depolarizations t hat are seen in long QT syndrome are due
31 • '"" 1'"0 ... 1"\1"\0nl nn 1"\f l_t-uno r~lri11rn rh ~ nn o lc ,.(,,,.-ir"'n ,.h e nl ~t-0 ~11 nh~~o ,...(t-ho,..~,.,.. ~ ~,.. ~,-.t-inn r'u"\t-ont-i~l

a
Lock
s
Suspend
8
End Bl ock
Item: 31 of 48 ~. I • M k <:] t> al ~· ~
QIO: 3826 .l. ar Previous Next lab 'Vfl1 ues Notes Calculator
y p g • g Acute krdney rnJury Urea Sepsrs Creatrnrne Unnary system Electrocardrography Unne Potassrum Krdney Blood plasma
11
Cardiac arrest Cardiac arrhythmia Nitrogen Serum (blood) Infection
12
B is no t co rrect. 1 7% cho se this.
13
Ca uses of QT prolongation include the long QT syndrome, a rare inborn heart condition in which delayed repolarization of the heart following a heartbea t
14 increa ses the risk of episodes of torsade de pointes, a form of irregular heartbea t that originat es from the ventricles. These episodes may lea d to
palpitations, fainting, and sudden dea th due to ventricular fibrillation . The so-called early after -depolarizations that are seen in long QT syndrome are due
15 to re-opening of L-type calcium channels during the plat eau phase of the cardiac action potential.
Ventricular tachycardia Action potential long QT syndrome Ventricular fibrillation Cardiac action potential Palpitations Ventricle (heart) Torsades de pointes
16
Repolarization Cardiac arrhythmia Voltage-dependent calcium channel l-type calcium channel Calcium channel Syncope (medicine) Heart rate Heart sounds
17
Calcium Cavl.l Cardiac cycle
18
c is no t co rrect. 10 % cho se this.
19 ST-segment depressions are most commonly associat ed with acute or chronic myocardial ischemia (eg, unstable angina). In renal failure due to
hypovolemia, myocardial ischemia is a possible manifestation that can occur and therefore it would not be uncommon to find ST-segment depressions on
20 ECG prior to fluid resuscitation . However; this patient's acute renal failure is not due to hypovolemia (as she has not had any blood or fluid loss), but
21 rather is due to ascending infection .
Coronary artery disease Hypovolemia Ischemia Acute kidney injury Electrocardiography Angina pectoris Unstable angina Fluid replacement
22
Cardiopulmonary resuscitation Resuscitation Kidney Infection
23
D is no t co rrect. 10 % cho se this.
24 T-wave inversions are an indication of chronic and/or acute myocardial ischemia. Ischemia may be found if the etiology of the underlying renal failure
were hypovolemia, not acute renal failure due to infection . In addition, in a patient just 32 years old and otherwise healthy, it is highly unlikely that there
25 would be any ischemic cardiac disea se present.
26 Hypovolemia Ischemia Acute kidney injury Coronary artery disease Cardiovascular disease Etiology Myocardial infarction T wave

27 E is no t co rrect. 13 % cho se this.


U waves (the physiologic basis of which are still unknown) can be seen in patients with marked hypokalemia. Hypokalemia can be caused by a number of
28 conditions, but most notably are conditions that result in the overstimulation of the renin-angiotensin-aldosterone axis, such as bilat eral renal artery
29 stenosis. Oversecretion of aldosterone lea ds to increa sed secretion of potassium into the urine, which can result in hypokalemia, which can be seen on
ECG by the presence of U waves.
30 Hypokalemia Renal artery stenosis U wave Aldosterone Electrocardiography Stenosis Urine Potassium

31 •

6
lock
s
Suspend
0
End Block
Item: 31 of 48 ~. I • M k <:] t> al ~· ~
QIO: 3826 .l. ar Previous Next lab 'Vfl1ues Notes Calculator

E is not correct. 13% chose this.


11
U waves (the physiologic basis of which are still unknown) can be seen in patients with marked hypokalemia. Hypokalemia can be caused by a number of
12 conditions, but most notably are conditions that result in the overstimulation of the renin-angiotensin-aldosterone axis, such as bilat eral renal artery
stenosis. Oversecretion of aldosterone lea ds to increa sed secretion of potassium into the urine, which can result in hypokalemia, which can be seen on
13 ECG by the presence of U waves.
14 Hypokalemia Renal artery stenosis U wave Aldosterone Electrocardiography Stenosis Urine Potassium

15
16 Bottom Line:

17 Untrea t ed urinary tract infections can ascend to infect the uret ers and kidneys, which can lea d to acute renal failure. One of the most concerning
fea tures of acute renal failure is hyperkalemia, which left untrea t ed can lea d to cardiac arrhythmias. One sign of hyperkalemia at dangerous levels is the
18 presence of peaked T waves on ECG .
Hyperkalemia Acute kidney injury Urinary system Urinary tract infection Cardiac arrhythmia Kidney Ureter
19
20
21
lijj ;fi IJ l•l for year:l 2017 .. ]
22 FIRST AID FAC T S .

23
24
25 FA17 p 560.2
Electrolyte disturbances
26
27 ELECTROLYTE LOW SERUM CONCENTRATION HIGHSERUM CONCENTRATION

28
Na+ Nausea and malaise, stupor, coma, seizures Irritability, stupor, coma
29 K+ U waves and flattened T waves on ECG, W ide QRS and peaked T waves on ECG,
30
arrhythmias, muscle cramps, spasm, weakness arrh}'thmias, muscle weakness
31
Ca2+ Tetany, seizures, QT prolongation, twitch ing Stones (renal), bones (pain), groans (abdominal

6
lock
s
Suspend
0
End Block
Item: 32 of 48 ~ 1 • M k -<:J 1>- Jil ~· !:';-~
QIO: 5190 ..L ar Pre vious Next Lab~lues Notes Calcula t o r

12
A

A 35-year-old woman presents to her physician with several swollen joints and an erythematous rash over the cheeks. Cardiac examination Is notable
for the sound heard in the audio clip . If her cardiac disease were to progress, she would gradually develop bilat eral crackles at the lung bases and be
lA• A] A

13
unable to sleep lying flat. Alternatively there is a decrease in the volume of the murmur with inspiration. The patient denies any childhood Illness or
14 Illicit drug use.

15 OPEN MEDIA

16
17 oooo 1ooo7 -~
II -
18
19
Which of the following is the most likely cause of her symptoms and would be seen on transesophageal echocardiogram?
20
21
A. Large, friable vegetations of the aortic valve
22
B. Large, friable vegetations of the mitral valve
23
24 C. Large, friable vegetations of the tricuspid valve

25 D. Small, verrucous vegetations of the aortic valve


26
E. Small, verrucous vegetations of the mitral valve
27
F. Small, verrucous vegetations of the tricuspid valve
28
29
30
31
. 32 •
a
Lock
s
Suspend
8
End Bl ock
Item: 32 of 48 ~ 1 • M k -<:J 1>- Jil ~· !:';-~
QIO: 5190 ..L ar Prev ious Next Lab~lues Notes Calculat o r

A
Th e co rrect a nswer i s E. 53% ch ose this. A

12
The audio clip is an example of mitral regurgitation . The murmur is holosystollc with a blowing quality, and
13 It Is heard with an S3 due to left atrial volume overload. The patient In the vignette demonstrates
sequelae of left-sided heart failure due to mitra l regurgitation. She has dyspnea, orthopnea, and bibasilar
14 crackles In her lung. She does not have evidence of right-sided heart failure such as jugular venous
15 distention or peda l edema. Also, the volume of t he murmur decreases with Inspiration. Since right-sided
murmurs Increase w1th inspiration, a left-sided murmur can be inferred If the volume decreases at
16 Inspiration. Given her rash and joint swelling, she likely has a diagnosis of systemic lupus erythematosus
(SLE), and her mitral regurgitation is due t o Ubman-Sacks endocarditis. The gross speamen in the image
17 here shows the verrucous vegetations of t he mitral valve. Most patients with SLE and m1tral regurgitation
are asymptomatic.
18
Orthopnea Systemic lupus erythematosus Dyspnea Endocarditis Mitral insufficiency Mitral valve Heart failure
19 Jugular venous pressure Edema Volume overload Asymptomatic Sequela Regurgitation (circulation)
20 Heart murmur Lung Vein Atrium (heart) Crackles
Image copyright ©2010 Bouma eta/;
21 licensee BioMed Central Ltd. Reproduced
with permission from Dr. s. Gonzalez.
22
23 A is not co rrect. 60/o chose t his.
Large friable vegetations are indicative of infectious endocarditis. The patient has no risk factors for this, and her murmur is inconsistent with aortic valve
24
pathology.
25 Endocarditis Aortic valve Infective endocarditis Pathology Heart murmur

26 B is not co rrect . 2 1 o;o chose this.


Based on the clinical presentation, the patient is unlikely to have infective endocarditis. The most likely cause of her symptoms is libman-Sacks
27
endocarditis, which consists of small, verrucous vegetations composed of fibrin and inflammatory cells.
28 Ubman-Sacks endocarditis Infective endocarditis Fibrin Endocarditis

29 C i s n ot correct. 5% ch ose this.


Large, friable vegetations are suggestive of infective endocarditis. The patient does not have risk factors for this condition, and her symptoms are most
30
likely related to her underlying lupus.
31 Infective endocarditis Endocarditis Lupus erythematosus Systemic lupus erythematosu~

32 D i s not correct. 9% chose t his.



a
Lock
s
Suspend
8
End Bl ock
Item:32of48 ~. , . M k <:] t> al ~· ~
QIO: 5190 .l. ar Previous Next Lab 'Vfll ues Notes Calculator
y y y
12
D is not correct. 9 % chose this.
13 The murmur is one of mitral regurgitation and not aortic regurgitation. Aortic regurgitation is a diastolic murmur. Although Libman-Sacks endocarditis can
14 affect the aortic valve, it is most likely to affect the mitral valve .
Aortic insufficiency Endocarditis Mitral insufficiency Aortic valve Mitral valve Diastole Regurgitation (circulation) Heart murmur
15
F is not correct. 6 % chose this.
16 The tricuspid valve is not likely to be affected since the patient only has symptoms of left -sided heart failure. If she had tricuspid valve regurgitation, she
17 would have pedal edem a and jugular venous distention.
Tricuspid valve Regurgitation (circulation) Jugular venous pressure Edema Vein Jugular vein Heart failure
18
19
Bottom Line:
20
Libman-Sacks endocarditis is seen in a sizable minority of patients with SLE, and it can present as small, verrucous vegetations of the mitral valve
21 causing mitral regurgitation. Verrucous endocarditis is typically asymptomatic.
libman-Sacks endocarditis Endocarditis Mitral insufficiency Mitral valve Asymptomatic Regurgitation (circulation) Systemic lupus erythematosus
22
23
24
lijj ;fi IJ l•l for year:l 2017 ..
25 FI RST AI D FA CTS

26

27 FA17 p443.1
Systemic lupus erythematosus
28

29
SYMPTOMS C lassic presentation: rash, joint pain, and fever, RASH OR PAIN:
most commonly in a female of reproductive Rash (ma lar rJ or discoid 1]1)
30
age and African-American descent. Arthritis (nonerosive)
31 Libman-Sacks Endocarditis-nonbacterial, Serositis
32 verrucous th rombi usually on mitral or aortic Hematologic disorders (eg, cytopenias)
6
lock
s
Suspend
0
End Block
Item: 32 of 48 ~ 1 • M k -<:J 1>- Jil ~· !:';-~
QIO: 5190 ..L ar Pre v ious Next Lab~lues Notes Calcula t o r
A A

12 TREATMENT 'SAl Ds, steroids, immunosuppressants,


13 hydroxych loroquine.
14
15 FA17p279.1

16 Heart murmurs

17 Systolic
18 Aortic stenosis Crescendo-decrescendo systolic ejection murmur (ejection click mar be present).
19
L >>aortic pressure during S)Stole. Loudest at heart base; radiates to carotids.
Sl S2
··Pulsus pan·us et tardus"-pulses are weak with a delayed peak. Can lead to
~I
20
Syncope, \ ngina, and DyspnC<l on exertion (S \D). lost commonly due to agc-
21 rehlted calcification in older patients (> 60 years old) or in younger patients" ith
22 early-onset calcification of bicuspid aortic vah-e.
23 Mitral/tricuspid regurgitation Holosystolic, high-pitched "blowing murmur."
24
Sl S2 Mitral-loudest at apex and rad iates toward axilla. ~IRis often due to ischemic heart
disease (post-M1), MVP, LV dilatation.
~
25
Tricuspid - loudest at tricuspid area. T R commonly caused by RV dilatation.
26
Rheumatic fever and infective endocarditis ca n cause either MR or T R.
27
Mitral valve prolapse Late systolic crescendo murmur with miclsystolic click (MC; due to sudden tensing
28
of chordae tcndineae). ~ l ost frequent vah-ular lesion. Best heard over apex. Loudest
Sl MC S2
29 just before $2. Usually benign. Can predispose to infective endocarditis. Can be
30
31
I Lwml caused by myxomatous degeneration (1° or zoto connecti,·e tissue disease such as
.\tfarfan or Ehlers-Danlos syndrome), rheumatic fe,·er, chordae rupture.
32 •
Ventricular septal defect Holosystolic, harsh-sounding murmur. Loudest at tricuspid area.

a
Lock
s
Suspend
8
End Bl ock
Item: 33 of 48 ~ 1 • M k -<:J 1>- Jil ~· !:';-~
QIO: 3815 ..L ar Pre v ious Next Lab~lues Notes Calcula t o r
A A

13 A 53-year-old man with diabetes presents to the em ergency department complaining of crushing substernal chest pain. He is diaphoretic and has
14 lateral ST elevations on ECG. He is rushed to the cardiac catheterization laboratory, where two coronary artery stents are placed in the circumflex
artery and dual antiplatelet therapy with aspirin and clopidogrel is started. Six days later, when preparing for discharge, the patient suddenly
15 experiences intense chest pain and lighthea dedness. The following values are obtained:

16 Temperature: 37.5°C (99.5°F)


Pulse: 125/ min
17 Blood pressure: 85/ 40 mm Hg
18 Respiratory rate: 18/ min
Oxygen saturation: 97% on room air
19
Auscultation of distinct heart sounds is difficult. The patient's neck veins are prominent. No other physical findings are appredated. After a few minutes the
20 patient loses consciousness.
21
22 What Is the most likely mechanism behind this patient's sudden symptoms?

23
A . Left ventricular free -wall rupture
24
25 B. Papillary muscle rupture

26 c. Pulmonary embolus
27
D. Relnfarction
28
29 E. Right ventricular free-wall rupture

30
31
32
. 33 •
a
Lock
s
Suspend
8
End Bl ock
Item: 33 of 48 ~ 1 • M k -<:J 1>- Jil ~· !:';-~
QIO: 3815 ..L ar Prev ious Next Lab~lues Notes Calculat o r

g p
13
14
15
16 The correct answer is A. 620/o ch ose t his.
17 Five to ten days after myocardial infarction, mechanical weakening of the necrotic and 1nflamed myocardium can result in rupture of the ventricular free
wall, resulting in hemopericardium and cardiac tamponade. The physical manifestations of cardiac tamponade indude distended neck veins, muffled or
18 distant heart sounds, and pulsus paradoxus. As a result of this card iac emergency the patient becomes severely hypotensive, because the blood in the
perica rdium restri cts diastolic filling and subsequently comprom ises overall ca rdiac function (card iogenic shock}.
19 Cardiac tamponade Cardiogenic shock Pericardium Hemopericardium Myocardia• mfarction Pulsus paradoxus Cardiac muscle Diastole Hypotension Heart sounds
20 Nee os·s Infa ction Ventricle "heart) Heart

21 B i s n o t co rrect. 1 2 % chose this.


22 As with ventricular wall rupture, papillary muscle rupture most frequently occurs 4-7 days after acute MI. Rupture of papillary muscles, specifically at the
m itral valve, leads to acute mitral regurgitation and sudden pulmonary edema. The typical presentation includes tachypnea, tachycardia, and hypotension
23 accompanied by diffuse pulmonary rales and respiratory distress. The classic murmur of mitral regurgitation may not be present because of the rapid
pressure equalization between the two chambers. Although this patient shows no signs of decreased cardiac output, his oxygen saturation Is preserved
24 and pulmonary findings are absent.
25 Papillary muscle Tachypnea Pulmonary edema Crackles Cardiac output Tachycardia Hypotension Mitral insufficiency Dyspnea Edema Mitral valve
Regurgitation (circulation) Oxygen saturation Muscle Ventricle (heart) Oxygen Heart murmur
26
C is no t co rrect . 70fo ch ose this.
27
Chest pain (usually pleuritic) can be a symptom of pulmonary embolism. Clues to the diagnosis of massive pulmonary embolus include t achycardia,
28 tachypnea, and hypoxia . Though this patient exhibits t achycardia he is not tachypneic nor hypoxic pointing away from the diagnosis of pulmonary
embolism, additionally distant heart sounds are not associated with PE.
29 Pulmonary embolism Tachypnea Hypoxia (medi cal) Embolism Pleurisy Embolus Heart sounds Chest pain Symptom Tachycardia
30 D i s n ot correct. 6% ch ose this.
31 Although patients certainly are at risk for reinfarction after myocardial Infarction (MI}, it is unlikely in this case. This patient underwent coronary
angiography with stenting of his vessels, at which time the integrity of the vessels as well as the patency of the stents would be checked. In addition,
32 unless it was a massive Ml, one would not expect the acute findings of severe hypotension and unresponsiveness in so short a time. The time frame after
this patient's Ml, given his symptoms, makes ventricular wall rupture more likely.
33 •
a
Lock
s
Suspend
8
End Block
Item: 33 of 48 ~ 1 • M k -<:J 1>- Jil ~· !:';-~
QIO: 3815 ..L ar Prev ious Next Lab~lues Notes Calculat o r

A A

13 Th e co rrect a nswer i s A. 620/o ch ose this.


Five to ten days after myocardial infarction, mechanical weakening of the necrotic and inflamed myocardium can result in rupture of the ventricular free
14 wall, resulting in hemopericardium and cardiac ta mponade. The physical manifestations of cardiac t amponade include distended neck veins, muffled or
15 distant heart sounds, and pulsus paradoxus. As a result of this cardiac emergency the patient becomes severely hypotensive, because the blood In the
pericardium restrrcts diastolic filling and subsequent ly compromises overall cardiac function (cardiogenic shock).
16 Cardiac tamponade Card1ogenic shock Pericardium Hemopericardium Myocardial infarction Pulsus paradoxus cardiac muscle Diastole Hypotension Heart sounds
Nee osis I fa ction Ventricle (heart) Heart
17
18 B is not correct. 12% chose t his.
As with ventricular wall rupture, papillary muscl e rupture most frequently o ccurs 4-7 days after acute MI. Rupture of papillary muscles, specifically at the
19 mitral valve, leads to acute mitral regurgitation and sudden pulmonary edema. The typical presentation includes t achypn ea, tachycardia, and hypotension
20 accompanied by d1ffuse pulmonary ral es and respiratory distress. The classic murmur of mitral regurgitation may not be present because of the rapid
pressure equalization between the two chambers. Although this patient shows no signs of decreased cardiac output, his oxygen saturation Is preserved
21 and pulmonary findings are absent.
Papillary muscle Tachypnea Pulmonary edema Crackles Cardiac output Tachycardia Hypotension Mitral insufficiency Dyspnea Edema Mitral valve
22
Regurgitation (circulation) Oxygen saturation Muscle Ventricle (heart) Oxygen Heart murmur
23
C is not co rrect. 7% chose t his.
24 Chest pain (usually pleuritic) can be a symptom of pulmonary embolism. Clues to the diagnosis of massive pulmonary embolus include tachycardia,
25 tachypnea, and hypoxia. Though this patient exhibits t achycardia he Is not tachypneic nor hypoxic pointing away from the diagnosis of pulmonary
embolism, additionally distant heart sounds are not associat ed with PE.
26 Pulmonary embolism Tachypnea Hypoxia (medical) Embolism Pleurisy Embolus Heart sounds Chest pain Symptom Tachycardia

27 D is not co rrect. 60/o ch ose this.

28 Although patients certainly are at risk for reinfa rction after m yocardial Infarction (MI), it is unlikely in t his case . This patient underwent coronary
angiography with stenting of his vessels, at which t ime t he integrity of the vessels as well as the pat ency of t he stents would be checked. I n addition,
29 unless It was a massive MI, one would not expect t he acute findings of severe hypotension and unresponsiveness in so sho1t a time. The time frame after
this patient's MI, given his symptoms, makes vent ricular wall rupture more likely.
30 Myocardial infarction Coronary catheterization Hypotension Angiography Percutaneous coronary intervention Stent Infarction Coronary stent ventricle (heart)
31 E i s n ot correct. 130/o ch ose this.
32 This patient had an anterior, left-sided myocardial infa rction, as evidenced by ST elevations in the lateral leads on ECG, and by stenting of circumflex
artery that supplies the left vent ricle of t he heart. Alt hough t his patient's symptoms certainly are consistent wit h vent ricular wall rupture, given his
33 • history, wall rupture would be left-sid ed, not riqht-sided .

a
Lock
s
Suspend
8
End Block
Item:33of48 ~. , . M k <:] t> al ~· ~
QIO: 3815 .l. ar Previous Next lab 'Vfl1ues Notes Calculator
unless it was a massive MI, one would not expect the acute findings of severe hypotension and unresponsiveness in so short a time . The time frame after
13 this patient's MI, given his symptoms, makes ventricular wall rupture more likely.
14 Myocardial infarction Coronary catheterization Hypotension Angiography Percutaneous coronary intervention Stent Infarction Coronary stent Ventricle (heart)

15 E is not correct. 13% chose this.


This patient had an anterior; left -sided myocardial infarction, as evidenced by ST elevations in the lateral leads on ECG, and by stenting of circumflex
16 artery that supplies the left ventricle of the heart . Although this patient's symptoms certainly are consistent with ventricular wall rupture, given his
17 history, wall rupture would be left-sided, not right -sided.
Myocardial infarction Ventricle (heart) Electrocardiography Stent ST elevation Percutaneous coronary intervention Anatomical terms of location
18
19
Bottom Line:
20
At 5-10 days after MI, patients are at risk for ventricular wall rupture. The rupture of the ventricular wall causes blood to spill into the pericardium,
21 resulting in tamponade . This most often presents with chest pain, sudden-onset severe hypotension, and unresponsiveness.
Pericardium Hypotension Chest pain Cardiac tamponade Tamponade Ventricle (heart) Ventricular system
22

23
24
25
I ill ;fi 1!1 I•J for year:[ 2017 ..
FI RST AI D FA CTS

26
27 FA17p294.1

28 Evolution of Commonly occluded coronary arteries: LAD > RCA > circu mflex.
29 myocardial infarction Symptoms: diaphoresis, nausea, vomiting, severe retrosternal pain, pain in left arm and/or jaw,
shortness of breath, fat igue.
30
31 TIME GROSS LIGHT MICROSCOPE COMPliCATIONS

32
0-24 hr None Early coagu lative necrosis, Ventricular arrhythmia, HF,
release of necrotic cell cardiogenic shock.
33

6
lock
s
Suspend
0
End Block
16
17
FA17 p 296.1
18 Myocardial infa rction complications
19
Cardiac arrhythmia Occurs'' ithin the first fe,, days arter \II. Important cause of death before reaching the hospital
20 and within the first 24 hours po t-VII.
21 Postinfarction Occurs l-3 days after ~ II . Friction rub.
22 fibrinous pericarditis
23 Papillary muscle Occurs 2-7 days after l\ 11. Po~tcromcdial papillar) muscle rupture rJ t risk due to single blood
24 rupture supply from posterior descending artery. Can result in severe mitral regurgitation.
25 Interventricular septal Occurs 3-5 clays after MI. l'vlacrophagc-mcdiatcd degradation ..... VSD.
26 rupture
27 Ventricular Occurs 3-14 days aft er MI. Contained rrce W<dl rupt ure [l.J; l CO, risk of arrhythmia, embolus
28 pseudoaneurysm from mural thrombus.
formation
29
Ventricular free wall Occurs 5-14 days after MI. Free wa ll rupture ..... cardiac tamponade. L hypertrophy and
30
rupture previous .\11 protect against free \\'all rttpture.
31
True ventricular Occurs 2 \\'eeks to several months after.\ II. Outward bulge with contraction ("dyskinesia"),
32
aneurysm associated with fibrosis.
33 • r. .. I 0 I • • '·. . ,- , . ....

Lock
a s
Suspend
8
End Block
Item: 34 of 48 ~ 1 • M k -<:J 1>- Jil ~· !:';-~
QIO: 1419 ..L ar Pre v ious Next Lab~lues Notes Calcula t o r

14
A

A 57-year-old white man presents to his primary care physician with dyspnea . He says that he likes to maintain his garden, but recently has had
trouble doing the work. He reports shortness of breath when walking up even one flight of stairs. On further questioning, he reports waking up In the
IA•A] A

15
middle of the night with shortness of breath. Physical examination demonstrates pitting ankle edema .
16
17 Which of the following findings would also be expected in this patient?

18
19 A. Decreased sympathetic outflow

20 B. Decreased venous pressure


21
c. Increased aldosterone secretion
22
D. Increased effective arterial blood volume
23
24 E. Increased glomerular filtration rate

25
26
27
28
29
30
31
32
33
o34 •
a
Lock
s
Suspend
8
End Bl ock
Item: 34 of 48 ~ 1 • M k -<:J 1>- Jil ~· !:';-~
QIO: 1419 ..L ar Previous Next Lab~lues Notes Calculat or
A A

14
Th e co rrect a n sw er i s c. 580/o chose this.
15 This patient is presenting with signs typical of congestive heart failure (CHF), which is associated with
16 Interstitial and alveolar edema as shown in the X-ray of the chest. In CHF, there is a decrease in effective
arterial blood volume due to the inability of the heart to effectively pump blood, which stimulates the
17 renin-angiotensin-a ldosterone axis to increase the tubular absorption of Na+ to help increase intravascular
volume.
18 Heart fa1 u1 e Congestive heart failure Edema Chest radiograph X-ray Blood vessel
19
20

21
22

23
24 Image courtesy of James Heilman, MD

25 A is not co rrect. 1 0 0/o chose this.


26 Sympathetic outflow would be increased to counteract the diminished cardiac output of the failing heart. Norepinephrine stimulates 13t -receptors on
myocardial cells to raise the heart rate and increa se contractility so more blood Is ejected with each beat .
27 Norepinephrine Cardiac output Heart rate Contractility Sympathetic nervous system

28 B is no t co rrect . 110/o chose this.


29 The venous pressure would actually be increa sed due to the inability of the heart to effectively pump blood into the arterial system. The blood backs up
and leads to passive congestion of the venous circulation . Increased venous pressure leads to increased intra capillary pressure causing fluid to leak out
30 Into the interstitial tissue space. The transudat e collects in dependent areas and manifests clinically as pitting ankle edema.
Edema Interstitial fluid Circulatory system Vein Transudate Artery
31
D i s n ot correct. 11Ofo ch ose this.
32
The effective arterial blood volume would be decreased due to the decreased ability of the heart to pump blood into the high-pressure arterial circulation.
33 Effective arterial blood volume Circulatory system
34 • F i c nnt rnrrPrt. 1 n otn rhn~~;.p t hi~~;..

a
Lock
s
Suspend
8
End Bl ock
Item:34of48 ~. , . M k <:] t> al ~· ~
QIO: 1419 .l. ar Previous Next Lab 'Vfll ues Notes Calculator

• Effective arterial blood volume Circulatory system


14
E is not correct. 10% chose this.
15
Glomerular filtration rate would be decreased because the ineffective pump leads to lowered effective arterial blood volume, resulting in a reduction in
16 renal blood flow . Although aldosterone -mediated angiotensin II secretion will increase and cause constriction of the efferent arteriole, GFR overall is
reduced due to decreased renal perfusion.
17 Efferent arteriole Arteriole Renal function Angiotensin II Angiotensin Blood volume Glomerulus Renal blood flow Kidney Perfusion Blood flow
18
19
Bottom Line:
20
The renin-angiotensin-aldosterone system and sympathetic outflow are stimulated in CHF secondary to a decreased effective circulating intravascular
21 volume .
Swiss franc
22
23
24
I iii I;fi 1!1 I•J for year:l 2017 ..
25 FI RST AI D FA CTS

26
FA17 p298.1
27
28
Heart failure C li nical syndrome of cardiac pump dysfunction - congestion and low perfusion. Symptoms
include dyspnea, orthopnea, fatigue; signs include S3 heart sound, rales, jugular venous distention
29
()VD), pitti ng edema · .
30 Systolic dysfunction - reduced EF, t EDV; ! contractility often zoto ischemiafM I or dilated
31 cardiomyopathy.
32 Diastolic dysfunction- presen ·ed EF, normal EDV; ! compliance often zoto myocardial
33 hypertrophy.
Right HF most often results from left HF. Cor pulmonale refers to isolated right HF due to
34 • '
6
lock
s
Suspend
0
End Block
14
15
Hepatomegaly t centra I venous pressure -+ t resista nee to
(nutmeg liver) portal Aow. Rarely, leads to "cardiac cirrhosis."
16
Jugular venous t ,-enous pressure.
17
distention
18
Peripheral edema t venous pressure - Auid transudation.
19
20
FA17 p 558.1
21
Renin-angiotensin-aldosterone system
22
23 I BP (JG cells)
24 I Na' delivery Acts at angiotensin II
- -+ Vasoconstriction

,
t BP
(macula densa cells) receptor. type 1 (AT 1) on
25
t sympathetic tone vascular smooth muscle
26 (~1 -receptors)

27 J Constricts efferent t FF to preserve renal function (GFR)


arteriole of glomerulus in low-volume states (ie, when RBF ~ )
28
29 Renin Ai E ----+ Bradykinin Aldosterone
'$> breakdown
t Na' channel insertion and - -+ Creates
(adrenal cortex)
30
Angiotensinogen
? t activity of Na' /K' pump;
enhances K" and H-
favorable Na·
gradient for
31 excretion by way of principal Na• and Hp
32 cell K' channels and reabsorption
a-intercalated cell H' ATPases
33
34 •
a
Lock
s
Suspend
8
End Block
Item: 34 of 48 ~ 1 • M k -<:J 1>- Jil ~· !:';-~
QIO: 1419 ..L ar Pre v ious Next Lab~lues Notes Calcula t o r
A A

14
15 Renin Secreted by JG cells in response to I renal arteria l pressure, t renal sympathetic discharge (~ 1
effect}, and l Na- delivery to macu la densa cells.
16
17
AT II Helps maintain blood ,oJume and blood pressure. Affects baroreceptor function; limits reOex
bradycardia, which \\'Ould normally accompany its pressor effects.
18
ANP, BNP Released from atria (ANP) and 'entricles (Bi\ P) in response to t volume; may act as a ··check"
19
on renin-angiotensin-aldosterone system; re)a,es vascular smooth muscle via cG\1P - t G FR,
20
I renin. Dilates afferent arteriole, constricts efferent arteriole, promotes natriuresis.
21
ADH Primarily regulates osmolarity; also responds to lo\\' blood ,-olume states.
22
Aldosterone Primaril~·
regulates ECF volume and • a• content; responds to low blood \'Oiume states. Responds to
23 hyperkalemia by t K+ excretion.
24
25
FA17 p 558 .2
26
Juxtaglomerular Consists of mesangial cells, JG cells (modified JGA maintains CFR via renin-angiotensin-
27
apparatus smooth muscle of afferent <Hteriolc) and the aldosterone system.
28 macula densa (NaCI sensor, part of DCT). ~-blockers can decrease BP by inhibiting
29 JG cells secrete renin in response to I rena l ~ 1 -rccept ors of the JGA - I renin release.
30 blood pressure and t sympathetic tone (~ 1 ).
Macula densa cells sense I aCI delivery to
31
OCT - t renin release - efferent arteriole
32
vasoconstriction - t GFR.
33
34 •
a
Lock
s
Suspend
8
End Bl ock
Item: 35 of 48 ~ 1 • M k -<:J 1>- Jil ~· !:';-~
QIO: 1431 ..L ar Pre v ious Next Lab~lues Notes Calcula t o r

15
A

A 3-year-old girl is brought to the pediatrician by her mother because of a 7-day history of fever that does not respond to acet aminophen. On physical
examination, the patient is found to be febrile with injected conjunctivae, edematous hands and feet, cracked and swollen lips, and desquamation of
lA• A] A

16
the skin on her fingertips.
17
18 What complication is most likely to be seen in this individual?
19
20 A. Coronary aneurysm

21 B. Hepatic dysfunction
22
c. Hepatitis B
23
24
o. Perforated nasal septum

25 E. Renal failure
26
27
28
29
30
31
32
33
34
o35 •
a
Lock
s
Suspend
8
End Bl ock
Item: 35 of 48 ~ 1 • M k -<:J 1>- Jil ~· !:';-~
QIO: 1431 ..L ar Prev ious Next Lab~lues Notes Calculat or
A A

15
Th e correct a n sw er i s A. 6 4 0/o ch ose this .
16 This Individual most likely is suffering from Ka wasa ki disea se, an acute necrotizing vasculitis of small- and
medium-sized vessels. I n severely affected vessels, t he m edia develops Inflammation, with necrosis of
17
smooth muscle cells. The internal and external elastic lamina e can split, leading to aneurysms (as seen in
18 the Image). Over the next few weeks t o m ont hs, t he active inflammatory cells are replaced by fibroblasts
and monocytes, and fibrous connective t issue begins t o form within the vessel wall. The int1ma prolifera tes
19 and thickens. The vessel wall event ually becomes narrowed or occluded, owing to stenosis or a thrombus.
Cardiovascular death may occur from a myoca rdial infarction secondary to thromboSIS of a coronary
20 aneurysm or from rupture of a larg e coronary a neurysm. Clinically, Kawasaki disease is an acute, self-
21 limiting Illness of infants and y oung childre n, ofte n of Asian descent. Signs and symptoms include fever,
limbic-sparing conjunctivitis, mucositis, vesicles or ulce rs of the lips and oral mucosa, and ly mphadenitis.
22 Coronary aneurysms (as seen in th e image ), may occur in nearly 25% of cases; therefore, any child
suspected of having Kawasaki disease should have a n echocardiogram performed. Medical managem ent of
23 Kawasaki disease primarily includes intrave nous y -globulin, as well as high-dose aspirin.
Kawasaki disease Coronary artery aneurysm Myocardial infarction Conjunctivitis Echo .ardiography Thrombus
24
vasculitis Necrosis Stenosis Oral mucosa Aspirin Connective tissue Inflammation Lymphadenopathy Thrombosis
25
Systemic vasculitis Tunica intima Intravenous therapy Fibroblast Monocyte Mucous membrane Aneurysm Fever
26 Circulatory system Vertebra Smooth muscle tissue Peptic ulcer Muscle
27
Image courtesy of Wikimedia Commons
28
B is no t co rrect. 120/o c hose this .
29
Hepatic dysfunction m ay occur from m any causes, including Reye syndrome, which Is an encephalopat hy associat ed wit h fatty degeneration of the liver
30 and delirium. Reye syndrome occurs in children who are given salicylates such as aspirin during a viral illness. Hepat ic dysfunction is not a characteristic
finding In Kawasaki disease .
31
Kawasaki disease Reye syndrome Aspirin Salicylic acid Fatty liver Uver Encephalopathy Steatosis Delirium Virus
32
C i s n o t co rrect. 50fo ch ose this .
33 Up to 30% of patients with polyarteritis nodosa test positive for hepatitis B surface antigen. Hepatitis B typica lly is not associat ed with Kawasaki disease.
Kawasaki d sease Polyarteritis nodosa Hepatitis B Antigen Hepatitis
34
0 i s not correct. 50/o chose t his.
35 •
a
Lock
s
Suspend
8
End Bl ock
Item:35of48 ~- , . M k <:] t> al ~· ~
QIO: 1431 .l. ar Previous Next lab 'Vfllues Notes Calculator

15 D is not correct. 5 % chose this.


16 A perforat ed nasal septum, a complication often associat ed with granulomatosis with polyangiitis (Wegener ), is not known to be associat ed with Kawasaki
disea se.
17 Granulomatosis with polyangiitis Kawasaki disease Nasal septum perforation Nasal septum
18 E is not correct. 14% chose this.
19 Renal failure is a complication of m any disea se processes, such as systemic lupus erythem atosus, gra nulomatosis with polyangiitis (Wegener ), and
polyarteritis nodosa, but not Kawasaki disea se.
20 Granulomatosis with polyangiitis Kawasaki disease Polyarteritis nodosa Systemic lupus erythematosus Kidney lupus erythematosus
21
22
Bottom Line:
23 Signs and symptoms of Kawasaki disea se include fever; congested conjunctivae, changes in the lips and oral mucosa, and lymphadenitis. Given their
24 increa sed likelihood of developing a coronary aneurysm, children suspected of having Kawasaki disea se should undergo echocardiography.
Kawasaki disease Coronary artery aneurysm Echocardiography Oral mucosa lymphadenopathy Aneurysm Mucous membrane Conjunctiva
25
26

27
I iii I;fi 1!1 I•J for year:l 2017 ..
FI RST AI D FA CTS
28

29
FA17 p 302.1
30
Vasculitides
31
EPIDEMIOLOGY/PRESENTATION PATHOLOGY/lABS
32
Large-vessel vasculitis
33
Giant cell (temporal) Usually elderly females. Most commonly affects branches of carotid
34 arteritis Unilateral headache (temporal artery}, jaw artery.
35 claudication. Focal r!ranulomatous inAammation D .
6
lock
s
Suspend
0
End Block
Item: 36 of 48 ~ 1 • M k -<:J 1>- Jil ~· !:';-~
QIO: 5 0 35 ..L ar Pre v ious Next Lab~lues Notes Calcula t o r
A A

16 A 59-year-old man is hospitalized after suffering a severe myocardial Infarction. He Is Initially treated with nitrates, 13-blockers, and aspirin, and
subsequently undergoes cardiac catheterization with placement of two stents. Following the procedure he is hemodynamically stable without
17
recurrence of chest pain. However, 5 days after admission his heart rate Is 134/min, blood pressure is 72/35 mm Hg, and respiratory rate is 29/mln.
18 Physical examination reveals distant heart sounds and an elevated jugular venous pressure.

19
Which of the following complications is most likely causing this patient's symptoms?
20
21
A. Aneurysm formation
22
B. Cardiac arrhythmia
23
24 c. Fibrinous pericarditis

25 o. Rupture of the papillary muscle


26
E. Rupture of the ventricular free wall
27
28
29
30
31
32
33
34
35
. 36 •
a
Lock
s
Suspend
8
End Bl ock
Item: 36 of 48 ~ 1 • M k -<:J 1>- Jil ~· !:';-~
QIO: 5 0 35 ..L ar Prev ious Next Lab~lues Notes Calculat o r

A A

16
17 Th e correct an swer i s E. 600/o ch ose this.
18 This patient is in shock due to cardiac tamponade secondary to the rupture of the ventricular free wall, which usually occurs 3-1 4 days after the Initial
myocardial Infarction (MI). It usually presents wit h sudden shock with signs of cardiac tamponade, such as the hypotension, distant heart sounds, and
19 elevated Jugular venous pressure observed in this patient. Survival depends primarily on the rapid recognition of myocardial rupture and the provision of
Immediate therapy, such as urgent pericardiocentesis or surgery. unfortunately, this complication of an MI is often a fata l one.
20 Cardiac tamponade Myocardia infarction Pericardiocentesis Jugular venous pressUI e Hypote1 sion Myocardial rupture Heart sounds Tamponade Jugular vein
21 Vento rcle eart· rein

22 A is not correct. 5% chose this.


23 Aneurysm formation may occur post-MI due to a lack of contractility that results in the infarcted area. Sequelae of ventricular aneurysm formation Include
decreased cardiac output, increased risk of arrhythmias, and potential mural thrombus formation with risk of embolization.
24 "'"hrombus Ca diac output Aneurysm Cardiac arrhythmia Embolization Infarction Ventricular aneurysm Sequela Ventricular system

25 B is not co rrect. 6% chose this.


26 Cardiac arrhythmias are a common cause of death in the first few days after an MI, especially within the first 24 hours. However, this patient's clinical
picture Is more consistent with a diagnosis of cardiac tamponade secondary to ventricular free wall rupture.
27 Cardiac tamponade Cardiac arrhythmia Tamponade Ventricle {heart) ventricular system

28 C is not co rrect. 160/o chose this.


29 Fibrinous pericarditis is a complication that usually occurs 1-3 days post-MI. This condition usually presents with chest pain that is relieved when the
patient leans forward; often a pericardia! friction rub can be auscultated on cardiac examination. However; this patient's clinical picture is more consistent
30 with a diagnosis of cardiac tamponade secondary to ventricular free wall rupture.
Cardiac tamponade Uremic pericarditis Per icar ditis Per icardia! friction rub Cardiac examination Chest pain Auscultation Per icardium Fibrin ventricle (heart)
31
D i s n ot co rrect . 130/o ch ose this.
32
Papillary muscle rupture is another complication of MI that usually occurs 3-14 days post-MI. However, patients with papillary muscle rupture usually
33 present with acute mitral regurgitation, causing the abrupt onset of heart failure in the setting of a new systolic murmur. Emergent surgical Intervention is
the treatment of choice for this condition.
34 Papillary muscle Mitral insufficiency Systole Regurgitation (circulation) Systolic heart murmur Heart failure Muscle Heart murmur
35
36 •
a
Lock
s
Suspend
8
End Bl ock
Item:36of48 ~. , . M k <:] t> al ~· ~
QIO: 5035 .l. ar Previous Next lab 'Vfl1 ues Notes Calculator

16
17 Bottom Line:
Free wall rupture is a complication that occurs 3 -14 days after an MI and can result in cardiac t amponade. Cardiac t amponade presents with
18
hypotension, distant heart sounds, and elevated jugular venous pressure.
19 Cardiac tamponade Jugular venous pressure Hypotension Heart sounds Jugular vein Vein Tamponade

20

21
22 lijl;fiiJI•l toryear:[2017 • ]
FIRST AID FACTS
23
24 FA17 p296.1
25 Myocardial infarction complications
26 Cardiac arrhythmia Occurs within the first few days after MI. Important cause of death before reaching the hospital
27 and within the first 24 hours post-MI.
28 Postinfarction Occurs 1-3 clays after MI. Friction rub.
fibrinous pericarditis
29
30 Papillary muscle Occurs 2-7 days after MI. Posteromedial papillary muscle rupture rJ t risk due to single blood
rupture supply from posterior descending artery. Can result in severe mitral regurgitation.
31
Interventricular septal Occurs 3-5 clays after MI. Macrophage-mediated degradation ..... VSD.
32
rupture
33
34
Ventricular Occurs 3-14 clays after MI. Contained free wall rupture rn;l CO, risk of arrhythmia, embolus
pseudoaneurysm from mural thrombus.
35 formation
36 ("') ,..,,.., .. ...,. t: 14 ,.1,. •.,. ,.c,,. .. l\Af 1?..,..,.. •••,.. 11 ... . ..."'"'' ..,.. r:=ll - , .....,1; ..,.,., .......,..., •.,,.....,_.,.,1,.. I \ / 1.,. . ...... ,.. .......,.. .... 1,., . ..... .,...,1

6
lock
s
Suspend
0
End Block
Item:36of48 ~. , . M k <:] t> al ~· ~
QIO: 5035 .l. ar Previous Next lab 'Vfl1 ues Notes Calculator

16 Fibrinous pericarditis is a complication that usually occurs 1-3 days post -MI. This condition usually presents with chest pain that is relieved when the
patient leans forward ; often a pericardiaI friction rub can be auscultated on cardiac examination . However; this patient's clinical picture is more consistent
17 with a diagnosis of cardiac t amponade secondary to ventricular free wall rupture.
Cardiac tamponade Uremic pericarditis Pericarditis Pericardia! friction rub Cardiac examination Chest pain Auscultation Pericardium Fibrin Ventricle (heart)
18
D is not correct. 1 3% chose this.
19
Papillary muscle rupture is another complication of MI that usually occurs 3 -14 days post-MI. However; patients with papillary muscle rupture usually
20 present with acute mitral regurgitation, causing the abrupt onset of heart failure in the setting of a new systolic murmur. Emergent surgical intervention is
the treatment of choice for this condition.
21 Papillary muscle Mitral insufficiency Systole Regurgitation (circulation) Systolic heart murmur Heart failure Muscle Heart murmur
22

23
Botto m Line:
24 Free wall rupture is a complication that occurs 3 -14 days after an MI and can result in cardiac tamponade. Cardiac tamponade presents with
25 hypotension, distant heart sounds, and elevated jugular venous pressure.
Cardiac tamponade Jugular venous pressure Hypotension Heart sounds Jugular vein Vein Tamponade
26
27
28
lijl;fiiJI•l to r year:[ 20 17 • ]
FI RST AID FAC T S
29
30
FA17 p296.1
31
Myocardial infarction complications
32
Cardiac arrhythmia Occurs within the first few days after MI. Important cause of death before reaching the hospital
33 and within the first 24 hours post-MI.
34
Postinfarction Occurs 1-3 clays after MI. Friction rub.
35 fibrinous pericarditis
36 P::>nill::>rv "" ocrl<>

6
lock
s
Suspend
0
End Block
Item: 36 of 48 ~ 1 • M k -<:J 1>- Jil ~· !:';-~
QIO: 5 0 35 ..L ar Pre v ious Next Lab~lues Notes Calcula t o r
A
- - A

16
17
FA17 p 294.1
18
Evolution of Commonly occluded coronar) arteries: I.AD > RCA > circumAex.
19 myocardial infarction S~ mploms: diaphoresis, nausea, \ Omiting, Se\ ere retrosternal pain, pain in left arm and/or ja\\,
20 shortness of breath, fatigue.
21 TIME GROSS LIGHT MICROSCOPE COMPliCATIONS
22 0-24 hr 'one Early coagulati' e necrosis, entricular arrhythmia, II F',
23 release of necrotic cell cardiogenic shock.
24 contents into blood; edema,
hemorrhage. wa\'y fibers.
25 Occluded eutrophils appear.
artety
26 Rcpcrfusion injury,
27 associated "ith generalion
of free radicals, leads to
28
··--'!~~ Dark mottling. hypercontraction of myofibrils
29 pale with through t rrce calcium in Aux.
30
31
32
tetrazolium
stain
~
0

Q -
- - Q
...
~
33
34
35
_.

- -
~ ~

36 1-3 days Extensi' e coagulati,·e necrosis. Postinfarction fibrinous



~··
a
Lock
s
Suspend
8
End Bl ock
Item: 37 of 48 ~ 1 • M k -<:J 1>- Jil ~· !:';-~
QIO: 1427 ..L ar Pre v ious Next Lab~lues Notes Calcula t o r
A A

17 A 16-year-old Japanese exchange student presents to the physician with a history of fevers, night sweats, and muscle pain. o n physical examination,
the patient has extremely weak pulses in her upper extremities.
18
19
Laboratory abnorma lities in which of the following parameters is most likely?
20
21
A. Anti-lgG antibodies
22
B. Antinuclear antibodies
23
24 c. Cytoplasmic antineutrophil cytoplasmic antibody

25 D. Erythrocyte sedimentation rate


26
E. Perinuclear antineutrophil cytoplasmic antibody
27
28
29
30
31
32
33
34
35
36
. 37 •
a
Lock
s
Suspend
8
End Bl ock
Item: 37 of 48 ~ 1 • M k -<:J 1>- Jil ~· !:';-~
QIO: 1427 ..L ar Previous Next Lab~lues Notes Calculat or
A E. Perinuclear antineutrophil cytoplasmic antibody A

17
18
19
20 The correct answer is o. 600/o chose t his.
21 This Individual most likely is suffering from Takayasu arteritis, also known as " pulseless disease. • This disease typically affects medium and large arteries,
resulting In thickening of the aortic arch and/ or proximal great vessels. Symptoms Include fevers, arthritis, night sweat s, myalgias, skin nodules, ocular
22 d isturbances, and weak pulses in the upper extremities. It is most common In young Asian females and is assodated w ith an elevated erythrocyte
sed imentation rate. As w ith g iant cell arteritis, granulomatous changes w ithin the wa ll s of affected large vessels can be seen on biopsy.
23 Ta ayas arteritis Giant-eel arteritis Erythrocyte sedimentation rate Red blood ce Biopsy Granuloma Artery Arthritis Night sweats Anatomoca terms of location
24 Arte itis
25 A is not correct. 7 % chose this.
26 AntHgG anti bod ies, more commonly known as rheumatoid factors, are associated with rheumatoid arthritis (RA). RA manifests as a symmetric
Inflammatory polyarthritis classically associated with morning stiffness, rheumato id nodules, and other features.
27 Rheumatoid arthritis Polyarthritis Rheumatoid nodule Antibody Arthritis Joint stiffness Nodule (medicine)
28 B is not co rrect. 9% chose t his.
29 Antinuclear antibodies are associated with, but not specific for, systemic lupus erythematosus (SLE). SLE is common among young to middle-aged women,
and a classic pattern of rashes (including a m alar rash or a discoid rash), renal disease, and other m anifestations would be expected .
30 Systemic lupus erythematosus Malar rash Anti -nuclear antibody Antibody Kidney disease Lupus erythematosus Rash Cheek Kidney
31 C is not co rrect. 120/o ch ose this.
32 Classical antlneutrophil cytoplasmic antibody is associat ed with small-vessel vasculitides, including granulomatosis with polyangiitis, which Is characterized
by pulmonary and renal focal necrotizing granulomatous vasculitis.
33 Granulomatosis with polyangiitis Anti-neutrophil cytoplasmic antibody vasculitis Antibody Granuloma Necrosis Cytoplasm Kidney
34 E i s n ot correct. 120/o ch ose this.
35 Perinuclear antineutrophil cytoplasmic antibody commonly is associated with small-vessel vasculitides, including microscopic polyangiitis and Churg-Strauss
syndrome. Patients with either condition would not present with absent pulses.
36 Churg-Strauss syndrome Microscopic polyangiitis Anti-neutrophil cytoplasmic antibody Vasculitis Antibody Cytoplasm
37 •
a
Lock
s
Suspend
8
End Bl ock
Item:37of48 ~- , . M k <:] t> al ~· ~
QIO: 1427 .l. ar Previous Next lab 'Vfl1ues Notes Calculator
• Granulomatosis with polyangiitis Anti -neutrophil cytoplasmic antibody Vasculitis Antibody Granuloma Necrosis Cytoplasm Kidney
17
18 E is not correct. 12% chose this.
Perinuclear antineutrophil cytoplasmic antibody commonly is associated with small-vessel vasculitides, including microscopic polyangiitis and Churg-Strauss
19 syndrome. Patients with either condition would not present with absent pulses.
20 Churg-Strauss syndrome Microscopic poly angiitis Anti -neutrophil cytoplasmic antibody Vasculitis Antibody Cytoplasm

21
22 Bottom Line:

23 Takayasu arteritis is a vasculitis that presents clinically with loss of pulse and symptoms such as fever; myalgia, and night sweats.
Takayasu' s arteritis Myalgia Vasculitis Night sweats Arteritis Fever Pulse
24
25
26
l i j l ; f i i J I • l foryear:[ 2017 • ]
27 FI RST AI D FA CTS

28
FA17 p 302.1
29
Vasculitides
30
EPIDEMIOLOGY/PRESENTATION PATHOlOGY/lABS
31
Large-vessel vasculitis
32
Giant cell (temporal) Usually elderly females. Most commonly affects branches of carotid
33
arteritis Unilateral headache (temporal artery}, jaw artery.
34 claudication. Focal granulomatous inAammation rJ.
35 May lead to irreversible blindness due to t ESR.
36 ophthalmic artery occlusion. Treat with high-dose corticosteroids prior to
37
Associated with polymyalgia rheumatica. tempore.! artery biopsy to prevent blindness.

6
lock
s
Suspend
0
End Block
Item: 38 of 48 ~ 1 • M k -<:J 1>- Jil ~· !:';-~
QIO: 1009 ..L ar Pre v ious Next Lab~lues Notes Calcula t o r

18
A

A 56-year-old man with hypertension, hyperlipidemia, and type 2 diabetes mellitus presents to his primary care physician with complaints of chest
pain. He reports that he intermittently has chest pain when walking up several flights of stairs or helping his wife m ove stones in the garden. The pain
IA•A] A

19
goes away when he takes a break from such activities.
20
21 What percentage of arterial stenosis is suspected in t his patient?

22
23 A. 10%

24 B. 100%
25
c. 45%
26
27
o. 60%

28 E. 75%

29 F. 90%
30
31
32
33
34
35
36
37
. 38

a
Lock
s
Suspend
8
End Bl ock
Item:38of48 ~. , . M k <:] t> al ~· ~
QIO: 1009 .l. ar Previous Next lab 'Vfl1ues Notes Calculator

18
19
The co rrect a nswer is E. 63% cho se this.
20
Luminal narrowing > 70% causes cardiac ischemia without cell dea th, known as stable angina. Acute angina is usually trea t ed with nitroglycerin pills.
21 However; stable angina is usually managed by lifestyle changes, control of LDL cholesterol, antiplat elet therapy in addition to revascularization procedures
( eg, stenting) in select patients.
22 Nitroglycerin Ischemia Angina pectoris Revascularization lumen (anatomy) low-density lipoprotein Antiplatelet drug Percutaneous coronary intervention Stent
23 Cholesterol Coronary stent Apoptosis Stable angina

24 A is no t co rrect. 3 % cho se this.


25 10% stenosis is not usually sufficient to cause any symptoms of heart disea se and may be an incidental finding in healthy adults. Although mild coronary
artery stenosis may reduce the cardiac blood flow to the mycordiac tissue supplied by the artery, the myocardium can still compensat e by increa sing the
26 oxygen extraction fraction .
Cardiac muscle Stenosis Cardiovascular disease Oxygen Coronary circulation
27
B is no t co rrect. 1 Ofo cho se this.
28
Complet e occlusion of a coronary artery or 100% stenosis lea ds to myocardial infarction (MI ) and not to stable angina. A sudden occlusion of a cornoary
29 artery (acute coronary syndrome) is a medical emergency that requires revascularization of ischemic tissue to prevent extensive necrosis and loss of
myocardial tissue. Symptoms of an acute coronoary syndome include sudden -onset severe pressure -like chest pain associat ed with diaphoresis and
30 shortness of brea th . Symptoms do not necessarily develop after exertion . Patients with myocardial infarction are at risk of post -MI complications, including
31 heart failure, arrythmias, left ventricular wall rupture, pericarditis, among others. Arrythmias, however; are the most frequent cause of dea th after MI.
Acute coronary syndrome Myocardial infarction Perspiration Pericarditis Stenosis Angina pectoris Ischemia Dyspnea Chest pain Revascularization
32 Medical emergency Necrosis Heart failure Coronary circulation Heart Ventricle (heart) Vascular occlusion Infarction
33
c is no t co rrect. 9 % cho se this.
34 45% stenosis is not usually sufficient to cause stable angina. Patients with moderate coronary artery stenosis can still be asymptomatic; however; they
are at risk of progression to more severe levels of stenosis or rupture lea ding to acute coronoray syndrome. Progression is usually prevented by lifestyle
35 changes and control of cardiovascular risk factors including diet, smoking, and physical activity. Patients usually do not require additional interventions.
36 Stenosis Angina pectoris Asymptomatic Stable angina Circulatory system Physical exercise

37 D is no t co rrect. 15 % cho se this.


60% stenosis is usually not sufficient to cause stable angina. Less than 70% occlusion of a coronary vessel is still considered moderat e coronary artery
38 • rlic::.o;::a c::.o ;::a nrl m ;:nt nnt l.o;::a rl tn C::\lmntnmc:: nf c::t;::a hl.o ;::a nnin;::a Drnnn:lo c::c:inn tn mnr·.o C::.O\t.or·.o d".onnc:k k IIC:II ;::a lht nn~ \t.o nt .o rl h\t lif.oc::t\1l.o r h ;::a nn10c:: ;::a nrl r nntn'\l nf

6
lock
s
Suspend
0
End Block
Item:38of48 ~. , . M k <:] t> al ~· ~
QIO: 1009 .l. ar Previous Next lab 'Vfl1ues Notes Calculator

18 45% stenosis is not usually sufficient to cause stable angina. Patients with moderate coronary artery stenosis can still be asymptomatic; however; they
are at risk of progression to more severe levels of stenosis or rupture lea ding to acute coronoray syndrome. Progression is usually prevented by lifestyle
19 changes and control of cardiovascular risk factors including diet, smoking, and physical activity. Patients usually do not require additional interventions.
Stenosis Angina pectoris Asymptomatic Stable angina Circulatory system Physical exercise
20
D is not correct. 15% chose this .
21
60% stenosis is usually not sufficient to cause stable angina. Less than 70% occlusion of a coronary vessel is still considered moderat e coronary artery
22 disea se and may not lea d to symptoms of stable angina. Progression to more severe stenosis is usually prevented by lifestyle changes and control of
cardiovascular risk factors including diet, smoking, and physical activity. Patients at the upper end of moderate coronary artery disea se (> 50% stenosis)
23 are evalauted for further work-up depending on symptoms. Notably, higher level stenosis is not associat ed with increa sed risk of plaque rupture. In fact,
24 acute coronary syndrome commonly occurs after rupture of plaques that cause less than 50% occlusion.
Acute coronary syndrome Coronary artery disease Angina pectoris Stenosis Stable angina Circulatory system Coronary circulation Vascular occlusion
25
F is not correct. 9 % chose this .
26 Stable angina typically begins to present with a narrowing of 75%, not 90% .
27 Angina pectoris Stable angina

28
29 Bottom Line:
30 Stable angina typically becomes clinically observable when coronary artery stenosis rea ches 75% .
Stenosis Angina pectoris Stable angina Coronary circulation
31
32
33
lijj ;fi IJ l•l for year:l 2017 ..
34 FIRST AID FAC T S

35
FA17 p 293.2
36
Ischemic heart disease manifestations
37
38
Angina C hest pain due to ischemic myocardium zoto coronary artery narrowing or spasm; no myocyte
6
lock
s
Suspend
0
End Block
19
20
FA17 p 292.1
21
Atherosclerosis Very common. Disease of clastic arteries and large- and medium-sized muscular arteries; a form of
22
arteriosclerosis caused by buildup of cholesterol plaques.
23
LOCATION Abdominal aorta > coronary artery > popliteal arter) > carotid artery fJ.
24
25
26
27
28
29
30
31 RISKFACTORS lodifiable: smoking, hypertension, hyperlipidemia (f LDL}, diabetes.
32
onmodifiable: age, sex (f in men and postmenopausal women), fami ly history.

33
SYMPTOMS Angina, claudication, but can be asymptomatic.
34 PROGRESSION lnAammation important in pathogenesis: endothelial cell dysfun ction .... macrophage and LDL
35
accumulation .... foam cell fonmttion .... fa tly slreal..s .... smooth muscle cell migration (in\'olves
PDGF and FGF}, proliferation, and extracellular matrix deposition .... fibrou s plaque .... complex
36
at·heromas Ill
37
38 •
a
Lock
s
Suspend
8
End Block
23 COMPLICATIONS Aneurysms, ischemia, infarcts, peripheral ,·ascular disease, thrombus, emboli.
24
25 FA17 p 305.2
26 Antianginal therapy Coal is reduction of myocardial 0 2 consumption ( ~ IV02) by I I or more of the determinants of
27 .\1V02: end-diastolic volume, BP, IIR, contractilil).
28 COMPONENT NITRATES j3 BLOCKERS NITRATES+ j3-8LOCKERS
29 o effect or t
30
End-diastolic volume
' 'o effect or I

31
Blood pressure
' ' 'Little/no effect
32
Contractility No effect
t (reAex response) ' o effect or I
33
Heart rate
Ejection time ' Little/no effect
34
' t
u
35
36
MV02
'
erapamil is similar to ~-blockers in effect. '
Pindolol and acebutolol are partial ~-agonists that should be used with caution in angina.
37
38 •
a
Lock
s
Suspend
8
End Block
Item: 39 of 48 ~ 1 • M k -<:J 1>- Jil ~· !:';-~
QIO: 1422 ..L ar Pre v ious Next Lab~lues Notes Calcula t o r

19
A

A tall, thin 25-year-old white m an with a history of joint hyperflexibility presents to the emergency department with a chief complaint of tearing chest
pain that radiates to his back. His blood pressure is 90/45 mm Hg, his pulse Is 123/min, and his respiratory rat e is 24/min. on physical examination,
lA• A] A

20
he Is found to have a difference of 21 mm Hg in his systolic blood pressure between Inspiration and expiration, accompanied by distended neck veins
21 and distant heart sounds.

22
Which of the following ECG findings would be most characteristic for this patient's pathophysiologic process?
23
24
A. 1.5-mm ST-segment elevations in anterior leads
25
B. Diffuse ST-segment elevations and PR depressions
26
27 c. Low-voltage QRS complexes and electrical altemans

28 D. S1Q3T3 pattern and evidence of right ventricular strain


29
E. T-wave inversions in the anterior leads
30
31
32
33
34
35
36
37
38
o39 •
a
Lock
s
Suspend
8
End Bl ock
Item: 39 of 48 ~ 1 • M k -<:J 1>- Jil ~· !:';-~
QIO: 1422 ..L ar Prev ious Next Lab~lues Notes Calculat o r

A A

19
20 Th e correct an swer i s c . 590/o chose this.
21 This Individual has Marfan syndrome and likely suffered a dissecting aortic aneurysm that led to cardiac t amponade. Ca rdiac ta mponade is the result of
fluid (typically blood) accumulating in t he potent ial space between the visceral and parietal pericardium. Because t he fibrous layer of the pericardium Is
22 relatively Inflexible, and because extra fluid is present in t he space, the myocardium is prevented from fully expanding during diastole (right ventricular
collapse Is noted on ECG). This leads t o severely decrea sed ventricular filling and decreased cardiac out put . ca rdiac tam ponade often manifests with Beck
23 triad: hypotension, Increased jugular venous pressure, and distant heart sounds. On ECG, the blood surrounding t he heart can attenuate the recorded
electrical signal, producing diffuse low-volta ge QRS complexes, and electrical altemans (as shown in t he image), in which t he amplitude of the QRS
24
complex oscillates with each beat, which is t hought t o occur as the heart "swings" w1th1n the pericardia! fluid.
25 Marfan syn orne Electrica alternans cardiac tamponade Pericardium Aortic dissection Diastole QRS complex Jugular venous pressure Cardiac muscle
Cardiac output Hypotension Heart sounds Electrocardiography Aortic aneurysm Pericardia! fluid Aneurysm Jugular vein
26
27
28 VI

29
30 oYL

31
32
33 Image copyright © 2009 Banham-Ha/1 and Bokhari; licensee Cases Network Ltd.
34 A is no t co rrect . aoto ch ose this.
35 Myocardial Infarction (MI) caused by an ST-segment eleva tion is defined by the presence of > 1 mm ST-segment elevations in two or more contiguous
leads on ECG (as in this ECG). MI t ypica lly manifests as pressing or crushing, not "tearing," substernal chest pain t hat radiat es to the left arm, not the
36 back. However, one of the complications of MI is ventricular wall rupture, which can lead to cardiac t amponade.
Cardiac tamponade Myocardial infarction Chest pain Sternum Infarction Ventricle (heart)
37
38
39 •
a
Lock
s
Suspend
8
End Block
Item: 39 of 48 ~ 1 • M k -<:J 1>- Jil ~· !:';-~
QIO: 1422 ..L ar Previous Next Lab~lues Notes Calculat or
A A

19
20 I mage copyright © 2009 Banham-Ha/1 and Bokhari; licensee Cases Network Ltd.
21 A is n o t correct. 80/o ch ose this.
22 Myocardial infarction (MI) caused by an ST-segment elevation is defined by the presence of > 1 mm ST-segment elevations in two or more contiguous
leads on ECG (as in this ECG). MI typically manifests as pressing or crushing, not "tearing, • substernal chest pain that radiates to the left arm, not the
23 back. However, one of the complications of MI is ventricular wall rupture, which can lead to cardiac tamponade.
24 Cardiac tamponade Myocardial nfarction Chest pain Sternum Infarction Ventric'e he 1rt

25
26 ...
27
28
29
30
31
32
33
34
B is no t co rrect . 170/o ch ose this.
35
Diffuse ST-segment elevations and PR-segment depressions are ECG findings (shown in this image) associated with ea rly acute pericarditis (Inflammation
36 of the pericardia I sac surrounding the heart) . Acute pericarditis typically manifests as chest pain that is pleuritic in nature and is relieved when the patient
leans forward. Physical examination often revea ls a pericardia! friction rub.
37
Acute pericarditis Pericarditis Pericardia! friction rub Pericardium Pleurisy Electrocardiography Chest pain Inflammation Physical examination
38
39 •
a
Lock
s
Suspend
8
End Bl ock
Item: 39 of 48 ~ 1 • M k -<:J 1>- Jil ~· !:';-~
QIO: 1422 ..L ar Prev ious Next Lab~lues Notes Calculat o r

~
A A

19
20
r f:,-"1----.'"""'--'~..._, --r _:}'0---r'-r---. ~~-''-lf~r+--'"('--"'~""-..
21
8 i s n o t correct. 170/o chose this.
22
Diffuse ST·segment elevations and PR-segment depressions are ECG findings (shown In this image) associated with early acute pericarditis (Inflammation
23 of the pericardia! sac surrounding the heart). Acute pericarditis typically manifests as chest pain that is pleuritic in nat ure and is relieved when the patient
leans forward. Physical examination often reveals a pericardia! friction rub.
24
Ac 1te pe arditis Pericarditis Pericardia! friction rub Pericardium Pleurisy Elect ocarooography Chest pain Inflammation Physical examination
25
26
27
28
29
30
31
32
33
34
35
Image courtesy of Jame Heilman, MD
36
D i s n o t correct. 9 % chose this.
37
The S 1 Q3T3 ECG pattern (shown in this image ) is specific but poorly sensitive for pulmonary embolism; an ECG showing this pattern will have an S wave In
38 lead I, a Q wave in lead Ill, and an inverted T wave in lead Ill. In patients with massive pulmonary emboli, such that resistance through the pulmonary
vasculature is increased, the ECG often will show evidence of right-sided heart strain. The most common ECG finding in pulmonary embolism, however, Is
39 • cln11C l":arkvr~rrii::. O••lrnnn.::.rv eornhnlkrn wn••lrl nr.:~oc::.ont with rheod" n:ain t::.rkvnno::. l"''vcnno::. t.:arhvr.=.rrti::. ;:~~nrt lnw- nr;:. rt c feov13r

a
Lock
s
Suspend
8
End Block
Item: 39 of 48 ~ 1 • M k -<:J 1>- Jil ~· !:';-~
QIO: 1422 ..L ar Prev ious Next Lab~lues Notes Calculat o r
.I!UI IIIII
19
I mage courtesy of lame Heilman, MD
20
D i s n ot correct. 90/o ch ose this.
21 The S 1 Q3T3 ECG pattern (shown in this image) is specific but poorly sensitive for pulmonary embolism; an ECG showing this pattern will have an s wave in
22 lead I, a Q wave in lead III, and an inverted T wave in lead III. In patients with massive pulmonary emboli, such that resistance through the pulmonary
vasculature Is Increased, the ECG often will show evidence of right-sided heart strain. The most common ECG finding in pulmonary embolism, however, Is
23 sinus tachycardia. Pulmonary embolism would present with chest pain, tachypnea, dyspnea, tachycardia, and low-grade fever.
Pulmonary embol sm Dyspnea Tachypnea Sinus tachycardia T wave Electrocardiog• a1 y c: est pain QRS complex Embolism Tachycardia Circulatory system
24
Heart failure Fever
25
26
27 "
28
~---~J-
n -------·-~o-r--~·.-...·
• I ~,2
•\1.. ---.~
f ~ . ~""'~
' ... '-"""-'(,.~ r, · f -
29
r T -wave inversion I Late u.~
30 I
m
31
32
33 II
"'--"'-""..__ ...,.._
34 Image copyright : @ 2009 Todd eta/.
35 E i s n ot co rrect . 70fo ch ose this.
36 T-wave inversions (shown in this image) are nonspecific ECG changes; if T-wave inversions occur in contiguous leads in the setting of chest pain or
pressure, hypotension, and autonomic activation, they can indicate ongoing, likely subendocardial ischemia.
37 Hypotension Ischemia T wave Electrocardiography Chest pain Autonomic nervous system Coronary circulation
38
39 • I~
I!' 11111111 I I!!! IIQ !Ill I 1111111111111111111111 iIi iiII~ ill.;
a
Lock
s
Suspend
8
End Bl ock
Item:39of48 ~. , . M k <:] t> al ~· ~
QIO: 1422 .l. ar Previous Next lab 'Vfl1ues Notes Calculator

19 E is no t co rrect. 7 % cho se this.


T-wave inversions (shown in t his image) are nonspecific ECG changes; if T-wave inversions occur in cont iguous lea ds in t he setting of chest pain or
20
pressure, hypotension, and autonomic activat ion, t hey can indicat e ongoing, likely subendocardial ischemia.
21 Hypotension Ischemia T wave Electrocardiography Chest pain Autonomic nervous system Coronary circulation

22

23
Vo

24
25
26
27 11,1

28
29
30
I mage courtesy of Michael Rosengarten, BEng, MD, McGill University
31
32
33 Bo tto m Line:

34 Defective fibrillin in Marf an syndrome predisposes pat ients to aort ic dissection, which may lea d to cardiac t amponade. Low -volt age ECG wit h electrical
alt ernans is pat hognomonic of t his condit ion. Ot her typical problems wit h Marf an syndrome include lens dislocat ion (ectopia lent is), pneumothorax,
35 mit ral va lve prolapse, and aort ic root dilat ion, which can lea d to aort ic dissection.
Marfan syndrome Cardiac tamponade Aortic dissection Ectopia lentis Mitral valve prolapse Pneumothorax Electrocardiography Mitral valve Fibrillin Aorta Prolapse
36 Ascending aorta
37
38
39

6
lock
s
Suspend
0
End Block
Item:39of48 ~. , . M k <:] t> al ~· ~
QIO: 1422 .l. ar Previous Next lab 'Vfl1ues Notes Calculator
• mitral va lve prolapse, and aortic root dilation, which can lead to aortic dissection .
19
Marfan syndrome Cardiac tamponade Aortic dissection Ectopia lentis Mitral valve prolapse Pneumothorax Electrocardiography Mitral valve Fibrillin Aorta Prolapse
20 Ascending aorta

21
22
23 I iii I;fi 1!1 I•J for year:l 20 17 ..
FIRST AID FAC T S
24
25 FA11 p293.1
26 Aortic dissection Longitudinal intimal tear forming a false lumen (arrows in fit show Aap extendi ng into ascending
27 aorta, Stanford type A dissection). Associated with hypertension, bicuspid aortic valve, inherited
28 connective tissue disorders (eg, Marfan syndrome). Can present with teari ng, sudden-onset chest
29
pain, radiating to the back+/- markedly unequal BP in arms. CXR shows mediastinal widening.
Can result in organ ischemia, aortic rupture, death. Two types:
30
• Stanford type A (proximal}: invokes Ascending aorta. May extend to aortic arch or descending
31 aorta. May result in acute aortic regurgitation or cardiac tamponade. Treatment: surgery.
32 • Stanford type B {distal): only involves descending aorta {Below ligamentum arteriosum). Treat
33 medically with ~-blockers, then vasodilators.
34
35 FA17p56.1

36 Autosomal dominant Achondroplasia, autosomal dominant polycystic kidney disease, fam ilial adenomatous polyposis,
37 diseases familial hypercholesterolemia, hereditary hemorrhagic telangiectasia, hereditary spherocytosis,
Huntington disease, Li-F'raumeni syndrome, Marfan syndrome, multiple endocrine neoplasias,
38
nemofibromatosis type I (,·on Recklinghausen disease}, neurofibromatosis type 2, tuberous
39 • ,.,..],..,..,..,,.:,. • .,.., •.., U ;......... ,..1 I ; .... ...1 ..... . ,.1: ,.,.....,...,,.

6
lock
s
Suspend
0
End Block
19 • Stanford type \ (proximal): involves \ scending aorta. May extend to aortic arch or descending
20 aorta. May result in acute aortic regurgitation or cardiac tamponade. Treatment: surgery.
21 • Stanford type B (distal): only involves descending aorta (Below ligamentum arteriosum). Treat
22
medically with ~-blockers, then vasodilators.

23
FA17 p 56.1
24
25 Autosomal dominant Achondroplasia, autosomal dominant polycystic kidney disease, familial adenomatous polyposis,
diseases familial hypercholesterolemia, heredilar) hemorrhagic telangiectasia, h ereditar~ spherOC) tosis,
26
Huntington disease, Li-F'raumen i S) ndrome, \ larfan syndrome, multiple endocrine neoplasias,
27
neurofibromatosis type I (,·on Recklinghausen disease), neurofibromatosis type 2, tuberous
28 sclerosis, von llippel-Lindau disease.
29
30 FA17 p 300.3
31 Cardiac tamponade Compression of the heart by nuid (eg, blood, effusions [arrows in tll in pericardiaI space) ..... l CO.
32 Equilibration of diastolic pressures in all 4 c hambe r~.
33 Find ings: Beck triad (hypotension, distended neck veins, distant heart sounds), f HR, pulsus
34 paradoxus. ECC shows low-voltage QRS and electrical alternans (due to "swinging" movement of
heart in large effu sion).
35
36 Pulsus paradoxus- l in amplitude of systolic BP by > 10 mm llg during inspiration. Seen in
cardiac tamponade, asthma, obstrueti\ e sleep apnea, pericarditis, croup.
37
38
39 •
a
Lock Suspend
s 8
End Block
Item: 40 of 48 ~ 1 • M k -<:J 1>- Jil ~· !:';-~
QIO: 1432 ..L ar Pre v ious Next Lab~lues Notes Calcula t o r
A A

20 A 52-year-old woman presents to her primary care physician for a routine check-up . She has a photosensitive rash and a history of arthralgias,
Raynaud phenomenon, serositis, and nephritis. On examination, she is found to have a new heart murmur.
21
22
Which of the following pathologic features is most likely?
23
24
A. "Fish mouth" deformation of t he mitral valve
25
B. A brcuspid aortic valve
26
27 c. Aschoff bodies

28 D. Endocardial plaques restricted to the right-sided valves


29
E. Large vegetations with numerous gram-posit ive cocci
30
F. Small sterile vegetations on both sides of the mitral valve
31
32
33
34
35
36
37
38
39
• 40 •
a
Lock
s
Suspend
8
End Bl ock
Item: 40 of 48 ~ 1 • M k -<:J 1>- Jil ~· !:';-~
QIO: 1432 ..L ar Prev ious Next Lab~lues Notes Calculat o r

A A

20
21 The correct an sw er i s F. 600/o chose this.
22 This patient has systemic lupus erythematosus (SLE) as indicated by her constellation of symptoms:
photosensitivity, rash, serositis, renal disease, and Raynaud phenomenon. An Individual with SLE is prone
23 to developing libman-Sacks endocarditis (LSE)- shown in this gross specimen. LSE Is a late feature of SLE
and Is characterized by small vegetations composed of fibrin and inflammatory cells that line either or both
24 surfaces of the valve leaflets. Importantly, t hese growths rarely cause significant valve dysfunction and
25 generally do not embolize.
Raynaud's pi eo omenon Ubman-Sacks endocarditis Systemic lupus erythematOS.JS Embol.sm Endocarditis Fibrin
26 Serositis LUpus erythematosus Rash Kidney disease Kidney
27
28
29
Image courtesy CDC/Or. Edwin P. Ewing, Jr.
30
31 A is not co rrect. 100/o chose this.
Rheumatic heart disease is a late consequence of infection by Streptococcus pyogenes (group A "strep") and causes characteristic thickening, calcification,
32 and fusion of the mitral valve leaflets, resulting in a "fish mouth" appearance.
33 Streptococcus pyogenes Rheumatic fever Mitral valve Cardiovascular disease Streptococcus Calcification Rheumatism Infection Rheumatology

34 B is no t co rrect . 60fo chose this.


A bicuspid aortic valve is a congenital defect in the aortic valve, which normally consists of three valve leaflets. It is present in approximately 1%-2% of
35 the population. It generally is a benign condition, but in a minority of patients may lead to valvular dysfunction (aort ic stenosis or aortic regurgitation)
later In life.
36
Aortic insufficiency Bicuspid aortic valve Aortic stenosis Aortic valve Congenital disorder Stenosis Regurgitation (circulation) Mitral valve
37
C i s n o t co rrect. 10% chose this.
38 Aschoff bodies are the classic pathology findings in rheumatic fever. These lesions are granulomas formed by giant cells. Anitschkow cells (enlarged
39 macrophages with wavy, rod-like nucleus) can often be found within these granulomas.
Rheumat c fever Aschoff body Granuloma Macrophage Anitschkow eel Pathology Fever Ce nucleus Rheumatism
40 •
a
Lock
s
Suspend
8
End Bl ock
Item: 40 of 48 ~. I • M k <:] t> al ~· ~
QIO: 1432 .l. ar Previous Next lab 'Vfllues Notes Calculator

20 Aortic insufficiency Bicuspid aortic valve Aortic stenosis Aortic valve Congenital disorder Stenosis Regurgitation (circulation) Mitral valve

21 c is not co rrect. 10 % chose this .


22 Aschoff bodies are the classic pathology findings in rheumatic fever. These lesions are granulomas formed by giant cells. Anitschkow cells (enlarged
macrophages with wavy, rod-like nucleus) can often be found within these granulomas.
23 Rheumatic fever Aschoff body Granuloma Macrophage Anitschkow cell Pathology Fever Cell nucleus Rheumatism
24 D is not co rrect. 6 % chose this .
25 Carcinoid endocarditis is caused by the secretory products of carcinoid tumors and typically involves the right-sided valves only. Ba cterial endocarditis in
intravenous drug users also is restricted more commonly to right-sided valves (tricuspid valve).
26 Tricuspid valve Endocarditis Carcinoid Infective endocarditis Intravenous therapy Neoplasm Carcinoid syndrome Drug injection

27 E is not co rrect. 8% chose this .


28 Ba cterial endocarditis is caused by pathogens such as Staphylococcus aureus and is often secondary to an infection elsewhere in the body. Infective
endocarditis can cause cardiac failure, myocardial abscess, and glomerulonephritis. It can also result in septic embolization to small vessels of the retina,
29 limbs, and other organs. Alternatively, bacterial endocarditis associat ed with intravenous drug use can affect right -sided heart valves and may result in
embolization to the lungs.
30
Infective endocarditis Staphylococcus aureus Endocarditis Retina Abscess Glomerulonephritis Drug injection Staphylococcus Intravenous therapy Heart valve
31 Heart failure Embolization lung Pathogen Infection

32
33 Botto m Li ne:
34 LSE is associat ed with SLE. It is characterized by deposition of small, sterile veget ations composed of fibrin and inflammatory cells along either side of
35 valve closure lines. They rarely cause serious valvular complications or embolize.
Fibrin Embolism Embolization Infertility
36
37
38 I ill ;fi 1!1 I•J fo r yea r:[ 2017 ..
FI RST AID FA CTS
39
40

6
lock
s
Suspend
0
End Block
Item: 40 of 48 ~. I • M k <:] t> al ~· ~
QIO: 1432 .l. ar Previous Next Lab 'Vfll ues Notes Calculator

20
Bottom Line:
21 LSE is associated with SLE. It is characterized by deposition of small, sterile vegetations composed of fibrin and inflammatory cells along either side of
22 valve closure lines. They rarely cause serious valvular complications or embolize.
Fibrin Embolism Embolization Infertility
23
24
25 I ill ;fi 1!1 I•J for year:[ 2017 ..
FI RST AID FAC TS
26

27
FA17 p443.1
28
Systemic lupus erythematosus
29
SYMPTOMS C lassic presentation: rash, joint pain, and fever, RASH OR PAIN:
30 most commonly in a female of reproductive Rash (ma lar rJ or discoid 1]1)
31 age and African-American descent. Arthritis (nonerosive)
32 Libman-Sacks Endocarditis-nonbacterial, Serositis
33 verrucous thrombi usually on mitral or aortic Hematologic disorders (eg, cytopenias)
valve and can be present on either surface of O ral/nasopharyngeal ulcers
34
the valve (but usually on undersurface) (LSE Renal disease
35 in SLE). Photosensitivity
36 Lupus neph ritis (glomerular deposition of anti- Antinuclear antibodies
37 DNA immune complexes) can be nephritic Immunologic disorder (anti-dsD A, anti-Sm,
38 or nephrotic (hematmia or proteinuria). antiphospholipid)
Most common and severe type is diffuse f\ eurologic disorders (eg, seizures, psychosis)
39
proI iferative.
40 r"""'''H"'""""' .... . ._ .. ( ' 0 (' "r ....l.o., .. J,.. ~" <:: 1 r:;-.

6
lock Suspend
s 0
End Block
Item: 40 of 48 ~ 1 • M k -<:J 1>- Jil ~· !:';-~
QIO: 1432 ..L ar Pre v ious Next Lab~lues Notes Calcula t o r
A A

20 hydroxych loroquine.
21
22 FA17 p 445.3
23 Raynaud phenomenon l blood Aow to skin due to arteriolar (small ,·es~el) vasospasm in response to cold or stress:
24 color change from white (ischemia) to blue (h) poxia) to red (reperfusion). lost often in the
25 fingers fJ and toes. Called Raynaud disease "hen I0 (idiopathic), Raynaud syndrome ''hen 2
26 to a disease process such as mixed connecti'e tissue disease, SLE, or CREST syndrome (limited
form of systemic sclerosis). Digital ulceration (critical ischemia) seen in 2° Raynaud srndrome.
27
Treat with Ca 2- channel blockers.
28
29
30
31
FA17 p 299.2
32
33
Bacterial endocarditis Fever (most common symptom), new murmur, Mitral valve is most frequently involved.
Roth spots (round white spots on retina Tricuspid va lve endocarditis is associated with
34
surrounded by hemorrhage fJ}, Osler nodes IV dmg abuse (don' t " tri" drugs). Associated
35 (tender raised lesions on fin ger or toe pads I}] with S aureus, Pseudomonas, and Candida.
36 due to immune complex deposition), Janeway Culture 8; most likely Coxiella bumetii,
37 lesions (small, painless, erythematous lesions Bartonella spp., IIACEK (llaemophi/us,
38 on palm or sole) m. glomerulonephritis, Aggregatibacter (form erly Actinobacilfus},
septic arterial or pulmonary emboli, splinter Cardiobaclerium, Eikenelfa, Kingella)
39
hemorrhages [!] on nai l bed. Multiple blood • Bacteria FRO~ l JA~E • :
40 •
a
Lock
s
Suspend
8
End Bl ock
Item: 41 of 48 ~ 1 • M k -<:J 1>- Jil ~· !:';-~
QIO: 4056 ..L ar Pre v ious Next Lab~lues Notes Calcula t o r

21
A

A 36-year-old man presents with fever, anorexia, abdominal pain, weight loss, and arthritis that began several weeks ago. Medical history includes an
episode of hepatitis B. Results are negative for antinuclear, anti-double-stranded DNA and antiphospholipid antibodies. Some cutaneous ulcerations
IA•A] A

22
and a purplish discoloration with some ulceration are evident on the lower extremities. Serum creatinine level is 3.2 mg/dl, and blood urea nitrogen
23 level Is 29 mg/dl.

24
Which of the following is another characteristic feature of this patient's most likely diagnosis?
25
26
A . Association w1th polymyalgia rheumatica
27
B. Heart murmur
28
29 c. Photosensitivity

30 o. Same stage of inflammation in different vessels


31
E. Transmural inflammation of blood vessels, which spares the pulmonary circulation
32
33
34
35
36
37
38
39
40
. 41 •
a
Lock
s
Suspend
8
End Bl ock
Item: 41 of 48 ~ 1 • M k -<:J 1>- Jil ~· !:';-~
QIO: 4056 ..L ar Prev ious Next Lab~lues Notes Calculat o r

A A

21
22 Th e correct an swer i s E. 5 4 0/o chose this.
23 Polyarteritis nodosa is associated with hepatitis B virus infection in up to 30% of cases. The disease is a
vasculitis of the small- to medium-sized arteries. Signs and symptoms are nonspecific and result from
24 damage to the organ the vessels are feeding. Musculoskeletal, cutaneous, gastrointestinal, cardiac, renal,
and neurologic symptoms can be seen; of note, t he lungs are spared (In contrast to granulomatosis with
25 polyanglitls). Patients often have a skin rash known as livedo reticularis, which is the purplish discoloration ~ ~

26 described in the v1gnette. Additionally, patients may have anemia, elevated erythrocyte sedimentation rate ~ k.' .L.
or c-reactive protein level and neutrophilic leukocytosis. In contrast, eosinophilia is rare and suggests
. I ..
Churg-Strauss d1sease. Episodes normally last a few weeks to a few months, and < 10% of patients will
27
suffer relapse , but disease can rapidly progress to fulminant illness. Histologically, transmural necrotizing .
\\

28 Inflammation is seen, and this may progress to fibrosis and thickening of vesse l wail , like that shown in this . .. ,\, '~ I~
Image. In contrast to microscopic polyangiitis, different stages of inflammation may coexist in different
29 vessels. 1;:.;\~ ?J
30 Granulomatosis with polyangiitis Uvedo reticularis C-reactive protein Microscopic polyangiitis . ...:...,. (-·
. . ,.,r,.
... ..-.... :. ..;..'·,._.:.r..
Erythrocyte sedimentation rate Leukocytosis Red blood cell Hepatitis 8 Eosinophilia Vasculitis Anemia
31 Image copyright ©2012 Rodrigo eta/.;
Inflammation Neutrophil Fibrosis Churg-Strauss syndrome Protein Virus Hepatitis Histology Artery Rash Lung licensee BioMed Central Ltd.
32
Necrosis Polyarteritis nodosa Gastrointestinal tract Infection Kidney Human musculoskeletal system
33 Hepatitis 8 virus

34 A is not co rrect . 1 30/o chose this.

35 Polymyalgla rheumatica is associated with t emporal a1teritis, which usually occurs In people >70 years old and rarely occurs in people <SO years old. It
affects women more often than men and involves vessels above the bifurcation of the ao1ta, without the cutaneous symptoms mentioned In the stem.
36 Polymyalgia rheumatica Giant-cell arteritis Aorta Arteritis

37 B is not co rrect . go;o ch ose this.

38 Heart murmur along with fever, Roth spots, Janeway lesions, and Osier nodes can be associated with infective endocarditis. Infective endocarditis can
Involve some of the constitutional symptoms described in this case, but the cutaneous symptoms described are not a feature of infective carditis.
39 Roth's spot Osier's node Janeway lesion Heart murmur Endocarditis Infective endocarditis Fever

40 C i s not cor r ect. 8 0/o ch ose this.

41 Classically, photosensitivity is associated with systemic lupus erythematosus (SLE) along w1th a malar or discoid rash. Both constitutional cutaneous and

a
Lock
s
Suspend
8
End Bl ock
Item:41of48 ~. , . M k <:] t> al ~· ~
QIO: 4056 .l. ar Previous Next lab 'Vfl1 ues Notes Calculator

21 B is not co rrect. 9 % chose this .


Heart murmur along with fever; Roth spots, Janeway lesions, and Osier nodes can be associat ed with infective endocarditis. Infective endocarditis can
22 involve some of the constitutional symptoms described in this case, but the cutaneous symptoms described are not a fea ture of infective carditis.
Roth' s spot Osier' s node Janeway lesion Heart murmur Endocarditis Infective endocarditis Fever
23
24 c is not co rrect. 8% chose this .
Classically, photosensitivity is associat ed with systemic lupus erythem atosus (SLE) along with a m alar or discoid rash. Both constitutional cutaneous and
25 renal involvem ent can be fea tures of SLE, but this patient's negative autoantibodies m ake SLE unlikely, and his prior case of hepatitis B m akes
polyarteritis nodosa most likely.
26
Polyarteritis nodosa Systemic lupus erythematosus Photosensitivity Hepatitis B Autoantibody Hepatitis lupus erythematosus Rash Cheek Kidney
27
D is not co rrect. 1 6 % chose this .
28 Microscopic polyangiitis is a necrotizing va sculitis which affects small vessels. In contrast to polyarteritis nodosa, all vessels are typically at the sam e stage
of inflammation . Granulomas are also absent in this disea se. It is associat ed with the MPO-AN CA antibody. The commonly affected organs are lung and
29 kidney.
30 Microscopic polyangiitis Polyarteritis nodosa Vasculitis Granuloma Systemic vasculitis Necrosis Antibody lung Inflammation Kidney

31
32 Botto m Li ne:
33 Polya rteritis nodosa is associat ed with hepatitis B and involves necrotizing immune complex inflammation and multiple organs.
Polyarteritis nodosa Immune complex Hepatitis B Hepatitis Inflammation Necrosis
34
35
36
lijl;fiiJI•l to r yea r:[ 20 1 7 • ]
37 FI RST AID FAC T S

38
FA17 p 302.1
39
Vasculitides
40
EPIDEMIOLOGY/PRESENTATION PATHOLOGY/lABS
41
·~

6
lock
s
Suspend
0
End Block
Item: 42 of 48 ~ 1 • M k -<:J 1>- Jil ~· !:';-~
QIO: 3218 ..L ar Pre v ious Next Lab~lues Notes Calcula t o r
A A

22 A 60-year-old woman presents to her physician for a routine visit. Physical examination reveals a rumbling, late diastolic murmur preceded by an
opening snap, heard best near the apex. The physician inquires about the patient's medical history and determines that this murmur is probably the
23
result of a prior illness.
24
25 Which untreated medical condition most likely caused the murmur in this patient?
26
27 A. Atrial fibrillation

28 B. I nfective endocarditis
29
c. Myocarditis
30
D. Streptococcal pharyngitis
31
32 E. Systemic lupus erythematosus

33
34
35
36
37
38
39
40
41
. 42 •
a
Lock
s
Suspend
8
End Bl ock
Item: 42 of 48 ~ 1 • M k -<:J 1>- Jil ~· !:';-~
QIO: 3218 ..L ar Previous Next Lab~lues Notes Calculat or
A A

22
Th e co rrect an sw er i s D. 680/o chose this.
23 The first step to answering this question is correctly identifying the murmur. Mitral stenosis (MS) often causes a sharp, high-pitched opening snap during
24 diastole (due to the opening of the stiffened mitral valve leaflets) followed by a harsh rumbling murmur. The murmur of MS is low pitched and Is best
heard at the apex of the heart. The most common cause of MS is rheumatic heart disease, which can occur as a result of untreated Group A streptococcal
25 pharyngitis. A few weeks after resolution of the pharyngitis, acute rheumatic fever may occur, characterized by fever, carditis, migratory polyarthritis,
chorea, and erythema marginatum. This condition, in turn, can lead (many years later) to chronic rheumatic heart disease, which often causes MS. The
26 pathogenesis of disease is related to the concept of molecular mimicry: Antibodies directed against streptococcal antigens cross-react with host antigens
on the cardiac valves.
27
Streptoco :ca pi aryngitis Erythema marginatum Rheumatic fever Molecular momict y Mito al vahte Po yarthritis Erythema Pharyngitis Mitral valve stenosis Chorea
28 Heart valve Carditis Antibody Antigen Diastole Streptococcus Cardiovascular disease Stenosis Heart murmur Pathogenesis
29 A i s not correct. 3% chose this.
30 Atrial fibrillation can be a direct result of MS. As the valve becomes Increasingly stenotic, diastolic pressures within the left atrium become greater and
greater as contraction must occur against a noncompliant valve. This leads to gradual dilatation of the left atrium, which in turn alters the electrical
31 circuits, causing atrial fibrillation. However, this question asks which condition would have caused the murmur; atrial fibrillation is a result, not a cause, of
MS.
32
Atrial fibrillation Atrium (heart) Diastole Stenosis Fibrillation Heart murmur
33
B is not co rrect. 200/o chose this.
34 Infective endocarditis, if allowed to progress, can result in valve destruction. However, this almost always causes a murmur of valvular insufficiency (can
35 be left- or right-sided), rather than stenosis. There are occasional cases of MS due to infective endocarditis, but rheumatic heart disease is much more
likely.
36 Infective endocarditis Rheumatic fever Endocarditis Stenosis Regurgitation (circulation) Cardiovascular disease Rheumatism Heart murmur

37 C is no t co rrect . soto ch ose this.

38 Myocarditis (infection or inflammation of the myocardium) can lead to various types of abnormal heatt sounds. An 5 3 or 5 4 heart sound may be heard due
to ventricular dysfunction, or if cardiac dilat ation occurs, mitral or tricuspid Insufficiency murmurs might be heard . However, MS would not occur.
39 Tricuspid insufficiency Myocarditis Cardiac muscle Heart sounds Inflammation Tricuspid valve Infection Tricuspid atresia

40 E i s n ot correct. 40/o ch ose this.

41 Valvular damage can occur in systemic lupus erythematosus (SLE) in the form of Ubman-Sacks endocarditis (seen in about 11% of patients with SLE).
Verrucae composed of immune complexes, fibrin, and other substances accumulate along the edges of either the mitral or the aortic valve. However, If a
42 murmur results, it is generally either a mitral or an aortic regurgitation murmur. Rheumatic heart disease is by far the most common cause of MS.

a
Lock
s
Suspend
8
End Bl ock
Item:42of48 ~. , . M k <:] t> al ~· ~
QIO: 3218

22
.l.
. ar Previous Next
.
lab 'Vfllues
.
Notes Calculator
. ' .
to ventricular dysfunction, or if cardiac dilatation occurs, mitral or tricuspid insufficiency murmurs might be heard . However; MS would not occur.
..
Tricuspid insufficiency Myocarditis Cardiac muscle Heart sounds Inflammation Tricuspid valve Infection Tricuspid atresia
23
E is not correct. 4 % chose this.
24
Valvular damage can occur in systemic lupus erythematosus (SLE) in the form of Ubman-Sacks endocarditis (seen in about 11% of patients with SLE).
25 Verrucae composed of immune complexes, fibrin, and other substances accumulate along the edges of either the mitral or the aortic valve . However; if a
murmur results, it is generally either a mitral or an aortic regurgitation murmur. Rheumatic heart disease is by far the most common cause of MS.
26
libman-Sacks endocarditis Systemic lupus erythematosus Rheumatic fever Aortic insufficiency Endocarditis Aortic valve Fibrin Cardiovascular disease
27 lupus erythematosus Heart murmur Immune complex

28
29 Bottom Line:
30 MS is a diastolic murmur best heard at the apex of the heart . The most common cause of MS is rheumatic heart disease, which results from untreated
31 group A streptococcal pharyngitis.
Streptococcal pharyngitis Rheumatic fever Pharyngitis Apex beat Cardiovascular disease Streptococcus Heart murmur
32
33
34 I ill ;fi 1!1 I•J for year:[ 2017 ..
FI RST AI D FA CTS
35
36
FA17 p 300.1
37
Rheumatic fever A consequence of pharyngeal infection with J'INES (major criteria):
38
group A ~-hemolytic streptococci. Late Joint (migratory polyarthritis)
39 'I (carditis)
sequelae include rheumatic heart disease,
40 wh ich affects heart valves-mitral > aortic>> Nodules in skin (subcutaneous)
41 tricuspid {high-pressure valves affected most). Erythema marginatum
42 Early lesion is mitral valve regurgitation; Sydenham chorea

6
lock
s
Suspend
0
End Block
Item: 42 of 48 ~ 1 • M k -<:J 1>- Jil ~· !:';-~
QIO: 3218 ..L ar Pre v ious Next Lab~lues Notes Calcula t o r

22
(enlarged macrophages with ovoid, wavy,
23 rod-like nucleus [red arrow in fJ]), t anti-
24 streptolysin 0 (ASO) titers.
25 Immune mediated (type II hypcrscnsit i' ity);
26 not a direct effect of bacteria. Antibodies
toM protein cross-react with self antigens
27
(molecular mimicry).
28
Treatment/prophylaxis: penicillin.
29

30
FA17 p 279.1
31
Heart murmurs
32
Systolic
33
Aortic stenosis Crescendo-decrescendo systolic ejection murmur (ejection click may be present).
34
L >>aortic pressure during S)$tole. Loudest at heart base; radiates to carotids.
35
51 52
"Pulsus parvus et tardus"-pulses are weak with a delayed peak. Can lead to
36
~I Syncope, Angina, and Dyspnea 011 exertion (SAD). Most commonly due to age-
related calcific;~ t ion in older patients(> 60 years old) or in younger patients with
37
38 early-onset calcificat ion of bicuspid aortic vah-e.
39 Mitral/tricuspid regurgitation Holosystolic, high-pitched "blowing murmur.''
51 52 Mitral-loudest at apex and radiates toward axilla. MR is often due to ischemic heart
40
disease (post-~ 1!), \1 VP, LV dilatation.
41
42
~ Tricuspid-loudest at tricuspid area. TR commonly caused by RV dilatation.
Rheumatic fever and infective endocarditis can cause either~ I R or T R.

a
Lock
s
Suspend
8
End Bl ock
Item: 43 of 48 ~ 1 • M k -<:J 1>- Jil ~· !:';-~
QIO: 5167 ..L ar Pre v ious Next Lab~lues Notes Calcula t o r

23
A

A 67-year-old man presents to the outpatient clinic with a chief complaint of progressive shortness of breath, particularly when he tries to lie flat . He
reports needing to use four pillows at night to breathe comfortably. Physical examination reveals a laterally displaced and diffuse point of maximal
IA•A] A

24
Impulse, an 53, an 54, pulmonary rales, hepatomega ly, and 2+ bilateral lower-extremity edema . x-ray of the chest shows a large, balloon-shaped
25 heart.

26
These findings most likely result from what condition?
27
28
A. Heavy alcohol use
29
B. Hypertrophic cardiomyopathy
30
31 c. Multiple mye loma

32 o. Rad iation therapy


33
E. Systemic sclerosis
34
35
36
37
38
39
40
41
42
. 43

a
Lock
s
Suspend
8
End Bl ock
Item: 43 of 48 ~ 1 • M k -<:J 1>- Jil ~· !:';-~
QIO: 5167 ..L ar Previous Next Lab~lues Notes Calculat or
A A

23
24 Th e correct a n swer i s A. 560/o chose this.
25 This patient shows signs and symptoms of dilated cardiomyopathy. Orthopnea, rales, hepatomegaly, and
edema are all suggestive of heart failure. The displaced point of maximal Impulse with a chest x-ray
26 showing a large, balloon-shaped heart (like t hat shown here) is indicative of dilated cardiomyopathy.
Dilated cardiomyopathy has several causes, which can be recalled with the mnemonic ABCCCD : Alcohol
27 abuse, Beriberi, Coxsackie B myocardit is, chronic Cocaine use, Chagas disease, and Doxorubicin toxidty.
likewise, the mnemonic DCCCBA (Dilat ed Ca rdiac Complications Caused By Alcohol) may aid in
28 re m em bering this spec1fic cause of d il ated card iomyopathy.
29 Ape beat Ortnopnea Myocarditis Hepatomegaly Dilated cardiomyopathy Crad..les Chest , adiograph

30 Cardiomyopathy Edema Heart failure X-ray

31
32
33 Image courtesy of Nevit Oilmen
34
B is not co rrect. 260/o chose this.
35
Hypertrophic cardiomyopathy is the m ost common of the genetic cardiovascular diseases. It is transmitted in an autosom al dominant pattern . As the
36 name Implies, It generally produces hypertrophic changes (and t ypically does not present as a dilat ed cardiomyopathy, unless in the advanced stages).
The manifestation of the phenotype at t his older age would be unusual for hypertrophic cardiomyopathy.
37 Hypertrophic cardiomyopathy Dilated cardiomyopathy Dominance (genetics) Hypertrophy Phenotype Ventricular hypertrophy Autosome Cardiomyopathy
38 Cardiovascular disease Circulatory system

39 C is n ot co rrect. 40/o ch ose this.


40 Multiple myeloma often causes the AL form of am yloid to deposit in cardiac t issues. This leads to restrictive ca rdiom yopathy due to stiffening of the
ventricular walls.
41 Multiple myeloma Restrictive cardiomyopathy cardiomyopathy Amyloid

42 0 is not corr ect. 70/o ch ose this.


Radiation therapy is known to cause fibrosis and scarring in exposed t issue. Fibrosis would lead to a restrictive ca rdiom yopathy, not the dilated
43 • .. .. ' .. . . ...
a
Lock
s
Suspend
8
End Block
Item: 43 of 48 ~. I • M k <:] t> al ~· ~
QIO: 5167 .l. ar Previous Next lab 'Vfl1ues Notes Calculator

23 c is no t co rrect. 4 % cho se this.


Multiple myeloma often causes the A L form of amyloid to deposit in cardiac tissues. This lea ds to restrictive cardiomyopathy due to stiffening of the
24 ventricular walls.
25 Multiple myeloma Restrictive cardiomyopathy Cardiomyopathy Amyloid

26 D is no t co rrect. 7 % cho se this.


Radiation therapy is known to cause fibrosis and scarring in exposed tissue. Fibrosis would lea d to a restrictive cardiomyopathy, not the dilat ed
27 cardiomyopathy described in this patient.
Dilated cardiomyopathy Radiation therapy Restrictive cardiomyopathy Fibrosis Cardiomyopathy Radiation
28
29 E is no t co rrect. 7 % cho se this.
Systemic sclerosis (scleroderma) is a disea se consisting of a triad of autoimmunity, noninflammatory vasculopathy, and collagen deposition with fibrosis. It
30 commonly involves sclerosis of skin (taut skin with no wrinkles) as well as the renal, pulmonary, GI, and cardiovascular systems. Systemic sclerosis causes
fibrosis of cardiac myocytes lea ding to a restrictive cardiomyopathy.
31
Collagen Systemic scleroderma Cardiomyopathy Autoimmunity Restrictive cardiomyopathy Cardiac muscle Myocyte Fibrosis Circulatory system
32 Cardiac muscle cell Vasculitis Kidney
33
34
Botto m Li ne:
35 Dilat ed cardiomyopathy has several causes, including chronic alcohol abuse, beriberi, coxsackie B myocarditis, chronic cocaine use, Chagas disea se, and
36 doxorubicin toxicity. In contrast, sarcoidosis and most deposition disea ses, including amyloidosis, lea d to restrictive cardiomyopathy in which the
ventricular walls stiffen and do not fill complet ely during diastole.
37 Chagas disease Beriberi Doxorubicin Sarcoidosis Dilated cardiomyopathy Myocarditis Diastole Amyloidosis Restrictive cardiomyopathy Cocaine Cardiomyopathy
Coxsackie B virus Alcohol Coxsackievirus Alcohol abuse Alcoholic beverage
38
39
40
I ill ;fi 1!1 I•J f o r yea r :[ 2017 ..
41 FI RST AI D FA CTS

42
43 FA17 p 297.1

6
lock
s
Suspend
0
End Block
Item: 43 of 48 ~ 1 • M k -<:J 1>- Jil ~· !:';-~
QIO: 5167 ..L ar Pre v ious Next Lab~lues Notes Calcula t o r
A A

23
24 ljl;fii;1!•J f o r yea r : 2017 ...
FIRST AID FAC T S
25
26
FA17 p 2 97.1
27
Cardiomyopathies
28
Dilated ~ lost common cardiomyopath} (90% of cases). Systolic dysfunction ensues.
29
cardiomyopathy Often idiopathic or familiaL Other etiologies Eccentric hypertrophy · (sarcomeres added in
30 include chronic \ lcohol abuse, wet Beriberi, series).
31 Coxsackie B viral myocarditis, chronic ABCCCD.
32
33
(. I
... ":..
-~

..-r
Cocaine use, C hagas disease, Doxorubicin
toxicity, hemochromatosis, sarcoidosis,
Takotsubo cardiomyopathy: "broken heart
syndrome"-\·entricular apical ballooning

34 '• -.......·- peripartum cardiomyopathy.


Findings: I IF, S3, systolic regurgitant murmur,
likcl) due to increased sympathetic stimulation
(stressful situations).
35 dilated heart on echocardiogram, bal loon
36 appearance of heart on CXR.
37 Treatment: a • restriction, ACE inhibitors,
~-bl ockers, diuretics, digoxin, lCD, hea rt
38
transplant.
39
Hypertrophic 60-70% of cases are familial , autosomal Diastol ic dysfunction ensues.
40
cardiomyopathy dominant (most common ly due to mutations Marked ventricu lar concentric hypertrophy
41 in genes encoding sarcomeric proteins, such as (sarcomcrcs added in parallel) rn, often septal
42 myosin binding protein C and ~ -myosin heav) predominance. J\lyofibrillar disarray and
43 chain). Can be associated with Friedreich fibrosis.

a
Lock
s
Suspend
8
End Bl ock
Ite m: 4 3 o f 48 ~ 1 • M k -<:J 1>- Jil ~· !:';-~
..L ar Lab~lues
QIO: 5167

23
Previous Next
. ~. .. .
Not es
. . .
Calculator
. .. .. . . . .
(although dilated cardiomyopathy is more a prominent eosinophilic infiltrate.
24 common) (Puppy LE.\ SII).
25
26 FA17 p 541 .2
27 Alcoholism Physiologic tolerance and dependence on alcohol with symptoms of withdrawal when intake is
28 interrupted.
29 Complications: alcoholic cirrhosis, hepatitis, pancreatitis, peripheral neuropathy, testicular atrophy.
30 Treatment: disulfiram (to condition the patient to abstain from alcohol use), acamprosate,
naltrexone, supportive care. upport groups such as Alcoholics nonymous are helpful in
31
sustaining abstinence and supporting patient and family.
32
Wernicke-Korsakoff Caused by vitamin B1 deficiency. Triad of confusion. ophthalmoplegia, ataxia (Wernicke
33 syndrome encephalopathy). May progress to irreversible memory loss, confabulation, personality change
34 (Korsakoff syndrome). Symptoms may be precipitated by giving dextrose before administering
35 vitamin B1 to a patient with thiamine deficiency. Associated with peri\·entricular hemorrhage/
36 necrosis of mammillary bodies. Treatment: I vitamin 8 1•
37
FA17 p 372.2
38
39 Cirrhosis and portal hypertension

40 Cirrhosis-diffuse bridging fibrosis (via stellate cells) and regenerative nodules (red arrows in · ;
white arro\\'s shows splenomegaly) disrupt normal architecture ofli\·er; t risk for hepatocellular
r.~~.. _.,
41
' Ji\ carcinoma (IICC). Etiologies include alcohol (60-70% of cases in the US), nonalcoholic
42
43 •
'
I •
steatohepatitis, chronic \'ira! hepatitis, autoimmune hepatitis, biliary disease, genetic/metabolic
rlisorriPrs.
a
Lock
s
Suspend
8
End Block
Ite m: 4 3 o f 48 ~ 1 • M k -<:J 1>- Jil ~· !:';-~
..L ar Lab~lues
QIO: 5167

23
Previous Next
. ~. .. .
Not es
. . .
Calculator
. .. .. . . . .
(although dilated cardiomyopathy is more a prominent eosinophilic infiltrate.
24 common) (Puppy LE.\ SII).
25
26 FA17 p 541 .2
27 Alcoholism Physiologic tolerance and dependence on alcohol with symptoms of withdrawal when intake is
28 interrupted.
29 Complications: alcoholic cirrhosis, hepatitis, pancreatitis, peripheral neuropathy, testicular atrophy.
30 Treatment: disulfiram (to condition the patient to abstain from alcohol use), acamprosate,
naltrexone, supportive care. upport groups such as Alcoholics nonymous are helpful in
31
sustaining abstinence and supporting patient and family.
32
Wernicke-Korsakoff Caused by vitamin B1 deficiency. Triad of confusion. ophthalmoplegia, ataxia (Wernicke
33 syndrome encephalopathy). May progress to irreversible memory loss, confabulation, personality change
34 (Korsakoff syndrome). Symptoms may be precipitated by giving dextrose before administering
35 vitamin B1 to a patient with thiamine deficiency. Associated with peri\·entricular hemorrhage/
36 necrosis of mammillary bodies. Treatment: I vitamin 8 1•
37
FA17 p 372.2
38
39 Cirrhosis and portal hypertension

40 Cirrhosis-diffuse bridging fibrosis (via stellate cells) and regenerative nodules (red arrows in · ;
white arro\\'s shows splenomegaly) disrupt normal architecture ofli\·er; t risk for hepatocellular
r.~~.. _.,
41
' Ji\ carcinoma (IICC). Etiologies include alcohol (60-70% of cases in the US), nonalcoholic
42
43 •
'
I •
steatohepatitis, chronic \'ira! hepatitis, autoimmune hepatitis, biliary disease, genetic/metabolic
rlisorriPrs.
a
Lock
s
Suspend
8
End Block
Item: 43 of 48 ~ 1 • M k -<:J 1>- Jil ~· !:';-~
QIO: 5167 ..L ar Pre v ious Next Lab~lues Notes Calcula t o r
A
.. . A

23 Cardiomyopathy
. - - - Peripheral edema
24
25
26
27
FA17 p 154.1
28
Protozoa-others
29 ORGANISM DISEASE TRANSMISSION DIAGNOSIS TREATMENT
30 Visceral mfections
31 Trypanosoma Chagas disease- dilated Rcduviid bug Trypomastigote in Bcnznidazolc
32 cruzi cardiomyopathy with ("kissing bug") blood smear fJ or nifurtimox;
33 apical atrophy, megacolon, feces, deposited C nJZing in my
34 megaesophag us; predominantly in a pai nlcss bite Bcn.r., with a fur
in South America (much like a li~!>) coat on
35
Unilateral periorbita I swelling
36
(Romaiia sign) characteristic of
37 acute stage
38
Leishmania Viscera l leishmaniasis Sandny Macrophages Amphotericin B,
39 donovani (kala-azar) - spiking fevers, containing sodium
40 hepatosplenomegaly, amastigotes : stibogluconate
41 pancytopenia
Cutaneous leishmaniasis-skin
42
ulcers
43 •
a
Lock
s
Suspend
8
End Bl ock
Item: 44 of 48 ~ 1 • M k -<:J 1>- Jil ~· !:';-~
QIO: 1057 ..L ar Pre vious Next Lab~lues Notes Calcula t o r

24
A

A previously healthy 31-year-old woman presents to the emergency department with sudden, complete loss of vision in her right eye. History Is
significant for malaise, chills, and fatigue of 3 days' duration, in addition to oral pain secondary to recent remova l of a wisdom tooth . Ophthalmologic
IA•A] A

25
examination reveals a gray-white retina with an associated cherry-red spot, two blot hemorrhages, and several segmented vessels with optic edema .
26 Physical examination reveals a murmur, like that heard in the audio clip .

27 OPEN MEDIA

28
29 II _ oooo tooo1 -~
30
31
Which of the following is the most likely cause of this patient's loss of vision?
32
33 A. Carotid artery stenosis
34
B. Collagen vascular disease
35
36 c. Diabetes mellitus

37 o. Endocarditis
38
E. Multiple sclerosis
39
40
41
42
43
• 44 •
a
Lock
s
Suspend
8
End Bl ock
Item: 44 of 48 ~ 1 • M k -<:J 1>- Jil ~· !:';-~
QIO: 1 0 57 ..L ar Previous Next Lab~lues Notes Calculat or
A A

24
25
The correct a n sw er is o. 650/o chose this.
26
The most likely cause of this patient's symptoms is septic emboli occluding the central retinal artery as a
27 result of infective endocarditis. These septic emboli are known as Roth spots (one indicated by arrow in the
Image.) This patient also has a heart murmur consistent with mitral valve prolapse. Although infective
28 endocarditis may arise occasionally without predisposing anatomic abnormalities, the risk is increased
29 significantly 1n patients with mitral valve prolapse. Individuals with central retinal artery occlusion typica lly
present with the acute onset of painless monocular visual loss. The occlusion Is usually the result of
30 thromboembolic phenomena re lating to vascular disease. In patients with valve abnormalities and a source
of bacteremia, such as the patient in this vignette, e mbolism of vegetations Is a possible cause.
31 Unfortunately, even with treatment, fewer than 25% of patients with this diagnosis regain useful vision in
the affected eye.
32
Roth's spot Bacteremia Infective endocarditis Endocarditis Central retinal artery occlusion Heart murmur
33 Mitral valve prolapse Embolism Mitral valve Vascular disease Septic embolism Blood vessel
34 Central retinal artery Ocular ischemic syndrome

35
36
I mage courtesy of Nicholas Mahoney, MD
37
38 A is not co rrect . 120/o chose this.
A large proportion of central retinal a1tery occlusions are secondary to thromboembolic phenomena originating from atherosclerotic carotid arteries.
39 However, given the history and this patient's lack of m acrovascular disease, this Is a less likely cause of this patient's condition .
Atherosclerosis Macrovascular disease Central retinal artery Common carotid artery Artery Carotid artery
40
B i s not co rrect. 120/o chose this.
41
Central retinal artery occlusion may result from sequelae of collagen vascular disease, but this is a relatively infrequent cause and is unlikely given the
42 lack of any chronic history consistent with such a diagnosis.
43 Autoimmune d sease Central retinal artery occlusion Ocular ischemic syndrome Central retinal artery Retinal Vascular disease Vascular occlusion Collagen
Blood vessel
44 •
a
Lock
s
Suspend
8
End Bl ock
Item: 44 of 48 ~. I • M k <:] t> al ~· ~
QIO: 1057 .l. ar Previous Next lab 'Vfl1ues Notes Calculator
• y • y y • y
24
B is no t co rrect. 12% cho se this.
25 Central retinal artery occlusion may result from sequelae of collagen va scular disea se, but this is a relatively infrequent cause and is unlikely given the
26 lack of any chronic history consistent with such a diagnosis.
Autoimmune disease Central retinal artery occlusion Ocular ischemic syndrome Central retinal artery Retinal Vascular disease Vascular occlusion Collagen
27
Blood vessel
28
c is no t co rrect. 7 % cho se this.
29 Although diabet es is a lea ding cause of microvascular disea se, it is a rare cause of acute central retinal artery occlusion as it is a chronic condition with an
insidious onset . Comparatively, endocarditis would have a much more acute onset .
30
Central retinal artery occlusion Endocarditis Microangiopathy Diabetes mellitus Central retinal artery Ocular ischemic syndrome Vascular occlusion
31
E is no t co rrect. 4 % cho se this.
32 Multiple sclerosis can cause optic neuritis, which is a sudden, monocular blindness that strongly resembles retinal artery occlusion. Optic neuritis differs
from retinal artery occlusion in that it is painful, particularly with eye movement, and is usually transient, lasting days to weeks. Ophthalmologic exam is
33 usually normal in optic neuritis, wherea s retinal artery occlusion has characteristic findings such as cherry-red spots and blot hemorrhages.
34 Optic neuritis Multiple sclerosis Ocular ischemic syndrome Central retinal artery Visual impairment Ophthalmology Bleeding Neuritis

35
36 Bo tto m Line:
37 Endocarditis can present with fever; Roth spots, Osier nodes, new murmur; Janeway lesions, and splinter hemorrhages. Roth spots are due to retinal
hemorrhages from septic emboli and are seen as central area clearing on funduscopic examination .
38 Roth's spot Osier's node Janeway lesion Endocarditis Ophthalmoscopy Embolism Septic embolism Bleeding Fever Heart murmur

39
40
41 lijj ;fi IJ l•l f o r year:l 2 0 1 7 ..
FIRST AID FAC T S
42
43
FA17 p 299.2
44 :_. -- _. ____ _.:..
,.. __ ,.. __ :_
r. +•'-- · -- ' .
- • r_ ol • _ l

6
lock
s
Suspend
0
End Block
Item: 45 of 48 ~ 1 • M k -<:J 1>- Jil ~· !:';-~
QIO: 5165 ..L ar Pre v ious Next Lab~lues Notes Calcula t o r
A A

25 A baby boy is delivered to a healthy mother. After 1 hour the infant is clearly cyanotic. The newborn is started on supplemental oxygen, but his color
does not Improve. A congenital heart defect is suspected. An echocardiogram reveals transposition of the great vessels. There is no atrial septal
26
defect and only a very small ventricular septal defect.
27
28 Infusion of which of the following is indicated?
29
30 A. Albuterol

31 B. Erythropoietin
32
c. Heparin
33
D. Indomethacin
34
35 E. Prostaglandin E

36
37
38
39
40
41
42
43
44
• 45 •
a
Lock
s
Suspend
8
End Bl ock
Item: 45 of 48 ~ 1 • M k -<:J 1>- Jil ~· !:';-~
QIO: 5165 ..L ar Prev ious Next Lab~lues Notes Calculat o r

A A

25
Th e co rrect an swer i s E. 800/o chose this.
26 In transposit ion of the great vessels, the aorta leaves the right ventricle and the pulmonary trunk leaves the left ventricle, creating two separate,
27 noncommunicating circulations. This is not compatible with life unless there Is a ventricular septal defect (VSD), atrial septal defect (ASD), or patent
ductus arteriosus (PDA) to allow some mixing of the blood. If no connection Is made between the right and left circulation, the lungs will continue to
28 oxygenate oxygenated blood and the systemic tissues will continue to circulate deoxygenated blood. This newborn's cyanosis will likely get worse as the
ductus closes over the next few days. Prostaglandin E should be used to keep the ductus arteriosus open until definitive surgical repair can occur.
29 Atna sept< defect Transposition of the great vessels Ventricular septal defect Cyano~ s Patent ductus arteriosus Prostaglandin Pulmonary artery Aorta
30 Duct IS a te1oosus Ventricle heart) Great vessels Blood Atrium (heart) Lung

31 A i s not correct. 2% chose this.


32 Albuterol Is a short-acting 13-agonist that is used to relieve bronchospasm by relaxing smooth muscle in the airways. This newborn's problem Is circulatory,
not respiratory.
33 Bronchospasm Smooth muscle tissue Salbutamol Muscle

34 B is not co rrect. 2% chose this.


35 Erythropoietin causes increased erythropoiesis and is used to raise the hematocrit in appropriate situations (eg, chronic renal failure). This newborn's
problem Is decreased oxygenation of systemic blood due to separation of the pulmonary and systemic circulation, not decreased oxygen-carrying capacity
36 of the blood.
Erythropoietin Erythropoiesis Hematocrit Chronic kidney disease Circulatory system Oxygen saturation (medicine) Kidney
37
C is not co rrect. 20/o chose t his.
38
Heparin Is an anticoagulant that is used for the trea tment of myocardial Infarctions, pulmonary emboli, and deep venous thrombosis. There Is no
39 Indication for heparin in this newborn's case.
Anticoagulant Deep vein thrombosis Heparin Pulmonary embolism Venous thrombosis Embolism Thrombosis Myocardial infarction Vein
40
D i s n ot co rrect . 140/o ch ose this.
41
Indomethacin Is a nonsteroidal anti-inflammatory drug that decreases the production of prostaglandins. It is used to close a patent ductus arteriosus, the
42 opposite of what this newborn currently needs. The newborn's life depends on an open conduit between both sides of the circulation so oxygenated blood
can make It to systemic tissues.
43 Nonsteroidal anti-inflammatory drug Patent ductus arteriosus Indometacin Prostaglandin Ductus arteriosus Anti-inflammatory Patent
44
45 •
a
Lock
s
Suspend
8
End Bl ock
Item: 45 of 48 ~. I • M k <:] t> al ~· ~
QIO: 5165 .l. ar Previous Next lab 'Vfl1ues Notes Calculator

25
26 BottomLine:
27 Transposition of the great vessels is not compatible with life unless a VSD, ASD, or PDA exists to allow mixing of blood between the two separate
circulations. Prostaglandin E should be given to m aintain patency of the PDA.
28 Transposition of the great vessels Prostaglandin

29
30
31 lijl;fiiJI•l toryea r:[2017 • ]
FIRST AID FACTS
32
33 FA17 p288.1
34 Congenital heart diseases
35 RIGHT·TO-LEFT SHUNTS Early cyanosis-"blue babies." Often diagnosed The 5 Ts:
36 prenatally or become evident immed iately l. Truncus arteriosus (l vessel)
after birth . Usually require urgent surgical 2. Transposition (2 switched vessels)
37
treatment and/or maintenance of a PDA. 3. Tricuspid atresia (3 = Tri)
38
4. Tetralogy of Fallot (4 = Tetra)
39 5. TAPVR (5 letters in the name)
40 Persistent truncus Truncus arteriosus fails to divide into
41 arteriosus pulmonary trunk and aorta due to lack of
42 aorticopulmonary septum formation; most
43 patients have accompanying VSD.
44 D-transposition of Aorta leaves RV (anterior) and pulmonary trunk
great vessels leaves LV (posterior) - separation of systemic
45 ' .. " . • -1 l

6
lock
s
Suspend
0
End Block
Item: 45 of 48 ~ 1 • M k -<:J 1>- Jil ~· !:';-~
QIO: 5165 ..L ar Pre v ious Next Lab~lues Notes Calcula t o r
A
-- - . . A

25 circulation; arteries erode ribs ..... notched


26 appearance on CXR. Complications include
27 HF, t risk of cerebral hemorrhage (berry
aneurysms}, aortic rupture, and possible
28
endocarditis.
29
30
FA17 p 290.1
31
Congenital cardiac DISORDER DEFECT
32 defect associations Alcohol exposure in utero (fetal alcohol SD, PDA. ASD, terralogy of Fallot
33 syndrome)
34
Congenital rubella PDA, pulmonary artery stenosis, septal defects
35
Down syndrome AV septal defect (endocardial cushion defect),
36 VSD,ASD
37 Infant of diabetic mother Transposition of great vessels
38
Marfan syndrome MVP, thoracic aortic aneurysm and dissection,
39 aortic regurgitation
40 Prenatal lithium exposure Ebstein anomaly
41 Turner svndrome Bicuspid aortic valve, coarctation of aorta
42 -
Williams syndrome Supravalvular aortic stenosis
43
22qll syndromes Truncus arteriosus, tetralogy of ~allot
44
45 •
a
Lock
s
Suspend
8
End Bl ock
Item: 46 of 48 ~ 1 • M k -<:J 1>- Jil ~· !:';-~
QIO: 1413 ..L ar Pre v ious Next Lab~lues Notes Calcula t o r
A A

26 A 56-year-old African-American woman with diabet es presents to the emergency department with diaphoresis and shortness of breath. Her blood
pressure Is 120/82 mm Hg, pulse is 108/min, and respiratory rate is 22/mln. An ECG shows ST-segment elevations and T-wave inversions in the
27
anterolateral leads. The patient is admitted and trea t ed for a myocardial Infarction.
28
29 Which of the following circumstances produces t he same type of tissue necrosis?

30
31 A. Complete occlusion of a renal artery

32 B. Focal bacterial infection w ithin th e lung


33
C. Hypoxic cell death within the central nervous system
34
D. Immune-mediated reactions within blood vessels
35
36 E. Inflammation associated with Mycobacterium tuberculosis infection

37
38
39
40
41
42
43
44
45
. 46 •
a
Lock
s
Suspend
8
End Bl ock
Item: 46 of 48 ~ 1 • M k -<:J 1>- Jil ~· !:';-~
QIO: 1413 ..L ar Previous Next Lab~lues Notes Calculat or
A A

26
Th e co rrect a n sw er i s A. 660/o ch ose this.
27 After myocardial infarction, dead tissue undergoes coagulative necrosis (shown In the image). Prolonged
Interruption of blood flow (the ischemic event) results in protein and enzymatic denaturation. As a result,

I
28
the general histolog ic architecture of the cell is preserved except for nuclear changes and increased
29 cytoplasmic binding of acidophilic dye. Over time, scavenger leukocytes remove dead cell debris from the
area and prom ote scar formation. This type of necrosis is seen in organs supplied by end arteries, such as
30 heart and kidneys.
31 Myoca doa fa ction Coagulative necrosis Protein White blood eel, Necrosis Ischemo Histology l<.odney Artery
Denaturation (biochemistry) Enzyme Cytoplasm Infarction
32
33
34
35
Image courtesy of Wikimedia Commons
36
37 B is not co rrect. 7 0/o chose t his.
38 Focal bacterial or fungal infections often result in localized regions of liquefactive necrosis. The host's immune response to such infections results In the
release of potent enzymes that inactivat e and digest microbes and nearby t issues.
39 Immune system Microorganism Fungus Liquefactive necrosis Necrosis Mycosis Enzyme

40 C is not co rrect . 130/o ch ose this.


41 Hypoxic cell death within the central nervous system (CNS) results in liquefactive necrosis. In this pattern of cell dea th, liberat ed intracellular enzymes
degrade dead tissues, producing a viscous m ass. The specific mechanism of liquefactive necrosis subsequent to hypoxic CNS injury is unknown.
42 Central nervous system Hypoxia (medical) Necrosis Apoptosis Liquefactive necrosis Hypoxia (environmental) Nervous system Intracellular Enzyme
43 D i s n ot correct. aoto ch ose this.
44 Deposit ion of antigen-antibody complexes in vessel walls results in leakage of fibrin from epithelial cells and the development of fibrinoid necrosis. This
pattern of necrosis is seen frequently in vasculitic syndromes.
45 Fibrinoid necrosis Fibrin Necrosis Immune complex Vasculitis Epithelium

46 • E i s not correct. 6 0/o chose t his•

a
Lock
s
Suspend
8
End Bl ock
Item: 46 of 48 ~ 1 • M k -<:J 1>- Jil ~· !:';-~
QIO: 1413 ..L ar Pre v ious Next Lab~lues Notes Calcula t o r
A Fibrinoid necrosis Fibrin Necrosis Immune complex vasculitis Epithelium A

26
E i s n o t co rrect . 60/o chose this.
27
Mycobacterium tuberculosis infection, in any tissue, is associated with caseous necrosis (like that shown in
28 the Image). In this pattern of destruction, the necrotic tissue is granular and "cheese-like" and is
surrounded by a rmg of inflammatory cells, forming the granulomas characteristic of this disease process.
29 Mycobacterium tuberculosis Caseous necrosis Tuberculosis Necrosis Granuloma Mycobacterium Infection
30
31
32
33
I
34
35
36
37
38
39
40
41
(

42 Image courtesy of Dr. Robert w. Novak


43
44
Bo tto m Line:
45
Coagulative necrosis occurs early (within hours) after the onset of a myocardial infarction. The various types of necrosis (coagulative, liquefactive,
46 • caseous. fat. fibrinoid. and aanarenous) occur characteristically in soedfic oraan svstems.

a
Lock
s
Suspend
8
End Bl ock
Item: 46 of 48 ~. I • M k <:] t> al ~· ~
QIO: 1413 .l. ar Previous Next lab 'Vfl1ues Notes Calculator
• Image courtesy of Or. Robert w. Novak
26
27
28
Botto m Line:
29 Coagulative necrosis occurs early (within hours) after the onset of a m yocardial infarction. The va rious t ypes of necrosis (coagulative, liquefactive,
30 caseous, fat, fibrinoid, and gangrenous) occur characteristically in specific organ systems.
Coagulative necrosis Myocardial infarction Necrosis Gangrene Infarction
31
32
33
I iii I;fi 1!1 I•J fo r year:l 20 17 ..
FI RST AI D FA CTS
34
35
FA17 p205.1
36
Necrosis Enzymatic degradation and protein denaturation of cell due to exogenous injury - intracellular
37
components leak. Inflammatory process (unl ike apoptosis).
38
TYPE SEEN IN DUETO HISTOLOGY
39
Coagulative Ischemia/infarcts in Ischemia or infarction; Cell outl ines preserved but nuclei disappear;
40 most tissues (except proteins denatu re, then t cytoplasmic binding of eosin dyes ·
41 brain) enzymatic degradation
42 Liquefactive Bacterial abscesses, Neutrophils release Early: cellular debris and macrophages
43 brain infarcts lysosomal enzymes Late: cyst ic spaces and cavitation (brain)
44 that digest the Ncutroph ils and cell debris seen with
tissue III; enzymatic bacterial infection
45
degradation first, then
46 •

6
lock
s
Suspend
0
End Block
27
28
FA17p207.1
29
Types of infarcts
30
Red infarct Red (hemorrhagic) infarcts occur in ' enous occlusion and tissues "ith multiple blood supplies,
31 such as li,·er, lung, intestine, testes; reperfusion (eg, after angioplasty). Reperfusion injury is due to
32 damage by free radicals.
33 Red = rcperfusion.
34
35
36
37
38

I
Pale infarct Pa le (anemic) infarcts [l] occur in solid organs with a single (end-arterial) blood supply, such as
39
heart, kidney, and spleen.
40
41
42
43
44
45
46 •
a
Lock Suspend
s 8
End Block
Item: 46 of 48 ~ 1 • M k -<:J 1>- Jil ~· !:';-~
QIO: 1413 ..L ar Pre v ious Next Lab~lues Notes Calcula t o r
lf
26
27
FA17 p294.1
28
29 Evolution of Commonly occluded coronar) arteries: I.AD > RCA> circumnex.
30
myocardial infarction S} mptoms: diaphoresis, nausea, \Omiting, se,ere retrosternal pain, pain in left arm and/or ja\\,
shortness of breath, fatigue.
31
TIME GROSS LIGHT MICROSCOPE COMPLICATIONS
32
0 - 24 hr None Early coagulati' e necrosis, entricular arrhythmia, II F',
33
release of necrotic cell cardiogenic shock.
34 contents into blood; edema,
35 hemorrhage, wavy fibers.
Occluded eutrophils appear.
36
Rcpcrfusion injury,
37 a~sociated "ith generalion
38 of free rad icals, leads to
39 ""i'i~ Dark mottling. hypercontraction of myofibrils
pale with through t Free calcium in Aux.

-
40 tetrazolium
stain

- - I
41
42 Q
Q
43
44
45
46 • .... ~ --~ ... 0
I '' n .. r .. rt ·

a
Lock
s
Suspend
8
End Bl ock
Item: 47 of 48 ~ 1 • M k -<:J 1>- Jil ~· !:';-~
QIO: 3818 ..L ar Pre v ious Next Lab~lues Notes Calcula t o r
A A

27 An 11-year-old child has a 1-month history of bluish-gray discoloration and a several-week history of dyspnea and decreased physical activity. The
child's birth was unremarkable and he has otherwise been healthy and active. Heart rate is 96/min, respiratory rate is 20/min, and blood pressure Is
28
110/72 mm Hg. There is moderate clubbing of t he distal extremities and a continuous murmur, best hea rd at the left second and third intercostal
29 spaces. Complete blood cell count revea ls:

30 wac count : 8200/ mm3


Hemoglobin: 17.3 g/ dl
31 Hematocrit: 54.2%
32 Platelets: 22S,OOO/ mm3

33
34 Which of the following is the most likely cause of the patient's cond ition?
35
36 A. Patent ductus arteriosus

37 B. Tetralogy of Fallot
38
c. Transposition of the great vessels
39
40 o. Tricuspid atresia

41 E. Truncus arteriosus

42
43
44
45
46
. 47 •
a
Lock
s
Suspend
8
End Bl ock
Item: 47 of 48 ~ 1 • M k -<:J 1>- Jil ~· !:';-~
QIO: 3818 ..L ar Previous Next Lab~lues Notes Calculat or
A A

27
28
29
30
The correct a n sw er i s A. 700/o chose this.
Left-to-right shunts are classically distinguished clinically from right-to-left shunts in that the former are not associated with early cyanosis ("blue babies").
Instead, left-to-right shunts are associated with late cyanosis ("blue kids"). With time, a child with a left-to-right shunt will develop progressive pulmonary
hypertension, leadmg to right ventricular hypertrophy. Eventually, the pressure on the right side of the heart can exceed that on the left side and reverse
I
31 the left-to-right shunt to a right-to-left shunt. Such a pathophysiologic shunt reversal is referred to commonly as Eisenmenger syndrome. The causes of a
left-to-right shunt, in order of decreasing frequency, are ventricular septal defect, atrial septal defect, and patent ductus arteriosus (PDA). Of these, PDA
32 Is the only choice presented in this question. PDA presents with a continuous " machine-like" murmur.
Atr a sep defect Ventricular septal defect Eisenmenger's syndrome Cyanosis Pate ductus arteriosus Pulmonary hypertension Right-to-left shunt
33
Right ventricular hypertrophy Ductus arteriosus cardiac shunt Hypertension Ventricular hypertrophy Hypertrophy Heart Heart murmur Atrium (heart)
34 Ventricle heart Shunt 'medical)
35 B i s not co rrect. 140/o chose this.
36 Tetralogy of Fallot (TOF), the most common cause of early cyanosis In congenital heart disease, Is a form of right -to -left shunt in which poorly oxygenated
blood on the right side of the heart mixes with freshly oxygenated blood on the left side of the heart. Because of early mixing of pulmonary and systemic
37 circulations, Infants appear cyanotic shortly after birth. At times TOF can present later In childhood, especially if the pulmonary outflow obstruction Is not
38 massive. Such children often experience the need to crouch down after physical activity to Increase peripheral vascular resistance to shunt blood to the
lungs. In this case, the lack a history of squatting combined with the absence of cyanosis at birth make TOF less likely.
39 Tetralogy of Fallot Cyanosis Right-to-left shunt Vascular resistance Congenital heart defect Cyanotic heart defect Congenital disorder Cardiovascular disease
40 Ventricle (heart) Shunt (medical) Lung Heart

41 C is not co rrect. 40/o ch ose this.


Transposition of the great vessels is characterized by a right-to-left shunt; these patients present with cyanosis ea rly in life, although with transposition of
42 the great arteries cyanosis may be limited to exercise (feeding or crying) . The aorta comes off of the right ventricle, thus returning deoxygenated blood to
43 the systemic circulation; the pulmonary artery comes off of the left ventricle, returning oxygenated blood to the pulmonary circulation . various other
cardiac abnormalities may be present in these patients, including patent ductus arteriosus or a ventral septal defect resulting in murmurs. Treatment Is
44 surgical.
Transposition of the great vessels Cyanosis Patent ductus arteriosus Pulmonary circulation Right-to-left shunt Pulmonary artery Aorta ventricle (heart)
45
Ductus arteriosus Circulatory system Great vessels Shunt (medical} Artery Great arteries Anatomical terms of location
46
D i s not correct. 6 0/o chose this.
47 • Tl"'irr•cnlrl :.troci::. ic rh.:::al""::'rl"ori?orf hv .::.non.:~o~ic: nf the trirr rc:nirl \.t::.h.1o rocrrltlnn In::. hvnnnl;::.c:;tir rinht v ontrirlo .:::. rinht- tn- loft chr rnt nf hlnnrt frnm t-ko loft

a
Lock
s
Suspend
8
End Block
Item:47of48 ~ ., . M k <:] t> al ~· ~
QIO: 3818 .l. ar Previous Next lab 'Vfl1ues Notes Calculator

27 D is not co rrect. 6 % chose this .


Tricuspid atresia is characterized by agenesis of the tricuspid valve, resulting in a hypoplastic right ventricle, a right-to -left shunt of blood from the left
28
ventricle to the pulmonary circulation via a ventricular septal defect (VSD ), and onset of cyanosis shortly after birth. An atrial septal defect also must be
29 present to allow blood to flow from the right atrium into the left atrium, the left ventricles, and back into the pulmonary circulation via the VSD.
Atrial septal defect Tricuspid atresia Ventricular septal defect Cyanosis Tricuspid valve Right-to-left shunt Atrium (heart) Pulmonary circulation Ventricle (heart)
30

I
Atresia Shunt (medical) Hypoplasia
31
E is not co rrect. 6 % chose this .
32 Truncus arteriosus is characterized by a right -to -left shunt early in life. Anatomically, it results in one grea t vessel that gives off the pulmonary, coronary,
and systemic arteries, thus mixing oxygenat ed and deoxygenat ed blood. Trea tment is surgical.
33
Right-to-left shunt Truncus arteriosus Great vessels Circulatory system Shunt (medical) Artery
34
35
Botto m Li ne:
36
A PDA is a congenital heart defect that causes a left -to -right shunt, resulting in pulmonary hypertension and hypoxia. Left untrea t ed, a left -to -right
37 shunt will eventually reverse to a right-to -left shunt, lea ding to lat e cyanosis associat ed with clubbing and polycythemia (ie, Eisenmenger syndrome).
The murmur associat ed with it is continuous and machine -like in nature.
38 Eisenmenger' s syndrome Cyanosis Congenital heart defect Pulmonary hypertension Hypoxia (medical) Right-to-left shunt Polycythemia Hypertension
Shunt (medical) Congenital disorder Cardiac shunt Heart murmur
39
40
41
lijl;fiiJI•l to r yea r:[ 20 17 • ]
42 FI RST AI D FA CTS

43
44 FA17 p288.1

45 Congenital heart diseases


46 RIGHT·TO·LEFT SHUNTS Early cyanosis-"blue babies." Often diagnosed The 5 Ts:
prenatally or become evident im med iately l. Truncus arteriosus (l vessel)
47 r. .. '
'1• . 1 T T 1 1
·~ ' '

6
lock
s
Suspend
0
End Block
Item: 47 of 48 ~ 1 • M k -<:J 1>- Jil ~· !:';-~
QIO: 3818 ..L ar Pre v ious Next Lab~lues Notes Calcula t o r
A •• • A

27
28
FA17 p288.1
29 Congenital heart diseases
30 RIGHI·IO·LEFT SHUNIS Early cyanosis-"blue babies." Often diagnosed The 5 Ts:
31 prenatally or become e"ident immediately 1. Truncus arteriosus (1 ,·essel)
32 after birth. Usually require urgent surgical 2. Transposition (2 switched vessels)
33
treatment andlor maintenance of a PDA. 3. Tricuspid atresia (3 = Tri)
4. 'Jetralogy of Fallot (-f = Tetra}
34
5. T. P R (5 letters in the name)
35
Persistent t runcus Truncus arteriosus fails to divide into
36
arteriosus pulmonary trunk and aorta due to lack of
37 aorticopulmonary septum formal ion; most
38 patients have accompanying VSD.
39 D-transposition of Aorta leaves RV (anterior) and pulmonary trunk
40
41
great vessels leaves LV (posterior) .... separation of systemic
and pulmonary circulations. Not compatible
with li fe unless <1 shunt is present to allow
I
42
mixing of blood (eg, VSD, PDA, or patent
43 foramen ovale). Right
ventricle
44 Due to failure of the aorticopulmonary septum
45 to spiral.
46 Without surgical inten·ention , most infants die Ventricular
within the first few months of life. septum
47 •
a
Lock
s
Suspend
8
End Bl ock
Item: 48 of 48 ~ 1 • M k -<:J 1>- Jil ~· !:';-~
QIO: 5 0 83 ..L ar Pre v ious Next Lab~lues Notes Calcula t o r

28
A

An 81-year-old previously healthy man presents with several months of progressive dyspnea on exertion and fatigue . On physical examination, he has
a low-amplitude carotid pulse and the murmur heard in the audio clip .
IA•A] A

29
30 OPEN MEDIA

31
32 II - ooOO i 0009 II>)

33
34
What Is the most likely pathophysiology behind his signs and symptoms?
35
36 A. Calcification of valve cusps due to pro liferative and inflammatory changes
37
B. Commissural fusion due to adhesions secondary to rheumatic heart disease
38
39
c. Valve perforation due to endocarditis

40 o. Valve prolapse due to myocardial infarction

41 E. Ventricular septal defect due to myocardial infarction


42
43
44
45
46
47
. 48

a
Lock
s
Suspend
8
End Bl ock
Item: 48 of 48 ~. I • M k <:] t> al ~· ~
QIO: 5083 .l. ar Previous Next lab 'Vfl1 ues Notes Calculator

28
The co rrect a nswer is A. 74% cho se this.
29 Aortic valve thickening (sclerosis) without stenosis is common in elderly adults. Aortic sclerosis is caused by calcification of the valve cusps from
30 proliferative and inflammatory changes that mimic those that cause calcification within the va sculature. It can often be det ected as a systolic ej ection
murmur on physical exam . Aortic valve sclerosis can progress to aortic stenosis (AS ) in adults, resulting in increa sed cardiovascular risk. In this question,
31 the murmur is a crescendo -decrescendo systolic murmur which is characteristic of AS . Signs and symptoms of AS include progressive dyspnea on exertion,
fatigue, syncope, angina, low -volume lat e pulses, and left ventricular hypertrophy.
32 Dyspnea left ventricular hypertrophy Aortic valve Aortic stenosis Ventricular hypertrophy Syncope (medicine) Systole Stenosis Angina pectoris Calcification
33 Hypertrophy Circulatory system Physical examination Heart murmur Heart Ventricle (heart) Fatigue (medical)

34 B is no t co rrect. 8% cho se this.

35 Commissural fusion from adhesions secondary to rheumatic heart disea se would produce this murmur; but it is much less common than calcific AS .
Rheumatic fever Cardiovascular disease Commissure Rheumatism Heart murmur
36
c is no t co rrect. 5 % cho se this.
37 Valve perforation from endocarditis does not fit because that murmur is a diastolic murmur when in the aortic valve, and holosystolic when in the tricuspid
38 or mitral va lves.
Endocarditis Aortic valve Diastole Tricuspid valve Heart murmur Mitral valve
39
D is no t co rrect. 8 % cho se this.
40 Valve prolapse from myocardial infarction typically affects the mitral va lve and causes a lat e systolic crescendo murmur with a midsystolic click.
41 Myocardial infarction Mitral valve Systole Prolapse Heart murmur

42 E is no t co rrect. 5 % cho se this.


A ventricular septal defect from myocardial infarction would cause a harsh holosystolic murmur often radiating from left to right .
43
Ventricular septal defect Myocardial infarction Ventricular system Ventricle (heart) Holosystolic Heart murmur
44
45
Botto m Li ne:
46
The most common cause of aortic stenosis is calcification of valve cusps, and it causes a crescendo-decrescendo systolic murmur.
47 Aortic stenosis Systole Stenosis Calcification Systolic heart murmur Heart murmur

48

6
lock
s
Suspend
0
End Block
Item: 48 of 48 ~. I • M k <:] t> al ~· ~
QIO: 5083 .l. ar Previous Next lab 'Vfl1 ues Notes Calculator

28
29 Bo tto m Line:
30 The most common cause of aortic stenosis is calcification of va lve cusps, and it causes a crescendo-decrescendo systolic murmur.
Aortic stenosis Systole Stenosis Calcification Systolic heart murmur Heart murmur
31
32
33
lijj ;fi IJ l•l f o r year:l 20 1 7 ..
34 FIRST AID FACTS

35
36 FA17p279.1

37 Heart murmurs

38 Systolic

39 Aortic stenosis Crescendo-decrescendo systolic ejection murmur (ejection click may be present).
40
LV>> aortic pressure during systole. Loudest at heart base; radiates to carotids.
Sl 52
"Pulsus pan·us et tardus"- pulses are weak with a delayed peak. Can lead to
41
42
~I Syncope, Angina, and Dyspnea on exertion (SAD). Most commonly due to age-
related calcification in older patients (> 60 years old) or in younger patients with
43 early-onset calcification of bicuspid aortic vah-e.
44 Mitral/tricuspid regurgitation Holosystolic, high-pitched "blowing murmur."
45
51 52
Mitral-loudest at apex and rad iates toward axilla. MR is often due to ischemic heart
disease (post-MT), MVP, LV dilatation.
~
46
Tricuspid- loudest at tricuspid area. TR commonly caused by RV dilatation.
47
Rheumatic fever and infective endocarditis can cause either IR or TR.
48 . '. ' • I • 1 • 1 • I • 1 ~ -- .- - t
' '

6
lock
s
Suspend
0
End Block
Item: 48 of 48 ~. I • M k <:] t> al ~· ~
QIO: 5083 .l. ar Previous Next lab 'Vfl1 ues Notes Calculator

28 ll ,,.·w w

29
30
FA17 p208.1
31
Types of calcification Dystrophic calcification Metastatic calcification
32
CAl+ DEPOSITION ln abnormal tissues In normal tissues
33
EXTENT Tends to be localized (eg, calcific aortic stenosis) Widespread {ie, diffuse, metastatic)
34
fJ shows dystrophic calcification (yellow star), [l] shows metastatic calcifications of alveolar
35
and thick fibrot ic wall (red arrows) walls in acute pneumonitis (arrows)
36
ASSOCIATED CONDITIONS TB (lung and pericardium) and other Predom inantly in interstitial tissues of kidney,
37 granulomatous infections; I iquefactive necrosis lung, and gastric mucosa (these tissues lose
38 of chron ic abscesses; fat necrosis; infarcts; acid quickly; t pH fa,ws Ca 2+ deposition)
39 thrombi; schistosomiasis; congenital CMV, Nephrocalcinosis of collecting ducts may lead
40
toxoplasmosis, rubella; psammoma bodies; to nephrogenic diabetes insipidus and renal
CREST syndrome failure
41
42
ETIOLOGY 2° to injury or necrosis zoto hypercalcemia (eg, 1° hyperparathyroidism,
sarcoidosis, hypervitaminosis D) or high
43
calcium-phosphate product levels {eg, chronic
44 renal fai lure with zohyperparathyroidism,
45 long-term dialysis, calciphylaxis, multiple
46 myeloma)
47 SERUM CA1+ LEVELS Patients are usually normocalcemic Patients are usually not normocalcemic
48

6
lock
s
Suspend
0
End Block
Item: 48 of 48 ~ 1 • M k -<:J 1>- Jil ~· !:';-~
QIO: 5 0 83 ..L ar Pre v ious Next Lab~lues Notes Calcula t o r

g ~ g y
28
p
toxoplasmosis, rubella; psammoma bodies; to nephrogenic diabetes insipidus and renal
29
CREST syndrome failure
30
ETIOLOGY zoto injury or necrosis zoto hypercalcemia (eg, l 0
hyperparathyroidism,
31 sarcoidosis, hypervitaminosis D) or high
32 calcium-phosphate product b ·els (eg, chronic
33 zo
renal failure "ith hyperparath~ roidism,

34 long-term dialysis, calciphylaxis, multiple


nweloma)

35
SERUM CA1+ LEVELS Patients are usually normocalcemic Patients are usually not normocalcemic
36
37
38
39
40
41
42
43
44
45
46
47
48 •

a
Lock
s
Suspend
8
End Bl ock

You might also like